Download as pdf or txt
Download as pdf or txt
You are on page 1of 184

Series

SeriesMAT/12/1
Series FH1EG/3
SCI/10/1 Set ~ 1
Roll No. Q.P Code 10/1/1
Candidates must write the Q.P Code
on the title page of the answer-book.

 Please check that this question paper contains 10 printed pages.


 Q.P. Code given on the right hand side of the question paper should be written on the title
page of the answer-book by the candidate.
 Please check that this question paper contains 39 questions.
 Please write down the serial number of the question in the answer-book before
attempting it.
 15 Minute time has been allotted to read this question paper. The question paper will be
distributed at 10:15 a.m From 10.15a.m to 10.30 a.m, the students will read the question
paper only and will not write any answer on the answer –book during this period

SCIENCE
Time Allowed : 3 hours Maximum Marks : 80

General Instructions:

i. This question paper consists of 39 questions in 5 sections.


ii. All questions are compulsory. However, an internal choice is provided in some questions.
A student is expected to attempt only one of these questions.
iii. Section A consists of 20 objective type questions carrying 1 mark each.
iv. Section B consists of 6 Very Short questions carrying 02 marks each. Answers to these
questions should be in the range of 30 to 50 words.
v. Section C consists of 7 Short Answer type questions carrying 03 marks each. Answers to
these questions should be in the range of 50 to 80 words.
vi. Section D consists of 3 Long Answer type questions carrying 05 marks each. Answer to
these questions should be in the range of 80 to 120 words.
vii. Section E consists of 3 source - based/case - based units of assessment of 04 marks
each with sub - parts

1 P.T.O
Section A 20 Marks
1 What do you observe when sodium sulphate is added to barium chloride
solution?

a) Transparent solution is formed

b) Bubbles are seen

c) Gas is released

d) A white insoluble substance is formed


ℎ𝑒𝑎𝑡
2 2FeSO4 → Fe 2 O 3 + SO 2 + SO 3

The above reaction is

a) Double displacement reaction


b) Combination reaction

c) Displacement reaction

d) Decomposition reaction

3 One of the constituents of baking powder is sodium hydrogencarbonate, the


other constituent is

a) Acetic acid

b) Sulphuric acid

c) Tartaric acid

d) Hydrochloric acid

4 Electrical wires have a coating of an insulting material. The material, generally


used is

a) PVC

b) Sulphur

c) All can be used

d) Graphite

2 P.T.O
5 You want to test for hardness of water but hard water is not available in the
laboratory. Which of the following compounds may be dissolved in pure water to
make it hard?

1. Hydrogen Carbonate of Sodium

2. Sulphate of Magnesium

3. Chloride of Calcium

4. Carbonate of Sodium

a) (i) and (ii)

b) (ii) and (iii)

c) (i) and (iv)

d) (iii) and (iv)

6 Name the blood vessel which carries deoxygenated blood from the heart to the
lungs.

a) Capillaries
b) Pulmonary vein

c) Pulmonary artery

d) Aorta

7 The hormone secreted by the thyroid gland is

1. Thyroxin

2. Calcitonin

3. Adrenaline

4. Insulin

a) B and C

b) A and B

c) A, B and D

d) All of these

3 P.T.O
8 Posture and balance of the body is controlled by.

a) Pons

b) Medulla

c) Cerebrum

d) Cerebellum

9 The speed of light in water is 2.25× 10 8 m/s.If the speed of light in a vacuum be
3.00 × 10 8 m/s, the refractive index of water:

a) 1.33

b) 1.34

c) 1.35

d) 1.32

10 Four optical media W, X, Y and Z have optical densities 1.35, 1.21, 1.58 and 1.002
respectively. In which optical medium will the light travel fastest?

a) W

b) Y
c) Z

d) X

11 The laws of reflection hold true for:


a) convex mirrors only

b) concave mirrors only

c) all reflecting surfaces

d) plane mirrors only

12 An achromatic glass used for end - vision on a car with a radius of 4.00 m. If a car
is situated at 6.00 m from this achromatic glass, find the position of the image.

a) 4 m

b) 3 m

c) 5 m

d) 2 m

4
P.T.O
13 A 2 kW heater, a 200 watt TV and three 100 watt lamps are all switched on from
6−10 PM. What is the total cost at Rupees 5.50 per kWh?

a) 55

b) 57

c) 56

d) 67

14 One coulomb charge is equivalent to the charge obtained in:


a) 6.2× 10 19 electrons

b) 6.25× 10 18 electrons

c) 2.6× 10 19 electrons

d) 2.65× 10 18 electrons

15 If two resistors of 25𝛺 and 15 𝛺 are joined together in series and then placed in
parallel with a 40 𝛺 resistor, the effective resistance of the combination is:

a) 0.1𝛺

b) 10𝛺
c) 0𝛺

d) 40𝛺

16 The slope of V - I graph gives


a) Potential difference

b)) urrent

c) Conductivity

d) Resistance

17 Assertion (A): Chemical reaction changes the physical and chemical state of a
substance.

Reason (R): When electric current is passed through water (liquid), it


decomposes to produce hydrogen and oxygen gases.

a) Both A and R are true and R is the correct explanation of A.

b) Both A and R are true but R is not the correct explanation of A.

c) A is true but R is false.

d) A is false but R is true.

5 P.T.O
18 Assertion (A): Ina candle, wax vapours burn an insufficient supply of oxygen,
which leads to a blue flame.

Reason (R): When the oxygen supply is sufficient, then fuels burn completely
producing a blue flame.

a) Both A and R are true and R is the correct explanation of A.

b) Both A and R are true but R is not the correct explanation of A.

c) A is true but R is false.

d) A is false but R is true.

19 Assertion (A): The walls of atria are thicker than those of the ventricles.
Reason (R): Ventricles have to pump blood into various organs at high pressure.

a) Both A and R are true and R is the correct explanation of A.

b) Both A and R are true but R is not the correct explanation of A.

c) A is true but R is false.


d) A is false but R is true.

20 Assertion (A): Seismonastic movement shown by Mimosa pudica plant.


Reason (R): It is due to change in turgidity of cells of pulvinus.

a) Both A and R are true and R is the correct explanation of A.

b) Both A and R are true but R is not the correct explanation of A.

c) A is true but R is false.

d) A is false but R is true.

Section B 12 Marks

21 A silver article generally turns black when kept in the open for a few days. The
article when rubbed with toothpaste again starts shinning. Why do silver articles
turn black when kept in the open for a few days? Name the phenomenon
involved.
22 What are the various ways in which an atom can achieve the noble gas
configuration?

6 P.T.O
OR

1. Why is𝑍𝑛𝑂 called an amphoteric oxide? Give another example.

2. What are alkalis? Give one example of alkali.

23 What happens when a molecule of hydrogen is added to ethyne? Explain.


24 Write a note on dental caries.
25 What are growth regulators?
OR

Define neurons. Identify the parts of a neuron at which:

1. information is all together collected.

2. impulses are converted to chemical signal for further transmission.

26 Redraw the diagram given below in your answer book and complete the path of
ray.

1.

2.

Section C 21 Marks

27 State one characteristic each of the chemical reaction which takes place when:
1. Dilute hydrochloric acid is added to sodium carbonate.

2. Lemon juice is added gradually to potassium permanganate solution.

3. Dilute sulphuric acid is added to the barium chloride solution.

4. Quick lime is treated with water.

5. Wax is burned in the form of a candle.

7 P.T.O
28 A compound which is prepared from gypsum has the property of hardening
when mixed with a proper quantity of water. Identify the compound. Write the
chemical equation for its preparation. For what purpose is it used in hospitals?
29 ’M’ is an element which may be one out of Cu, Fe, Al, Na. It shows the following
properties:

(i) One of its ore is rich in 𝑀2 𝑂3 .

(ii) 𝑀2 𝑂3 is not affected by water.

(iii) It corrodes easily.

(iv) It form to chlorides 𝑀𝐶𝑙2 and 𝑀𝐶𝑙3 . Identify ’M’.

30 1. What is the name of next homologue of C5 H 11 COOH?

2. C60 and C 70 are important members of which type of allotropeof carbon?

31 "If there were no algae there would be no fish in the sea." Comment.
32 Nervous and hormonal system together performs the functions of control and
coordination in human beings. Justify the statement.
33 1. State Joule’s law of heating. Express it mathematically when an appliance
of resistance R is connected to a source of voltage Vand the current I flows
through the appliance for a time t.

2. A 5𝛺 resistor is connected across a battery of 6 volts. Calculate the energy


that dissipates as heat in 10s.

Section D 15 Marks

34 Compare nervous and hormonal mechanism for control.


OR

’Nervous and hormonal systems together perform the function of control and
coordination in human beings.’ Justify the statement.

35 Form the image in case an object is moved from infinity to the concave mirror.
OR
1. How are the images formed in convex mirror when object is moved from
infinity to the mirror?

8 P.T.O
36 1. For the combination of resistors shown in the following figure, find the
equivalent resistance between M & N.

2. State Joule’s law of heating.

3. Why we need a 5 A fuse for an electric iron which consumes 1 kW power


at 220 V?

4. Why is it impracticable to connect an electric bulb and an electric heater


in series?

OR

In an electric circuit three bulbs of 100 W each are connected in series to a


source. In another circuit set of three bulbs of the same wattage are connected in
parallel to the same source.

5. Will the bulb in the two circuits glow with the same brightness? Justify
your answer.

Now, let one bulb in both the circuits get fused. Will the rest of the bulbs continue
to glow in each circuit? Give reason for your answer.

Section E 12 Marks

37 Read the text carefully and answer the questions: Redox reactions are those
reactions in which oxidation and reduction occur simultaneously. A redox
reaction is made up of two half reactions. In the first half reaction, oxidation takes
place and in second half reaction, reduction occurs. Oxidation is a process in
which a substance loses electrons and in reduction, a substance gains electrons.
The substance which gains electrons is reduced and acts as an oxidising agent. On
the other hand, a substance which loses electrons is oxidised and acts as a

reducing agent.

1. Illustrate where oxidation and reduction occurs together with an


example?

2. What do you mean by oxidising agent and reducing agent?

3. For the given reaction, identify the oxidation and reduction parts.

ZnO + CO → Zn + CO 2

4. In the following reaction, which substance is reduced?

PbS + 4H 2 O 2 → PbSO 4 + 4H 2 O

9 P.T.O
OR

Illustrate where oxidation and reduction occurs together with an


example(which should not be in question paper)?

38 Read the text carefully and answer the questions: Non - metals are either
solids or gases. Non - metal can exist in different forms such as carbon. Each form
is called allotrope. Alkali metal is so soft that it can be cut with a knife. They have
low density and low melting point. Some metal can melt if they are kept in the
palm.

1. What are the physical properties of metals?


2. Differentiate between the physical properties of metals and non metals ?
3. Give some exception of metals which does not obey the properties of
metals?
4. Why is sodium always kept in kerosene oil?

OR

Which salt is used for removing the permanent hardness of water?

39 Read the text carefully and answer the questions: The compounds which have
the same molecular formula but differ from each other in physical or chemical
properties are called isomers and the phenomenon is called isomerism. When the
isomerism is due to difference in the arrangement of atoms within the molecule,
without any reference to space, the phenomenon is called structural isomerism.
In other words, structural isomers are compounds that have the same molecular
formula but different structural formulas, i.e., they are different in the order in
which different atoms are linked. In these compounds, carbon atoms can be
linked together in the form of straight chains, branched chains or even rings.

1. Write two compounds having same molecular formula with there


structure?

2. Butane and 2 - methylpropane have same molecular formula but different

structural formula. Explain.


3. Which have longest chain iso - pentane or 2 - methylpentane?
4. Draw all possible isomers of pentane.
OR

How will you obtain sulphuric acid from an acidic oxide?

10 P.T.O
Maximum Marks: 80 Time Allowed: 3 hours
General Instructions:
i. This question paper consists of 39 questions in 5 sections.

ii. All questions are compulsory. However, an internal choice is provided in some
questions. A student is expected to attempt only one of these questions.

iii. Section A consists of 20 objective type questions carrying 1 mark each.

iv. Section B consists of 6 Very Short questions carrying 02 marks each. Answers to
these questions should be in the range of 30 to 50 words.

v. Section C consists of 7 Short Answer type questions carrying 03 marks each.


Answers to these questions should be in the range of 50 to 80 words.

vi. Section D consists of 3 Long Answer type questions carrying 05 marks each.
Answer to these questions should be in the range of 80 to 120 words.

vii. Section E consists of 3 source-based/case-based units of assessment of 04 marks


each with sub-parts

Multiple
Assertion & Choice Subjective
Chapter Name - Reason Question Question Total

Chemical - 1 (1) 2 (1) 1 (2) 5 (8)


Reactions and
Equations 1 (3)

Acids Bases and - - 1 (1) 1 (3) 2 (4)


Salts

11 P.T.O
Metals and Non- - - 1 (1) 1 (2) 3 (6)
metals
1 (3)

Carbon and its - 1 (1) 1 (1) 1 (2) 4 (7)


Compounds
1 (3)

Life Processes - 1 (1) 1 (1) 1 (2) 4 (7)

1 (3)

Control and - 1 (1) 2 (1) 1 (2) 6 (13)


Coordination
1 (3)

1 (5)

Light Reflection - - 4 (1) 1 (2) 6 (11)


and Refraction
1 (5)

Electricity - - 4 (1) 1 (3) 6 (12)

1 (5)

(5 Grouped 1 (4) - - - 1 (4)


Questions)
Chemical
Reactions and
Equations

(4 Grouped 1 (4) - - - 1 (4)


Questions)
Carbon and its
Compounds

12 P.T.O
(4 Grouped 1 (4) - - - 1 (4)
Questions)
Metals and Non-
metals

Total 3 4 (4) 16 (16) 16 (48) 39


(12) (80)

13 P.T.O
1.
(d) A white insoluble substance is formed
Explanation: When barium chloride combines with sodium sulphate in the form of their aqueous solutions, a white precipitate
of barium sulphate is formed which is insoluble in water. The reaction also creates sodium chloride, which remains dissolved in
water and so cannot be seen.
BaCl2(aq) + Na2SO4(aq) → BaSO4(s)(White precipitate) + 2NaCl(aq)
It is known as the precipitation reaction.
2.
(d) Decomposition reaction
Explanation: Ferrous sulphate crystals contain water molecules (FeSO4. 7H2O). On heating, ferrous sulphate crystals lose
water and anhydrous ferrous sulphate (FeSO4) is formed. So their colour changes from light green to white.
On further heating, anhydrous ferrous sulphate decomposes to form ferric oxide (Fe2O3), sulphur dioxide (SO2) and sulphur
trioxide (SO3). So, the gas emitted smells like burning sulphur.
In this reaction, the single reactant FeSO4 decomposes to form three different products. So, the reaction is a decomposition
reaction.
3.
(c) Tartaric acid
Explanation: Tartaric acid used in baking powder where it serves as the source of acid that reacts with baking soda. This
reaction produces carbon dioxide gas and lets products rise using active yeast cultures as a source of carbon dioxide gas.
4. (a) PVC
Explanation: An insulating substance is required to coat the electrical wire such as PVC as it does not allow an electric current
to pass through it.
5.
(c) (i) and (iv)
Explanation: (i) and (iv)
6.
(c) Pulmonary artery
Explanation: The pulmonary arteries carry deoxygenated blood from the right ventricle to the lungs. The pulmonary veins
carry oxygenated blood from the lungs to the left atrium.
7.
(b) A and B
Explanation: Thyroxin is one of the major hormone secreted by the thyroid gland as well as Calcitonin is a hormone secreted
by the thyroid gland that reduces the concentration of blood calcium level when it has risen to an above-normal level.
8.
(d) Cerebellum
Explanation: It coordinates and regulates muscular activity.
9. (a) 1.33
Explanation: Speed of light in vacuum = 3.00 × 108 m/s
Speed of light in water = 2.23 × 108 m/s
(speed of light in vacuum)
Refractive index of water =
(speed of light in water)

Refractive index of water = 1.33


10.
(c) Z

14 P.T.O
Explanation: The lesser the optical density, the more is the speed of light.
11.
(c) all reflecting surfaces
Explanation: The laws of reflection hold true for all the reflecting surfaces either spherical or plane. Therefore, the plane
mirror, concave mirror, and the convex mirror all will follow the laws of reflection.
12.
(b) 3 m
Explanation: 3 m
13. (a) 55
Explanation: For heater,
P = 2kWh
T = 4 hours
Energy consumed = PT = 8kWh
For TV,
P = 200 watt = 0.2kW
T = 4 hours
Energy consumed = PT = 0.8kWh
For lamps,
P = 100 watt = 0.1kW
T = 4 hours
n=3
Energy consumed = PTn = 1.2kWh
Total energy consumed = 8 + 0.8 + 1.2 = 10kWh
Cost of 1kWh = Rupees 5.50
Total cost = Rupees 55
14.
(b) 6.25 × 1018 electrons
Explanation: There are 6.25 × 1018 electrons in one-coulomb charge.
15.
(c) 20 Ω
Explanation: The resultant resistance obtained from the resistors that are placed in series combination is connected with 40
Ω in parallel such that the overall resistance obtained is equal to 20 Ω .

16.
(d) Resistance
Explanation: Slope of V-I graph represents the resistance in the conductor.
V
Resistance R = I

17.
(b) Both A and R are true but R is not the correct explanation of A.
Explanation: Both A and R are true but R is not the correct explanation of A.
18.
(d) A is false but R is true.
Explanation: The outer portion of the flame of a candle is blue. this is because, in the outer portion of the flame of a candle,
there is a sufficient amount of oxygen where complete combustion takes place.
19. (a) Both A and R are true and R is the correct explanation of A.
Explanation: Both A and R are true and R is the correct explanation of A.
20. (a) Both A and R are true and R is the correct explanation of A.
Explanation: Both A and R are true and R is the correct explanation of A.
Section B
21. When silver is kept in open for a few days, it reacts with atmosphere oxygen to form oxide. In due course of time, this react with
oxygen and hydrogen sulphide (in the atmosphere) to form silver sulphide; which appears as black layer. This phenomenon is

15 P.T.O
called tarnishing of silver.
4Ag + O2 + 2H2 S → 2Ag2 S + 2H2 O

22. An atom can achieve the noble gas configurations by transfer of electrons to / from the other atom(s) or by sharing electrons with
other atom(s).
OR
a. Zinc oxide (ZnO ) is called an amphoteric oxide as it behaves bath as an acidic oxide and a basic oxide.
ZnO + 2HC l → ZC l2 + H2 O
(BasicOxide) (Acid) (Z incChloride) water

ZnO + 2N aOH → N a2 ZnO2 + H2 O


(BasicOxide) (Base) (SodiumZ incate) water

Aluminum oxide (Al O ) is another amphoteric oxide.


2 3

b. Water soluble hydroxides of metals are known as alkalies. For example, N aOH
23. Ethyne is converted into ethene, when a molecule of hydrogen is added to it in the presence of a catalyst Ni or Pd.
N i/pd

H − C ≡ C − H(s) + H2(g) −−−→


Ethyne Heat

The reaction stops here, i.e. the obtained alkene does not react with hydrogen again because only a one molecule of hydrogen is
taken. If one more molecule of hydrogen is taken to react with ethene then C2H4 will convert into C2H6(ethane) in presence of
Ni/Pd catalyst
24. Dental caries or tooth decay causes gradual softening of enamel and dentine. It starts when bacteria acting on sugars present in
oral cavity produce acids that softens or de-mineralises the enamel. Masses of bacterial cells together with food particles stick to
the teeth to form dental plaque. Salvia cannot reach the tooth surface to neutralise the acid as plaque covers the teeth. Brushing the
teeth after eating removes the plaque before the bacteria produce acids. If untreated, microorganisms may invade the pulp, causing
inflammation and infection.
25. Growth is regulated not only by environmental factors like light and temperature, but also by certain chemical substances within
the plants. These substance are known as plant hormones, growth hormones, phytohormones or growth regulators. A plant
hormone can be defined as a chemical substance, which is capable of translocation and regulating one or more physiological
reactions, when present in low concentrations. The plant hormones can have a positive effect on process and thus promote it, or
they have a negative effect and cause inhibition. A particular hormone may promote certain processes and inhibit some other
processes.
OR
The structural and functional units of nervous system are called neurons. It means the units which makes up the nervous system
are called neurons. These are specialized units for acceptance and transmission of impulses to and fro between the brain and other
body parts.
a. End of dendrite tip of nerve cell.
b. Dendrite → cell body → axon to its ends.

26. a.

b.

Section C
27. i. When dilute hydrochloric acid is added to sodium carbonate - Evolution of carbon dioxide gas takes place.
ii. Lemon juice is added gradually to potassium permanganate solution - Change its colour from purple to colourless.
iii. When dilute sulphuric acid is added to the barium chloride solution - Formation of white precipitate of barium sulphate.

16 P.T.O
iv. Quick lime is treated with water - change in temperature.
v. Wax is burned in the form of a candle - Change in state from solid to liquid and gas.
28. The compound prepared from gypsum on heating it till 100 C , is known as Plaster of Paris. Its chemical formula is

H O . Hence, its chemical name is calcium sulphate hemihydrate. The chemical equation for its preparation is as
1
C aSO .4 2
2

follows:

100 C
1 1
C aSO4 .2H2 O −−−→ C aSO4 . H2 O + 1 H2 O
2 2
Gypsum W ater

Plaster of Paris is used in hospitals mainly as plaster for supporting fractured bones in the right position. In dentistry, it is used for
making casts.
29. (i) As the metal 'M' forms oxide M O it is trivalent. Out of the metals listed, only F e and Al are trivalent.
2 3

(ii) M O is not affected by water, so 'M' can be out of F e or Al.


2 3

(iii) F e and Al both corrode easily.


(iv) Out of Al and F e, only F e can form divalent chloride, so the element 'M' is F e.
30. i. The next homologue of C5H11COOH (Pentanoic acid) is C6H13COOH (Hexanoic acid). Two adjacent homologues in a series
differ by 1 carbon atom and 2 hydrogen atoms i.e. by a CH2 group.
ii. Carbon can exist in three solid forms (Diamond, Graphite, Fullerenes) called allotropes. C60 and C70 are members of
Fullerenes (Allotropes of carbon). Buckminsterfullerene contains a cluster of 60 carbon atoms joined together to form
spherical molecules.
31. Algae are the main source of productivity in sea. they prepare there own food with the help of photosynthesis. they are food for
many small aquatic animals living in the sea. hence the algae is food for small fishes and small fishes are food for big fishes. it is
like a food chain.
Algae produce O2 as a result of photosynthesis. This oxygen is utilized by fishes in the sea for carrying out respiration. If there
were no algae, no oxygen would have been produced. Thus fishes might have died. Hence we can say if there were no algae there
would be no fish in the sea.
32. A number of interactions between the environment and the animal are a result of combined action of both neurons and endocrine
system for which specialized tissues are used to provide control and co – ordinations activities. Nervous system that includes
brain, spinal cord along with other peripheral nervous transfer the information for procesisng. The endocrine system releases
hormones in response to stimulus to control and co-ordinate the functions.
33. a. Statement of Joule's law
Joule’s law of heating states that, when a current 'i' passes through a conductor of resistance ‘R’ for time ‘t’ then the heat
developed in the conductor is equal to the product of the square of the current, the resistance and time.
Mathematical explanation
Let H be the heat produced when a current 'i' passes through a conductor of resistance ‘r’ for time ‘t’ then
i. H ∝ i 2

ii. H ∝ R
iii. H ∝ t
∴ H ∝ i2Rt
⇒ H = i2Rt
b. Here V = 6V and R = 5Ω
The current flowing through the resistor I = V

R
=
6V


= 1.2 A
H = I2RT = (1.2)2 × 5 × 10 = 72J

17 P.T.O
Section D

34. Nervous System Hormonal System

Made of neuron Made of Hormone

Message transmitted in the form of chemicals called


Messages transmitted in the form of electrical impulses
hormones.

Messages transmitted along nerve fibres. Messages transmitted through blood stream.

Messages travel very quickly. Messages travel very slowly.

Effect of message usually lasts for a very short while Effect of message usually lasts longer

Sense organs are receptors as nerve cells located in them receive Glands are effectors which secrete hormones when the brain
information. commands.
OR
The working together of various organs of human being in a systematic, controlled and efficient way to produce a proper response
to various stimuli is known as coordination.
In human beings, the control and coordination is brought about by both nervous system and endocrine system. Nervous system
consists of receptors that receive the stimulus from surrounding environment and send the message received by them to the spinal
cord and brain in form of electrical impulses through the sensory nerves.
The motor nerves then transmit the response to the effector. The effectors are mainly the muscles and glands of our body. Thus,
endocrine glands secreting hormones are directly or indirectly controlled by the nervous system. For example, when an emergency
stimulus is detected by the nervous system, the stimulus is detected by the nervous system , the stimulus is received and analysed
by central nervous system that send message to effectors to provide proper response. At the same time, the sympathetic nervous
system activates adrenal gland to release adrenaline that prepares body by increasing heart rate, blood pressure,respiration and
dilates pupil etc.
Hence, control and coordination in humans (or animals) depend on two things for transmitting information, i.e. chemical signals
of hormones and nerve impulses.
If they depended only on nerve impulses through nerve cells, only a limited range of tissues would be stimulated. Since, they get
additional chemical signals as well, a large number of tissues are stimulated. This is why animals can show a wide range of
response to stimulus.
35. Images formed by concave mirror.
i. Object at Infinity. Two cases arise :
a. When mirror is in parallel plane to the object. In such a case, rays from infinity come parallel to principal axis. After
reflection they pass through principal focus F (Rule 1). Image is extremely small, it is real, inverted and at principal focus.

Object at infinity, real extremely diminished image is formed at principal focus.

18 P.T.O
b. When mirror is inclined so that the rays strike the mirror obliquely. The ray AB passing through F after reflection goes
parallel to principal axis towards BA' (Rule 2). Another ray DE through C striking the mirror at E is reflected back. The
two form an image at A' Image is real, inverted, extremely diminished and at F.

Object at infinity, image at F. It is real, inverted, very much diminished.


c. Object beyond C. A ray AD from A parallel to principal axis after reflection passes through F (Rule 1), Another ray from
A through C, ray AG is reflected back along the same path (Rule 3), forming real, diminished, inverted image of AB is
formed at A'B', between F and C.

Object beyond C, a real, inverted diminished image between F and C, inverted at C and is of same size as that of object.
d. Object at C i.e. at 2f. A ray AD from A parallel to principal axis after reflection from mirror passes through F (Rule 1).
Another ray AD' from A through F, goes parallel to principal axis i.e. towards D'A' (rule 2) forming real, inverted image of
AB at C i.e. at 2f. The image is of the same size as the object.

Object at C, Image is also at C. It is real.


e. Object between F and C (f and 2f) A ray AD from object going parallel to principal axis is reflected towards F (Rule 1).
Another ray AE through C is reflected back (Rule 3) forming image of A at A'. Similarly image of B is formed at B'.
Image is real, inverted, enlarged and beyond C (2f) i.e. as shown in fig.

f. Object at F. A ray AD parallel to principal axis passes through F. Another ray AE strikes the mirror normally at E is
reflected back as it passes through C (Rule 3). They form image of object AB at infinity. The image is very much enlarged
and is real and inverted.

Object at E. Real, inverted, extremely enlarged image is formed at infinity.


g. Object between F and P. A ray AD from A goes parallel to principal axis after reflection passes through F (rule 1). Another
ray AE striking the mirror normally through C is reflected back (rule 3). They form virtual image of the object behind the
mirror. The image is erect and enlarged.

P.T.O
19
Object between F and P. An erect, enlarged, virtual image is formed behind the mirror.
Images Formed by a Concave mirror

Position of Object Position of Image Size of the Image Nature of Image

At infinity At focus F Highly diminished Real and inverted

At C At C Same size Real and inverted

At F At infinity Highly Enlarged Real and inverted

Beyond C Between F and C Diminished Real and inverted

Between F and C Beyond C Enlarged Real and inverted

Between P and F Behind the mirror Enlarged Virtual and erect


OR
Object at Infinity: Rays from infinity come parallel to principal axis. Ray AB is reflected towards BH and ray EG is reflected
towards GJ. When produced back they meet at F, the principal focus (Rule 1).
An extremely small, erect, virtual image is formed at F.

Object at infinity: A virtual, erect, extremely diminished image is formed at F.


Object placed anywhere except infinity. A ray AD parallel to principal axis after reflection appears to pass through F. Another ray
AE normal to mirror and appearing to pass through C is reflected back along the same path. They appear to meet at A' forming
image of A and A'. Similarly, image of B is formed at B'. This virtual, erect image of AB is formed at A'B' between P and F
behind the mirror. Image is erect and diminished in size.

Object any where, A virtual, erect, diminished image is formed behind the mirror between F and P
36. i. Equivalent resistance between M and N
(R3 ×R4 )
=[ (R3 +R4 )
]

ii. Joule's law of heating states that when a current i passes through a conductor of resistance r for time t then the heat developed
in the conductor is equal to the product of the square of the current, the resistance, and time. This can be expressed as:
H = I2Rt
iii. We need a fuse of 5A for an electric iron which consumes 1 kW power at 220 V. It is because:
Given P = 1000 W, V = 220 V
As we know, P = V × t
P
Or I = V
1000
Or I = 220V
= 4.5 A
Hence,4.54 ampere current flows in the circuit, the fuse should be of 5A.

20 P.T.O
iv. As you know, in a series circuit the current is constant throughout the electric circuit. Thus it is obviously impracticable to
connect an electric bulb and an electric heater in series because they need currents of widely different values to operate
properly.
OR

Bulbs in series combination

Bulbs in parallel combination


Let the resistance of each bulb is R
In series:
Current in each bulb = V

3R

In Parallel:
Current in each bulb = 3V

R
I V
Current will get equally divided in three bulb = 3
=
R

i. In series, potential difference get divided whereas in parallel potential difference across each bulb remains full. Hence in series
bulbs glow with less brightness than bulbs in parallel connection.
ii. Now, if one bulb gets fused in a series connection, all the other bulbs will stop glowing, whereas in a parallel connection, other
bulbs will continue to glow. This is because in series combination even a single faulty component break the circuit whereas
this does not happen in parallel circuit.

Section E
37. Read the text carefully and answer the questions:
Redox reactions are those reactions in which oxidation and reduction occur simultaneously. A redox reaction is made up of two
half reactions. In the first half reaction, oxidation takes place and in second half reaction, reduction occurs. Oxidation is a process
in which a substance loses electrons and in reduction, a substance gains electrons. The substance which gains electrons is reduced
and acts as an oxidising agent. On the other hand, a substance which loses electrons is oxidised and acts as a reducing agent.
(i) The oxidation and reduction occurring together are called a redox reaction. CuO + H2 ⟶ Cu + H2O. Example: In
this reaction, copper oxide is being reduced to copper whereas hydrogen is being oxidised to water.
(ii) Oxidising agents give oxygen to or removes hydrogen from other substances. The reducing agent undergoes loss of
electrons. A reducing agent (also called a reductant or reducer) is an element or compound that loses (or "donates") an

21 P.T.O
electron to another chemical species in a redox chemical reaction.
(iii)ZnO is being reduced
ZnO is reduced to Zn and CO is oxidised to CO2
(iv)H2O2 is reduced to water by removal of oxygen.
OR
The oxidation and reduction occurring together are called a redox reaction. CuO + H2 ⟶ Cu + H2O. Example: In
this reaction, copper oxide is being reduced to copper whereas hydrogen is being oxidised to water.
38. Read the text carefully and answer the questions:
Non-metals are either solids or gases. Non-metal can exist in different forms such as carbon. Each form is called allotrope. Alkali
metal is so soft that it can be cut with a knife. They have low density and low melting point. Some metal can melt if they are kept
in the palm.
(i) Metals are malleable and ductile. Metals are good conductors of heat and electricity. Metals are lustrous (shiny) and can
be polished. Metals are solids at room temperature (except mercury, which is liquid).
(ii) Metals Non-metals

These are solids at room temperature except mercury These exist in all three states

These are very hard except sodium These are soft except diamond

These are malleable and ductile These are brittle and can break down into pieces

These are shiny These are non-lustrous except iodine

Electropositive in nature Electronegative in nature

Have high densities Have low densities


(iii)Exceptions of Metals on the basis of their Physical Properties are given below:
Malleability: Zinc, Arsenic, Mercury, Antimony are non-malleable and non-ductile metals.
Conductivity: Aluminium and Titanium are poor conductors of Heat and Electricity.
Lustre: Sodium is a non-lustrous metal.
Hardness: Cesium, Mercury (Liquid Metal), Gallium are soft Metals.
(iv)Sodium is stored in kerosene because sodium reacts vigourously with oxygen and moisture due to its high reactivity
so keeping it in kerosene will prevent sodium from coming in contact with oxygen and moisture.
OR

Washing soda is used for removing the permanent hardness of the water.

39. Read the text carefully and answer the questions:


The compounds which have the same molecular formula but differ from each other in physical or chemical properties are called
isomers and the phenomenon is called isomerism. When the isomerism is due to difference in the arrangement of atoms within the
molecule, without any reference to space, the phenomenon is called structural isomerism. In other words, structural isomers are
compounds that have the same molecular formula but different structural formulas, i.e., they are different in the order in which
different atoms are linked. In these compounds, carbon atoms can be linked together in the form of straight chains, branched
chains or even rings.
(i) Butane and iso-butane both have molecular formula as C4H10
CH 3 CH 2 CH 2 CH 3 , CH 3 CHCH 3
|

CH
3

22 P.T.O
(ii) Yes, Butane and 2-methylpropane
CH3CH2CH2CH3 and CH CHCH have different structural formula but same molecular formula. These two
3 3
|

CH3

molecules, butane and 2-methylpropane, both have the molecular formula C4H10 , but they are different molecules since
the atoms are arranged differently.
(iii)2-methylpentane has the longest chain
iso-Pentane:
1 2 3 4

H3 C − C−CH2 − CH3 (4 carbon atoms)


|

CH3

2-Methylpentane:

1 2 3 4 5

CH3 − CH − CH2 − CH2 − CH3 (5 carbon atoms)


|

CH3

Hence, 2-methylpentane has the longest carbon chain.


(iv)Pentane (C5H12) has three structural isomers:
CH3 − CH2 − CH2 − CH2 − CH3
n−Pentane

CH3 − CH− CH2 − CH3


|

CH3

iso−Pentane

CH3

CH3 − C−CH3
|

CH3

neo−Pentane

OR

When sulphur trioxide (acidic oxide) is dissolved in water, an exothermic reaction takes place with the
formation of sulphuric acid.

SO3+ H2O H2SO4

23 P.T.O
Series SCI/10/2 Set ~ 2
Roll No. Q.P Code 10/2/2
Candidates must write the Q.P Code
on the title page of the answer-book.

 Please check that this question paper contains 10 printed pages.


 Q.P. Code given on the right hand side of the question paper should be written on the title
page of the answer-book by the candidate.
 Please check that this question paper contains 39 questions.
 Please write down the serial number of the question in the answer-book before
attempting it.
 15 Minute time has been allotted to read this question paper. The question paper will be
distributed at 10:15 a.m From 10.15a.m to 10.30 a.m, the students will read the question
paper only and will not write any answer on the answer –book during this period

SCIENCE
Time Allowed : 3 hours Maximum Marks : 80

General Instructions:

i. This question paper consists of 39 questions in 5 sections.


ii. All questions are compulsory. However, an internal choice is provided in some questions.
A student is expected to attempt only one of these questions.
iii. Section A consists of 20 objective type questions carrying 1 mark each.
iv. Section B consists of 6 Very Short questions carrying 02 marks each. Answers to these
questions should be in the range of 30 to 50 words.
v. Section C consists of 7 Short Answer type questions carrying 03 marks each. Answers to
these questions should be in the range of 50 to 80 words.
vi. Section D consists of 3 Long Answer type questions carrying 05 marks each. Answer to
these questions should be in the range of 80 to 120 words.
vii. Section E consists of 3 source - based/case - based units of assessment of 04 marks
each with sub - parts

1 P.T.O
Section A 20 Marks
1 The chips packet is flushed with a gas X to prevent rancidity. Identify X .

a) Oxygen

b) Carbon dioxide

c) Nitrogen

d) Water vapour
2 We will observe white ppt in which of the following reaction?

a) Barium chloride is mixed with hydrochloric acid

b) Barium chloride is mixed with sodium chloride solution

c) Barium chloride is mixed with sodium sulphate solution

d) Barium carbonate is mixed with sodium sulphate solution

3 Bleaching powder is treated with CO2 :

a) It absorbs the gas.

b) CaO is formed.

c) CaCl2 is formed.

d) CaCO3 and Cl 2 are formed.

4 Which of the following are present in a dilute aqueous solution of hydrochloric


acid?

a) Unionised HCI

b) Cl+ OH

c) H3 O + + Cl −

d) H3 O + + OH

2 P.T.O
5

Which of the following is correct conclusion?

a) Al is more reactive than Cu and Fe but less reactive than Zn

b) Al is more reactive than Cu but less reactive than Zn and Fe

c) Al is more reactive than Zn and Cu but less reactive than Fe

d) Al is more reactive than Zn, Cu, Fe

6 Clove oil and onion are examples of


a) basic indicator

b) acid - base indicator

c) olfactory indicator

d) artificial indicator

7 Which of the following are combination reactions?

1. 2KClO$_{ 3$ 2KCl+ 3O 2

2. MgO + H2 O → Mg(OH) 2

3. 4Al+ 3O2 → 2A1 2 O 3

4. Zn + FeSO4 → ZnSO 4 + Fe

a) (ii) and (iii)

b) (i) and (iii)

c) (iii) and (iv)

d) (ii) and (iv)


8 Oxygen liberated during photosynthesis comes from

a) Carbon dioxide

b) Water

c) Glucose

d) Chlorophyll

3 P.T.O
9 A doctor advised a person to take an injection of insulin because

a) his heart was beating slowly

b) he was suffering from goitre

c) his sugar level in blood was high

d) his blood pressure was low

10 The movement of sunflower in accordance with the path of sun is due to


a) hydrotropism

b) chemotropism

c) phototropism

d) geotropism

11 Following diagram show the structure of a neuron .

Through which part of neuron information travels as an electrical impulse?

a) Nucleus

b) Dendrite

c) Axon

d) Cyton

12 Under the high power objective of a microscope, an epidermal peel of a leaf


shows

a) stomata surrounded by several guard cells each

b) stomata surrounded by several epidermal cells

c) stomata surrounding by a pair of guard cells each

d) stomata surrounding many guard cells

4 P.T.O
13 An electric bulb is rated 220 V and 100 W. When it is operated on 110 V, the
power consumed will be -

a) 100 W

b) 50 W

c) 25 W

d) 75 W

14 What is the current in a circuit if the charge passing each point is 20 C in 40 s?


a) 0.5 C

b) 0.5 A

c) 0.2A

d) 0.2 C

15 The refractive indices of four substances P, Q, R, and S are 1.50, 1.36, 1.77, and
1.31 respectively. The speed of light is the maximum in the substance:

a) R

b) S

c) P

d) Q

16 A lens of focal length ’ f ’ is cut into two equal parts without affecting its
curvature. The two pieces will have equal focal length of :
𝑓
a)
2

b) f
𝑓
c)
3

d) 2f
17 Assertion (A): Silver articles become black after sometime when exposed to
sunlight.

Reason (R): It is because silver reacts with carbonates present in the air.

a) Both A and R are true and R is the correct explanation of A.

b) Both A and R are true but R is not the correct explanation of A.

c) A is true but R is false.

d) A is false but R is true.

5 P.T.O
18 Assertion (A): Respiration is a biochemical process opposite to photosynthesis.
Reason (R): Energy is released during respiration.

a) Both A and R are true and R is the correct explanation of A.

b) Both A and R are true but R is not the correct explanation of A.


c) A is true but R is false.

d) A is false but R is true.

19 Assertion (A): Voltmeter is always connected in parallel across the circuit while
measuring the potential difference.

Reason (R): As the voltage in parallel circuits are measured to be the same.

a) Both A and R are true and R is the correct explanation of A.

b) Both A and R are true but R is not the correct explanation of A.

c) A is true but R is false.

d) A is false but R is true.

20 Assertion (A): Convex mirrors are used for rear view on vehicles.
Reason (R): The size of the image formed by a convex mirror will be same.

a) Both A and R are true and R is the correct explanation of A.

b) Both A and R are true but R is not the correct explanation of A.

c) A is true but R is false.

d) A is false but R is true.

Section B 12 Marks

21 A student added few pieces of aluminium metal to two test tubes A and B
containing aqueous solution of iron sulphate and copper sulphate. In the second
part of her experiment, she added iron metal to another test tube C and D
containing aqueous solution of aluminium sulphate and copper sulphate.

In which test tube or test tubes will she observe colour change? On the basis of
this experiment state which one is the most reactive metal and why?

22 Plants have low energy needs as compared to animals. Explain.


23 What are the differences between the transport of materials in xylem and
phloem?

6 P.T.O
OR

What is lymph?

24 If the image formed by a lens for all positions of an object placed in front of it is
always erect and diminished, what is the nature of this lens? Draw a ray diagram
to justify your answer. If the numerical value of the power of this lens is 10 D,
what is its focal length in the Cartesian system?

25 What is the cause of refraction of light when it passes from one medium to
another?

OR

To construct a ray diagram we use two rays which are so chosen that it is easy to
know their directions after reflection from the mirror. List two such rays and
state the path of these rays after reflection in case of concave mirrors. Use these
two rays and draw ray diagram to locate the image of an object placed between
pole and focus of a concave mirror.

26 State Ohm’s law? How can it be verified experimentally? Does it hold good under
all conditions? Comment.
21 Marks
Section C

27 Identify the type of reaction in the following


1. ZnCO3 + 2HCl(aq) → ZnCl 2 (aq) + H 2 CO 3 (aq)

2. 2NaBr(aq) + Cl2 (g) → 2NaCl(aq) + Br 2 (aq)


ℎ𝑒𝑎𝑡
3. 2CuO(s) → 2Cu(s) + O 2 (g)

28 Leaves of healthy potted plant were coated with vaseline. Will this plant remain
healthy for long? Give reasons for your answer.

OR
"If there were no algae there would be no fish in the sea." Comment.

29 The diagram given below is the experiment set - up of show that carbon dioxide
is given out during respiration. In this set - up what does test tube marked (A)
contain? What are its role in the experiment

30 Nervous and hormonal system together performs the functions of control and
coordination in human beings. Justify the statement.
31 An object is kept at a distance of 18 cm, 20 cm and 30 cm, from a lens of
power+5D. (i) In which case or cases would you get a magnified image? (ii)
Which of the magnified image can we get on a screen? (b) List two widely used
applications of a convex lens.

7 P.T.O
32 1. Write the relationship between electrical resistance and electrical
resistivity for a metallic conductor of cylindrical shape. Hence derive the
SI unit of electrical resistivity.

2. Find the resistivity of the material of a metallic conductor of length 2 m

and area of cross - section 1.4× 10 −6 m 2 . The resistance of the


conductor is 0.04 ohm.

33 The given figure shows a battery, a switch and two bulbs. Complete the diagram
to show the electric connections of the bulbs to the battery. How have you joined
the bulbs? Give a reason.

Section D 15 Marks

34 Give the name of raw materials that are required for the manufacture of washing
soda. Write the reaction involved in the process.

OR

1. Write three physical properties each of acids and bases,

2. How will you show with an example that metal oxides are basic in nature?
Give chemical equation also.

35 Draw a labelled diagram of human brain and mention the functions of the
following: Medulla oblongata, cerebellum and forebrain.

OR

Draw the structure of a neuron and describeits function.

36 Define the principal focus of concave mirror.


OR

1. What is a lens? List two main categories of lenses. In which category is a


double concave lens placed?

2. A convex lens of focal length 15 cm forms a real image at a distance of 20


cm from its optical centre. Find the position of the object. Is the image
formed by the lens magnified or diminished?

8 P.T.O
Section E 12 Marks
37 Read the text carefully and answer the questions: The teacher while
conducting practicals in the laboratory divided the students into three groups
and gave them various solutions to find out their pH and classify them into acidic,
basic and neutral solutions.

Group A - Lemon juice, vinegar, colourless aerated drink

Group B - Tomato juice, coffee, ginger juice

Group C - Sodium hydroxide, sodium chloride, lime water

1. For the solutions provided, which group is/are likely to have pH value (i)
less than 7, and (ii) greater than 7?

2. List two ways of determining pH of a solution.

3. How can the electrical conductivity of a solution help to differentiate


between acidic, basic and neutral solutions?
OR

Explain, why the sour substances such as lemon juice are effective in cleaning
the tarnished copper vessels.

38 Read the text carefully and answer the questions: In animals, control and
coordination are provided by nervous and muscular tissues. Touching a hot
object is an urgent and dangerous situation for us. We need to detect it and
respond to it. How do we detect that we are touching a hot object? All
information from our environment is detected by the specialised tips of some
nerve cells. These receptors are usually located in our sense organs, such as the
inner ear, the nose, the tongue, and so on. So gustatory receptors will detect taste
while olfactory receptors will detect the smell. This information, acquired at the
end of the dendritic tip of a nerve cell, see figure, sets off a chemical reaction that
creates an electrical impulse. This impulse travels from the dendrite to the cell
body, and then along the axon to its end.

9
P.T.O
1. Name the largest cell present in the body.

2. What is an axon ?

3. Name one gustatory receptor and one olfactory receptor present in a


human beings.

OR

Name the followingparts of a neuron:

a. Where information is acquired.

b. Through which information travels as an electrical impulse.

Question No. 39 to 42 are based on the given text. Read the text carefully
and answer the questions:

The dissolving of an acid or a base in water is a highly exothermic reaction. Care


must be taken while mixing concentrated nitric acid or sulphuric acid with water.
The acid must always be added slowly to water with constant stirring. If water is
added to a concentrated acid, the heat generated may cause the mixture to splash
out and cause burns. The glass container may also break due to excessive local
heating. Look out for the warning sign on the can of concentrated sulphuric acid
and on the bottle of sodium hydroxide pellets.

39 What is the exothermic reaction?


40 Write an example of an exothermic reaction.
41 How will you obtainsulphuric acid from an acidic oxide?
42 While diluting an acid, why is it recommended that the acid should be added to
water and not water to the acid ?

10 P.T.O
Maximum Marks: 80 Time Allowed: 3 hours
General Instructions:

i. This question paper consists of 39 questions in 5 sections.


ii. All questions are compulsory. However, an internal choice is provided in some
questions. A student is expected to attempt only one of these questions.
iii. Section A consists of 20 objective-type questions carrying 1 mark each.
iv. Section B consists of 6 Very Short questions carrying 02 marks each. Answers to
these questions should be in the range of 30 to 50 words.
v. Section C consists of 7 Short Answer type questions carrying 03 marks each.
Answers to these questions should be in the range of 50 to 80 words.
vi. Section D consists of 3 Long Answer type questions carrying 05 marks each.
Answers to these questions should be in the range of 80 to 120 words.
vii. Section E consists of 3 source-based/case-based units of assessment of 04 marks
each with sub-parts.

MCQ - Multiple
Chapter Assertion Picture Choice Subjective
Name - & Reason Based Question Question Total

Chemical - 1 (1) 2 (1) 2 (1) 1 (3) 6 (8)


Reactions and
Equations

Acids Bases - - - 3 (1) 1 (2) 4 (5)


and Salts

Life Processes - 1 (1) - 2 (1) 1 (2) 5 (8)

1 (3)

11
P.T.O
Control and - - 1 (1) 2 (1) 1 (3) 4 (6)
Coordination

Light - 1 (1) - 2 (1) 1 (2) 5 (8)


Reflection and
Refraction 1 (3)

Electricity - 1 (1) - 2 (1) 1 (2) 6 (11)

2 (3)

(4 Grouped 1 - - - - 1 (4)
Questions) (4)
Control and
Coordination

(3 Grouped 1 - - - - 1 (4)
Questions) (4)
Acids Bases
and Salts

(2 Internal - - - - 1 (2) 1 (2)


Choice)
Life Processes
Life Processes

(2 Internal - - - - 1 (2) 1 (2)


Choice)
Light
Reflection and
Refraction
Light
Reflection and
Refraction

(2 Internal - - - - 1 (3) 1 (3)


Choice)

12 P.T.O
Life Processes
Life Processes

(2 Internal - - - - 1 (5) 1 (5)


Choice)
Acids Bases
and Salts
Acids Bases
and Salts

(2 Internal - - - - 1 (5) 1 (5)


Choice)
Control and
Coordination
Control and
Coordination

(2 Internal - - - - 1 (5) 1 (5)


Choice)
Light
Reflection and
Refraction
Light
Reflection and
Refraction

(Paragraph - - - - 4 (1) 4 (4)


Questions)
Acids Bases
and Salts

Total 2 4 (4) 3 (3) 13 (13) 20 (52) 42


(8) (80)

13 P.T.O
Section A
1.
(c) Nitrogen
Explanation: Rancidity can be prevented by packaging fat and oil-containing foods in nitrogen gas as it is unreactive. Hence,
the chips packets are flushed with nitrogen gas to prevent aerial oxidation of fats and oil containing chips.
2.
(c) Barium chloride is mixed with sodium sulphate solution
Explanation: On mixing a solution of barium chloride with sodium sulphate, a white precipitate of barium sulphate is
immediately formed. These reactions are ionic in nature.
BaCl2 + Na2SO4 →
​ ​BaSO4 + 2NaCl

3.
(d) CaCO3 and Cl2 are formed.
Explanation: CaCO3 and Cl2 are produced when Bleaching powder is treated with CO2.
C a(OC l)2 + C O2 → C aC O3 + C l2 ↑

4.
(c) H3O+ + Cl-

Explanation: Any acid produces hydrogen ion (H+) which is present as hydronium ion (H3O) because of combination with a
water molecule.
5.
(d) Al is more reactive than Zn, Cu, Fe
Explanation: 2Al + 3ZnSO4 → Al2(SO4)3 + 3 Zn
2Al + 3CuSO4 → Al2(SO4)3 + 3 Cu
2Al + 3FeSO4 → Al2(SO4)3 + 3Fe
Al is more reactive than Zn, Cu and Fe because it displaces them from their salt solution.
6.
(c) olfactory indicator
Explanation: An Olfactory indicator is defined as a substance whose smell varies when it is mixed with an acidic or basic
solution. Such substances can be used in the laboratory to test whether a solution is a base or an acid, and this process is called
olfactory titration. All onion, clove oil, and vanilla extract are examples.
i. Onion: Paste or juice of onion loses its smell when added with base. It does not change its smell with acid.
ii. Clove oil: In bases, its characteristic smell cannot be detected.

7. (a) (ii) and (iii)


Explanation: A combination reaction is one in which or more reactants combine to form a single product. In reaction (ii),
MgO and H2O combine to form Mg(OH)2. In reaction (iii). Al and O2 combine to form Al2O3. A combination reaction is also
known as a synthesis reaction.
Reaction (i) is decomposition reaction and reaction (iv) is displacement reaction.
8.
(b) Water
Explanation: During photosynthesis, water molecule splits to produce Oxygen and Hydrogen ions. Oxygen is expelled out of
plants and Hydrogen is used to reduce Carbon-di-oxide to produce carbohydrates.
9.
(c) his sugar level in blood was high

14 P.T.O
Explanation: Patient suffering from diabetes will have high blood glucose due to non-functioning or lack of insulin
hormone. Such patients are administered with insulin injection to regulate blood glucose.
10.
(c) phototropism
Explanation: Phototropism is the response of the plants towards light. Thus, sunflower moves in accordance with the path of
sun.
11.
(c) Axon
Explanation: At the end of the axon, the electrical impulse sets off the release of some chemicals, which cross the synapse and
start a similar electrical impulse in a dendrite of the next neuron. In this way nerve impulses travel in the body.
12.
(c) stomata surrounding by a pair of guard cells each
Explanation: Onion peel possesses stomata, guard cells, and epidermal cells. The nuclei are present in both epidermal and
guard cells. Stomata is a pore surrounded by guard cells, so it does not possess any nucleus.
13.
(c) 25 W
2 2

Explanation: Resistance of the electric bulb is R = V

P
or R = 220

100

2 2
V 110 × (100) 100
When it is operated at 110 V, the power consumed will be P =
R
or P =
2
or P =
4
or P = 25W
( 220 )

14.
(b) 0.5 A
Explanation: Given,
Charge moved, Q = 20C
Time taken, t = 40s
To find,
Current, I = ?
We know that,
Q
I= t

Substituting the values in the above equation we get


I= 20

40
= 0.5A
Therefore, 0.5A is required for moving 20C charge in 40s.
15.
(b) S
Explanation: S
16.
(b) f
Explanation: Since the lens is cut into two equal parts without affecting its curvature, it means the radius of curvature R is
same for both parts and hence the focal length( F = ) will remain same.
R

17.
(c) A is true but R is false.
Explanation: Silver reacts with sulphur present in the air and forms a layer of silver sulphide, therefore, silver articles get
tarnished. Thus assertion is true, but reason is false.
18. (a) Both A and R are true and R is the correct explanation of A.
Explanation: Respiration is defined as the process of the biochemical oxidation of nutrients at the cellular level. It occurs in
the presence of specific enzymes at optimum temperature in the cells to release energy for various metabolic activities.
Both A and R are true and R is the correct explanation of A.
19. (a) Both A and R are true and R is the correct explanation of A.
Explanation: Voltage measured in the parallel circuits are always equal. As all the parallel circuits start from one point and end
at another point and always the potential difference between the two points will always be the same. So, this is the reason why
voltmeter is always connected in parallel across the circuit. Thus, both assertion and reason are true and reason is the correct
explanation of assertion.

15 P.T.O
20.
(b) Both A and R are true but R is not the correct explanation of A.
Explanation: Convex mirrors are not used for rear view on vehicles. We use only concave mirrors for rear view. They will
always give erect and diminished image of the object. They provide a wider field for clear view. For this reason, the convex
mirrors are fitted on both sides of the vehicles. Therefore, the given assertion is true, but reason is not the correct explanation of
assertion.
Section B
21. Only in test tubes A and B she will observe colour change from green to colourless in A. because aluminium is more reactive then
iron it will displace iron. She will observe colour change from blue to colourless in test tube B. Aluminium is the most reactive
metal because it displace all the other three metals.
22. Plants are non-motile whereas animals are motile which require high amount of energy expenditure. Moreover, most of the
movements of substances in plants happen through passive transport through dead cells which does not need energy. Due to this,
the energy requirement in plants is low as compared to animals.
23. Differences between transport in xylem and phloem.
Transport in xylem Transport in phloem
1) Sucrose, amino acids and other substances are
1) Water and minerals are transported through xylem.
transported through phloem.
OR
Lymph is the plasma and formed elements that have leaked out of the capillaries into extracellular environment. It carries food and
wastes to the body cells outside the circulatory system. Lymph is formed from the fluid which leaks from blood capillaries and
goes to the intercellular spaces in the tissues. This fluid is collected through lymph vessels and finally returns to the blood
capillaries. Lymph also plays an important role in the immune system.
24. It is a concave lens.

The power of a lens is given by the relation


1
∵ P= f ( in metre )

P = 10 D
∴ f = = 0.1 m
1 1
=
P 10

or -10 cm, as lens is concave lens.


25. Light travels with different speeds in different media, and this is the cause of refraction when it passes from one medium to
another. Larger the difference in speed of light between the two media, greater will be the refraction and vice-versa.
OR
A ray of light incident parallel to the principal axis after reflection passed through the principal focus.

A ray of light which passes through the centre of curvature after reflection retraces its path back.

16 P.T.O
Image of an object placed between pole and focus:

26. According to Ohm’s law, the current (I) flowing through a conductor is directly proportional to the potential difference (V) across
its ends, provided its physical conditions remain the same.
Ohm's law does not hold good under all conditions as it is not a fundamental law of nature like Newton's laws.
It is obeyed by metallic conductors only when physical conditions like temperature etc. are kept unchanged. It is not obeyed by a
lamp filament, Junction diode, thermistor, etc.
Section C
27. a. Double decomposition reaction [An exchange of ions took place]
b. Displacement reaction [A more reactive non-metal displaces a less reactive non-metal from its salt solution.]
c. Decomposition reaction/Reduction reaction [A compound decomposes to form two or more products./CuO is reduced to Cu.]
28. If leaves of a healthy potted plant are coated with vaseline, it will clog the stomatal pores on leaves. Blockage of stomatal will
stop transpiration and exchange of gases from leaves. Transpiration plays an important role in ascent of sap in plants. Hence, lack
of transpiration will stop ascent of sap. Moreover, stoppage of exchange of gases would also stop respiration and photosynthesis in
leaves. This will result in death of leaves and finally the plant would die due to lack of food.
OR
Algae are the main source of productivity in sea. they prepare there own food with the help of photosynthesis. they are food for
many small aquatic animals living in the sea. hence the algae is food for small fishes and small fishes are food for big fishes. it is
like a food chain.
Algae produce O2 as a result of photosynthesis. This oxygen is utilized by fishes in the sea for carrying out respiration. If there
were no algae, no oxygen would have been produced. Thus fishes might have died. Hence we can say if there were no algae there
would be no fish in the sea.
29. The test-tube (A) contains potassium hydroxide. It absorbs the CO2 released during the respiration of seeds which creates a partial
vacuum in conical flask causing the rise in the water level of the bent delivery tube. Rise in level of water shows that CO2 is
released during respiration.
30. A number of interactions between the environment and the animal are a result of combined action of both neurons and endocrine
system for which specialized tissues are used to provide control and co – ordinations activities. Nervous system that includes
brain, spinal cord along with other peripheral nervous transfer the information for procesisng. The endocrine system releases
hormones in response to stimulus to control and co-ordinate the functions.
31. (i) Focal length = 1/power = 1/5D = 1/5m = 20cm
It is convex lens of focal length 20cm.
So, the Magnified image will be formed in all cases, 20cm is focus, 18 cm is on focal length, 22 cm and 30 cm is between focus
and focus and center of curvature. In all cases, magnified image is formed.
(ii) In case of 22 cm and 30 cm image formed is real and hence can be obtained on screen.
32. i. ρ = R A

l
2

SI unit of ρ = ohm × m

= ohm × metre/Ω m
A
ii. ρ = R l
−6 2
0⋅04Ω×1⋅4×10 m
=
2 m
−8
= 2 ⋅ 8 × 10 Ωm

17
P.T.O
33. The two bulbs are connected in parallel and the complete circuit diagram is drawn below:

The reason for connecting the two bulbs in parallel is that (i) both the bulbs glow at the same voltage, and (ii) if one bulb stops
glowing, the other bulb remains unaffected.
Section D
34. Raw materials : Sodium chloride (NaCl), Ammonia (NH3), Lime stone (CaCO3)
Reaction:
NaCl + NH3 + H2O + CO2 → NH4Cl + NaHCO3

Ammonium Sodium hydrogen


Chloride carbonate
Heat
2NaHCO3 −−−→
Na2CO3 + H2O + CO2
Sodium hydrogen sodium
carbonate carbonate
Na2CO3 + 10H2O → Na2CO3.10H2O
Sodium carbonate Washing soda
Heat

CaCO3 −−−→ CaO + CO2


Limestone Quick lime
CaO + H2O → Ca(OH)2
Ca(OH)2 + 2 NH4Cl → NaCl2 + 2NH3 + 2H2O
OR
i. Properties of acids
(a) Acids are sour in taste.
(b) Acids turn blue litmus paper to red.
(c) Acids conduct electricity in its solution.
Properties of bases
(a) Bases have a bitter taste.
(b) Bases feel soapy to touch.
(c) Bases turn red litmus paper to blue.
ii. The reaction between copper oxide and dil. hydrochloric acid:
C uO(s) + 2HC l(aq)⟶ C uC l2 (aq) + H2 O(l)
Copper oxide Copper chloride (Blue - Green) W ater

When copper oxide reacts with dil. hydrochloric acid it forms copper chloride and water. This shows that copper oxide (metal
oxide) is a basic oxide.
35. Functions of forebrain: It helps in speech, facial activities, mental activities, hearing, smell, sight, taste, smell, touch, etc.
Functions of the cerebellum: It maintains the posture, equilibrium and muscle tone.
Functions of Medulla Oblongata: it helps in maintaining the rate of heartbeat, breathing movements, sneezing, coughing,

18 P.T.O
vomiting, regulates blood pressure, etc.

OR
Neuron or nerve cell is a structural and functional unit of the nervous system that is specialised to receive, conduct and transmit
nerve impulses. A neuron (nerve cell) has three components:
i. Cell body (cyton)
ii. Dendrites
iii. Axon

Functions: The information acquired at the end of the dendritic tip of a neuron sets off a chemical reaction which creates an
electrical impulse. This impulse travels from die dendrite to the cyton along the axon of its end. At the end of axon, the electrical
impulse sets off the release of some chemicals, which cross the synapse and start a similar electrical impulse in a dendrite of the
next neuron. In this way nerve impulses travel in the body, from one neuron to another till it reaches the brain or the target organ.
Thus, a nervous tissue is made up of an organised network of nerve cells or neurons which are specialised in conducting
information via electrical impulse from one part of the body to another.
36. Light rays that are parallel to the principal axis of a concave mirror converge at a specific point on its principal axis after
reflecting from the mirror. This point is known as the principal focus of the concave mirror.
Principal focus (p.f.) is a point on principal axis of a concave mirror where the rays parallel to principal axis meet after reflection
from the mirror.

Additional reading. For convex mirror, principal focus is a point on principal axis of a convex mirror where rays parallel to
principal axis appear to diverge from after reflection from the mirror.

OR
a. A transparent material bounded by two surfaces of which one or both surfaces are spherical / curved.
a. Converging lens
b. Diverging lens
Double concave lens is a diverging lens.
b. To find the position of the object, we can use the lens formula:
f = 15 cm, v = 20 cm, u = ?

19 P.T.O
1 1 1
= −
f v u

1 1 1
⇒ = −
u v f

= 1

20 cm

1

15 cm
4−3
= 60

u = -60 cm.
image is diminished.
Section E
37. Read the text carefully and answer the questions:
The teacher while conducting practicals in the laboratory divided the students into three groups and gave them various solutions to
find out their pH and classify them into acidic, basic and neutral solutions.
Group A - Lemon juice, vinegar, colourless aerated drink
Group B - Tomato juice, coffee, ginger juice
Group C - Sodium hydroxide, sodium chloride, lime water
(i) i. Groups A and B - less than 7
ii. Group C - greater than 7
(ii) pH paper and universal indicator.

(iii) Acids and bases are electrolytes, meaning they dissociate into ions in water and conduct electricity.
Neutral solutions typically have lower conductivity compared to acidic or basic solutions.

OR
i. Copper vessel is tarnished due to formation of basic copper oxide.
ii. Lemon juice being acidic react with copper oxide and the salt formed is washed away.

38. Read the text carefully and answer the questions:


In animals, control and coordination are provided by nervous and muscular tissues. Touching a hot object is an urgent and
dangerous situation for us. We need to detect it and respond to it. How do we detect that we are touching a hot object? All
information from our environment is detected by the specialised tips of some nerve cells. These receptors are usually located in
our sense organs, such as the inner ear, the nose, the tongue, and so on. So gustatory receptors will detect taste while olfactory
receptors will detect the smell. This information, acquired at the end of the dendritic tip of a nerve cell, see figure, sets off a
chemical reaction that creates an electrical impulse. This impulse travels from the dendrite to the cell body, and then along the
axon to its end.

(i) Nerve cell is the largest cell present in the body.


(ii) Axon is a large, single, unbranched nerve fibre arising from the cyton. It carries impulses from cyton located in CNS to
the effectors.
(iii)Gustatory receptor: Taste buds on the tongue. The receptors for gustation are located in the oral cavity, which brings
food and fluids from outside the body into the gastrointestinal tract.
Olfactory receptor: Receptor in the nose. These receptors are common to arthropods, terrestrial vertebrates, fish, and
other animals.
OR
a. Dendrites.
b. Axon.

20 P.T.O
39. An exothermic reaction is a chemical reaction that releases energy through light or heat.
40. Mixing of acid with water is a highly exothermic reaction.
41. When sulphur trioxide (acidic oxide) is dissolved in water, an exothermic reaction takes place with the formation of sulphuric
acid.
SO3 + H2O → H2SO4
42. Since the process of dissolving an acid in water is exothermic, it is always recommended that acid should be added to water. If it
is done the other way, then it is possible that because of the large amount of heat generated, the mixture splashes out and causes
burns.

21 P.T.O
Series
SeriesMAT/12/1
Series SCI/10/1
FH1EG/3
SCI/10/1 Set ~ 3
Series
Roll No. Q.P Code 10/3/3
Candidates must write the Q.P Code
on the title page of the answer-book.

 Please check that this question paper contains 11 printed pages.


 Q.P. Code given on the right hand side of the question paper should be written on the title
page of the answer-book by the candidate.
 Please check that this question paper contains 39 questions.
 Please write down the serial number of the question in the answer-book before
attempting it.
 15 Minute time has been allotted to read this question paper. The question paper will be
distributed at 10:15 a.m From 10.15a.m to 10.30 a.m, the students will read the question
paper only and will not write any answer on the answer –book during this period

SCIENCE
Time Allowed : 3 hours Maximum Marks : 80

General Instructions:

i. This question paper consists of 39 questions in 5 sections.


ii. All questions are compulsory. However, an internal choice is provided in some questions.
A student is expected to attempt only one of these questions.
iii. Section A consists of 20 objective type questions carrying 1 mark each.
iv. Section B consists of 6 Very Short questions carrying 02 marks each. Answers to these
questions should be in the range of 30 to 50 words.
v. Section C consists of 7 Short Answer type questions carrying 03 marks each. Answers to
these questions should be in the range of 50 to 80 words.
vi. Section D consists of 3 Long Answer type questions carrying 05 marks each. Answer to
these questions should be in the range of 80 to 120 words.
vii. Section E consists of 3 source - based/case - based units of assessment of 04 marks
each with sub - parts

[1]
Section A 20
Which characteristic is observed by the reaction shown in the given Marks
1
image?

a) Formation of a precipitate

b) Change in temperature

c) Evolution of a gas
d) Both change in temperature and evolution of gas

2 On the basis of evolution or absorption of heat, chemical reactions can be


divided in how many types ?

a) One

b)) wo

c) hree

d) F ur

3 Which of the following substance will not give carbon dioxide on


treatment with dilute acid?

a) arble

b)) mestone

c) L me

d) Baking soda
4 Sodium hydroxide is termed an alkali while Ferric hydroxide is not
because:

1. Sodium hydroxide is a strong base, while Ferric hydroxide is a


weak base.

2. Sodium hydroxide is a base which is soluble in water while Ferric


hydroxide is also a base but it is not soluble in water.

3. Sodium hydroxide is a strong base while Ferric hydroxide is a


strong acid.

4. Sodium hydroxide and Ferric hydroxide both are strong base but
the solubility of Sodium hydroxide in water is comparatively higher
than that of Ferric hydroxide.

a) Option (c)

b) Option (b)

c) Option (a)

d) Option (d)

5 Some crystals of copper sulphate were dissolved in water. The colour of


the solution obtained would be:
a) Green

b)) ed

c) Brown

d) lue

6 The crystals of ferrous sulphate on heating gives:

a) F 2 O 3 + H 2 SO 4 +H 2 O

b)) O+ H2 O + SO 2

c) F O + SO3 + H 2 SO 4 + H 2 O

d) F 2 O 3 + SO 2 + SO 3 + H 2 O

7 What happens when a solution of an acid is mixed with a solution of a base


in a test tube

1. The temperature of the solution increases

2. The temperature of the solution decreases

3. The temperature of the solution remains the same

4. Salt formation takes place


a) (ii) and (iii)

b) (i) and (iii)

c) (i) only

d) (i) and (iv)

8 Name the passage that leads bile from the liver into the gall bladder.

a) Colon

b) Cystic duct

c) Caecum

d) Rectum

9 The substance that triggers the fall of mature leaves and fruits from plants
is due to

a) cytokinin
b) gibberellin

c) abscisic acid

d) auxin

10 When a person is suffering from severe cold, he or she cannot -


a) Differentiate the taste of an apple form that of an ice – cream

b) Differentiate the smell of a perfume from that of an agarbatti

c) Differentiate a hot object form a cold object

d) Differentiate red light from green light

11 In which of the following group/ groups of animals, the heart does not
pump oxygenated blood to different parts of the body?

a) Pisces only

b)) mphibians only

c) Pisces and amphibians

d) Amphibians and reptiles only

12 The chemical used to test the starch:


a) Methyl alcohol

b) Safranin

c) Glycerine

d) Iodine
13 You are given water, mustard oil, glycerine and kerosene. In which of
these media a ray of light incident obliquely at same angle would bend the
most?

a) Glycerine

b) Kerosene

c) Water

d) Mustard oil

14 An ammeter has 20 divisions between mark 0 and mark 2 on its scale. The
least count of the ammeter is
a) 0.01 A

b) 0.02 A

c) 0.2 A

d) 0.1 A

15 Which one of the following materials cannot be used to make a lens?


a) Clay

b) Plastic

c) Water

d) Glass

16 A resistance of 25𝛺 is connected to a 12 V battery. The heat energy in


joules generated per minute:

a) 300 J

b) 347.6 J

c) 345.6 J

d) 355.7J

17 Assertion (A): Iron articles are painted so as to prevent them from


rusting.

Reason (R): When the surface of iron is coated with paint, its surface does
not come in contact with oxygen and moisture therefore rusting does not
take place.

a) Both A and R are true and R is the correct explanation of A.

b) Both A and R are true but R is not the correct explanation of A.

c) A is true but R is false.

d) A is false but R is true.


18 Assertion (A): Blood takes up oxygen from the alveolar air and release CO
2 during exchange.

Reason (R): The concentration of O 2 is more in alveolar air.

a) Both A and R are true and R is the correct explanation of A.

b) Both A and R are true but R is not the correct explanation of A.


c) A is true but R is false.

d) A is false but R is true.

19 Assertion (A): When the length of a wire is doubled, thenits resistance


also gets doubled.

Reason(R): The resistance of a wire is directly proportional to its length.

a) Both A and R are true and R is the correct explanation of A.

b)) oth A and R are true but R is not the correct explanation of A.

c) A is true but R is false.

d) A is false but R is true.

20 Assertion (A): Copper vessels get covered with green coating in rainy
season.

Reason (R): It is because of the formation of copper carbonate.

a) Both A and R are true and R is the correct explanation of A.

b) Both A and R are true but R is not the correct explanation of A.

c) A is true but R is false.

d) A is false but R is true.


Section B 12 Marks
21 (i) Name the products formed when sodium hydrogen carbonate is
heated.

(ii) Write the chemical equation for the reaction involved in (i)

22 Which part of the plant shows positive geotropism and why?


23 The BP of a person is reported to be 120 and 80. What do the upper and
lower numbers indicate?
OR

Dark reaction of photosynthesis does not need light. Do plants undergo


dark reaction at night explain.

24 A convex lens of focal length 15 cm forms an image 10 cm from the lens.


How far is the object placed from the lens? Draw the ray diagram.
25 Write the chemical composition of tooth enamel. Under what conditions of
pH it starts corroding? Explain the reason of tooth decay and suggest one
method to prevent it.

OR

What is salt hydrolysis ?

26 You have two circuits:


(i) a 6V battery is series with 1 𝛺 and 2 𝛺 resistors

(ii) a 4V battery in parallel with 12 𝛺 and 2 𝛺 . resistors

Compare the power used in 2𝛺 resistor in each case.

Section C 21 Marks

27 While eating food, you spill some curry on your white shirt. You
immediately scrub it with soap. What happens to its yellow colour on
scrubbing with soap? What happens to this stain when the shirt is washed
with plenty of water?
28 Why do fire flies glow at night?
OR

Silver chloride kept in a china dish turns grey in sunlight.

1. Write the colour of silver chloride when it was kept in the china
dish.

2. Name the type of chemical reaction taking place and write the
chemical equation for the reaction.

3. State one use of the reaction. Name one more chemical which can
be used for the same purpose.
29 Describe an experiment to demonstrate thatCO2 is essential for
photosynthesis.
30 Differentiate between exocrine gland and endocrine gland.
31 How are the power and focal length of a lens related? You are provided
with two lenses of focal length 20 cm and 40 cm respectively. Which lens
will you use to obtain more convergent light?
32 1. Two electric lamps rated 100 W,220 V and 60 W,220 V are
connected in parallel to electric mains supply. Calculate thecurrent
drawn from the mains if the supply voltage is 220 V?

2. A lamp consumes 50 W and is lighted 2 h daily in month of April.


How many units of electric energy is consumed ?
33 In a circuit find
1. total resistance

2. current shown by ammeter.

15 Marks
Section D

34 What happens when baking powder is added to the dough? Write the
reaction also.

OR

A compound ’A’ is used in fire extinguishers,as an antacid and its small


amount is also used in making bakery items. Identify the compound and
also explain the reason for above mentioned uses of the compound ’A’.

35 Study the diagram given below and answer the following:


1. Label the parts numbered 1 - 10. What does the figure represent?

2. Name all the parts in a sequence through which air from outside
reaches the last part of lungs.

3. What is the structural and functional unit of lungs? What important


role is played by them?

OR

What are hormones? State their role in the working of the human body.Or
Define ’Hormone’. What are the general functions of ’hormones’?

36 A 2.0 cm tall object is placed perpendicular to the principal axis of a


convex lens of focal length 10 cm. The distance of the object from the lens
is 15 cm. Find the nature, position, and size of the image. Also, find its
magnification.

OR

1. A concave mirror of focal length 10 cm can produce a magnified


real as well as virtual image of an object placed in front of it. Draw
ray diagrams to justify this statement.

2. An object is placed perpendicular to the principal axis of a convex


mirror of focal length 10 cm. The distance of the object from the
pole of the mirror is 10 cm. Find the position of the image formed.

Section E 12 Marks

37 Read the text carefully and answer the questions:


Copper sulphate crystal contains water of crystallisation when the crystal
is heated the water is removed and salt turns white. The crystalcan be
moistened again with water. The water of crystallisation is the fixed
number of water molecules present in 1 formula unit of copper sulphate.
On heating gypsum at 373K, it loses water molecules and became calcium
sulphate hemihydrate.
1. If the crystal is moistened with water, then which colour of the
crystal reappears?

2. What is the commercial name of calcium sulphate hemihydrate?

3. How does heating gypsum change its properties?

OR

4. How many water molecules arepresent in one formula unit of


copper sulphate?

38 Read the text carefully and answer the questions:


Fill a conical flask with water. Cover the neck of the flask with a wire
mesh. keep two or three freshly germinated bean seeds on the wire mesh.
Take a cardboard box which is open from one side. Keep the flask a wire
mesh. Kin the box in such a manner that the open side of the box faces
light coming from a window as shown in the given figure. After two or
three days, you will notice that the shoots bend towards light and roots
away from light. Now turn the flask so that the shoots are away from light
and the roots towards the light. Leave it undisturbed in this condition for a
few days. Plants show tropism in response to other stimuli as well. The
roots of a plant always grow downwards while the shoots usually grow
upwards and away from the earth. This upward and downward growth of
shoots and roots, respectively, in response to the pull of earth or gravity, is
obviously, geotropism.

1. What has represented by the given activities?

2. Do old parts of the shoot and root change direction? Is there any
difference in the direction of the new growth?

3. What can we conclude from this activity?

OR

4. What isgeotropism?
39 Read the text carefully and answer the questions:
In 1827, a German physicist Georg Simon Ohm (1787 - 1854) found out
the relationship between the current I, flowing in metallic wire and the
potential difference across its terminals. He stated that the electric current
flowing through a metallic wire is directly proportional to the potential
difference V, across its ends provided its temperature remains the same.

The resistance of a circuit is defined as the ratio between the voltage


applied to the current flowing through it. Rearranging the above relation,

$R = {{V} \over I}$

Electric charge flows easily through some materials than others. The
electrical resistance measures how much the flow of this electric charge is
restricted within the circuit.

1. What is the unit of electrical resistance?

2. Define Ohm’s law.

3. From graph which resistancehave high resistance?

OR

4. What does the slope of V - I graph at any point represent?


Series
Series
Series SCI/10/4
MAT/12/1
FH1EG/3
SCI/10/1 Set ~ 4
Series
Roll No. Q.P Code 10/4/4
Candidates must write the Q.P Code
on the title page of the answer-book.

 Please check that this question paper contains 10 printed pages.


 Q.P. Code given on the right hand side of the question paper should be written on the title
page of the answer-book by the candidate.
 Please check that this question paper contains 39 questions.
 Please write down the serial number of the question in the answer-book before
attempting it.
 15 Minute time has been allotted to read this question paper. The question paper will be
distributed at 10:15 a.m From 10.15a.m to 10.30 a.m, the students will read the question
paper only and will not write any answer on the answer –book during this period

SCIENCE
Time Allowed : 3 hours Maximum Marks : 80

General Instructions:

i. This question paper consists of 39 questions in 5 sections.


ii. All questions are compulsory. However, an internal choice is provided in some questions.
A student is expected to attempt only one of these questions.
iii. Section A consists of 20 objective type questions carrying 1 mark each.
iv. Section B consists of 6 Very Short questions carrying 02 marks each. Answers to these
questions should be in the range of 30 to 50 words.
v. Section C consists of 7 Short Answer type questions carrying 03 marks each. Answers to
these questions should be in the range of 50 to 80 words.
vi. Section D consists of 3 Long Answer type questions carrying 05 marks each. Answer to
these questions should be in the range of 80 to 120 words.
vii. Section E consists of 3 source - based/case - based units of assessment of 04 marks
each with sub - parts

[1]
Section A 20 Marks

1 Zinc pieces were placed in each of the four test tubes containing different salt
solutions as shown below

A colour change would be observed in solutions:

a) IIand IV

b) III and IV
c) IIand III

d) Iand IV

2 The green coating on copper appears on exposure to air. It is:

a) Copper carbonate

b) Copper sulphate

c) Copper nitrate

d) Copper sulphide

3 Which one of the following types of medicines is used for treating indigestion?

a) Antacid

b) Antiseptic

c) Antibiotics

d) Analgesic

4 What happens when dilute HCl is slowly added to copper oxide in a beaker?

a) solution turns blue - green


b) solutionturns green

c) solution turns blue

d) solution turns brown

5 What happens when silver chloride is placed in sunlight?


a) Silver chloride turns black

b) Silver chloride turns grey

c) Silver chloride turns blue

d) Silver chloride show no change

6 Washing soda is a

a) acidic salt

b) neutralized salt

c) amphoteric salt
d) basic salt

7 Which of the following is nastic movement?

a) Shoots moving towards light

b) Roots moving downwards

c) The growth of pollen tube towards a chemical

d) Opening of flower (evening primrose)


8 A well - stained leaf peel mount when observing under the high power of a
microscope shows nuclei in

a) only epidermal cells

b) guard cells and epidermal cells

c) guard cells, epidermal cells and stoma.

d) only guard cells


9 What is the function of glucagon?

a) Decrease the blood sugar

b) All of these

c) Regulate the metabolism of fat

d) Increase the blood sugar


10 The pancreatic juice does not contain one of the following enzymes.
a) Lipase

b) Amylase

c) Ptyalin

d) Trypsin

11 Acetic acid, when dissolved in water, dissociates into ions reversibly because it is:

a) Weak base

b) Strong base

c) A weak acid

d) Strong acid
12 In the slide of an epidermal peel, the parts which appear pink coloured after
staining with safranin are

a) nuclei only

b) all parts in the peel

c) cell membrane and cytoplasm

d) stomata only

13 If the current I through a resistor is increased by 100%(at constant temperature),


the increase in power dissipated will be

a) 400%

b) 300%

c) 100%

d) 200%

14 Which of the following does not represent electrical power?

a) I2 R
𝑉2
b)
𝑅

c) IR2

d) VI
15 The refractive indexes of four substances P, Q, R, and S are 1.77, 1.50, 2.42, and
1.31 respectively. When light travelling in air is incident on these substances at
equal angles, the angle of refraction will be the maximum in:

a) substance S

b) substance P

c) substance Q

d) substance R

16 In a voltmeter, there are 20 divisions between the 0 mark and 0.5 V mark. The
least count of the voltmeter is

a) 0.050 V

b) 0.250 V
c) 0.025 V

d) 0.020 V

17 Assertion (A): MnO 2 + 4HCl → MnCl 2 + Cl 2 + 2H 2 O, is a redox reaction.


Reason (R): MnO 2 oxidises HClto Cl 2 and gets reduced to MnCl 2 .

a) Both A and R are true and R is the correct explanation of A.

b) Both A and R are true but R is not the correct explanation of A.

c) A is true but R is false.

d) A is false but R is true.

18 Assertion (A): Reflex actions are automatic and rapid responses to stimuli.
Reason (R): These actions are controlled by the brain.

a) Both A and R are true and R is the correct explanation of A.

b) Both A and R are true but R is not the correct explanation of A.

c) A is true but R is false.

d) A is false but R is true.


19 Assertion (A): The refractive index of Kerosene is 1.44 which is optically denser
than water.

Reason (R): The mass density of kerosene is lesser than water.

a) Both A and R are true and R is the correct explanation of A.

b) Both A and R are true but R is not the correct explanation of A.

c) A is true but R is false.

d) A is false but R is true.

20 Assertion (A): Longer wires have greater resistance and the smaller wires have
lesser resistance.

Reason (R): Resistance is inversely proportional to the length of the wire.

a) Both A and R are true and R is the correct explanation of A.

b) Both A and R are true but R is not the correct explanation of A.

c) A is true but R is false.


d) A is false but R is true.
Section B 12 Marks
21 Identify the substance oxidized and substance reduced in the following reaction.
Write the ionic equation for the substance oxidized and reduced.

H 2 (g) + CuO(s)

→ Cu(s) + H 2 O ( l )

22 1. A white power is an active ingredient of antacids and is used in


preparation of baking powder. Name the compound and explain that how
it is manufactured. Give chemical equation.

2. Write a chemical equation to show the effect of heat on this compound.

23 The plants were wilting in a garden and the gardener watered the plants. The
plants became fresh again. Which part of the plant is responsible for conducting
water in it and how does the process of conduction of water take place in plants?
OR

How does aerobic respiration differ from anaerobic respiration?


24 1. Water has refractive index 1.33 and alcohol has refractive index 1.36.
Which of the two medium is optically denser? Give reason for your
answer.

2. Draw a ray diagram to show the path of a ray of light passing obliquely
from water to alcohol.

3. State the relationship between angle of incidence and angle of refraction


in the above case.

25 If the current I through a resistor is increased by 100% (assume that


temperature remains unchanged), then find the increase in power dissipated.

OR

A 4𝛺 resistance wire is doubled on it. Calculate the new resistance of the wire.

26 Rohit wants to have an erect image of an object using a converging mirror of focal
length 40 cm.

1. Specify the range of distance where the object can be placed in front of the
mirror. Give reason for your answer.

2. Will the image be bigger or smaller than the object?


3. Draw a ray diagram to show the image formation in this case.

Section C 21 Marks

27 A milkman adds a very small amount of baking soda to fresh milk.


1. Why does he shift the pH of the fresh milk from 6 to slightly alkaline?

2. Why does this milk take a long time to set as curd?

3. What do you expect to observe when milk comes to boil?

28 When solutions of silver nitrate and sodium chloride are mixed, white precipitate
forms. The ionic equation for the reaction is

Ag + (ag) + Cl − → AgCl(s)

1.
a. What is the name of the white precipitate?

b. Is it a soluble or insoluble compound?

2. Is the precipitation of silver chloride a redox reaction?


OR

During the reaction of some metals with dilute hydrochloric acid, the following
observations were made by a student:

1. Silver does not show any change.

2. Some bubbles of a gas are seen when lead is reacted with the acid.

3. The reaction of sodium is found to be highly explosive.

4. The temperature of the reaction mixture rises when aluminium is added


to the acid.

Explain these observations giving appropriate reason.

29 Describe the structure and functioning of nephrons.


30 Nervous and hormonal system together performs the functions of control and
coordination in human beings. Justify the statement.
31 How are the power and focal length of a lens related? You are provided with two
lenses of focal length 20 cm and 40 cm respectively. Which lens will you use to
obtain more convergent light?
32 Calculate:
1. the effective resistance of the circuit and the current in the circuit
2. Potential difference across 10𝛺 resistor of a circuit shown in the figure.

33 What kind of graph is obtained by plotting values of V and I? Why?


Section D 15 Marks

34 How are acids classified based upon (a) source (b) strength (c) Concentration.
OR

Discuss the role of pH in (a) digestive system (b) causes of tooth decay.

35 1. Name the disorder which a person is likely to suffer fromdue to the


following:

a. Over - secretion of growth hormone

b. Deficiency of oestrogen in females


c. Less secretion of thyroxine

Also name the gland that secretes each of the hormones mentioned
above.

2. How is the timing and amount of hormone released regulated? Explain


with the help of an example.

OR

What are reflex actions? Give examples? Explain reflex arc by an example with
labelled diagram?

36 A convex lens of focal length 20 cm can produce a magnified virtual as well as


real image. Is this a correct statement? If yes, where shall the object be placed in
each case for obtaining these images?

OR

An object 5 cm in length is held 25 cm away from a converging lens of focal length


10 cm. Draw the ray diagram and find the position, size and the nature of the
image formed.

Section E 12 Marks

37 Read the text carefully and answer the questions:


Salt of a strong acid and strong base is neutral with a pH value of 7. NaCl common
salt is formed by a combination of hydrochloride and sodium hydroxide solution.
This is the salt that is used in food. Some salt is called rock salt, bed of rock salt
was formed when seas of bygone ages dried up.The common salt thus obtained is
an important raw material for various materials of daily use, such as sodium
hydroxide, baking soda, washing soda, and bleachingpowder.

1. If given acids are phosphoric acid, carbonic acid, hydrochloric acidand


sulphuric acid, then which aciddoes not form an acidic salt?

2. What is the formula of baking soda?

3. The passage mentions two sources of salt: table salt and rock salt. How are
these sources formed, and how does the text differentiate between them?

OR
Name the substance which on treatment with chlorine to obtain bleaching
powder.
38 Read the text carefully and answer the questions:
We have seen that the different parts of our body have specific functions. Our
mouth waters when we see the food we like without our meaning to. Our heart’s
beat without our thinking about it. In fact, we cannot control these actions easily
by thinking about them even if we wanted to. So, in between the simple reflex
actions like change in the size of the pupil, and the thought out actions such as
moving a chair, there is another set of muscle movements over which we do not
have any thinking control. Many of these involuntary actions are controlled by
the mid - brain and hind - brain. All these involuntary actions including blood
pressure, salivation and vomiting are controlled by the medulla in the hind -
brain. Think about activities like walking in a straight line, riding a bicycle,
picking up a pencil. These are possible due to a part of the hind - brain called the
cerebellum. It is responsible for the precision of voluntary actions and
maintaining the posture and balance of the body. Imagine what would happen if
each of these events failed to take place if we were not thinking about it.

1. Identify the part of the nervous system which controls the reflex action.

2. Does reflex actioninvolve all parts of the voluntary nervous system?

3. Identify the part of the autonomic nervous system which controls


involuntary actions.

OR

Beating of heart muscles , which type of action is this? Out of voluntary and
involuntary action which is slower?

39 Read the text carefully and answer the questions:


Consider the following electrical circuit diagram in which nine identical resistors
of 3𝛺 each are connected as shown. If the reading of the ammeter A 1 is 1 ampere,
answer the following questions:

1. What is the relationship between the readings of A1 and A 3 ? Give reasons


for your answer.

2. What is the relationship between the readings of A2 and A 3 ?

3. Determine the reading of the voltmeter V1.

OR

Find the total resistance of the circuit.


Series
Series
Series SCI/10/5
SeriesMAT/12/1
SCI/10/1
FH1EG/3
SCI/10/1 Set ~ 5
Series
Roll No. Q.P Code 10/5/5
Candidates must write the Q.P Code
on the title page of the answer-book.

 Please check that this question paper contains 11 printed pages.


 Q.P. Code given on the right hand side of the question paper should be written on the title
page of the answer-book by the candidate.
 Please check that this question paper contains 39 questions.
 Please write down the serial number of the question in the answer-book before
attempting it.
 15 Minute time has been allotted to read this question paper. The question paper will be
distributed at 10:15 a.m From 10.15a.m to 10.30 a.m, the students will read the question
paper only and will not write any answer on the answer –book during this period

SCIENCE
Time Allowed : 3 hours Maximum Marks : 80

General Instructions:

i. This question paper consists of 39 questions in 5 sections.


ii. All questions are compulsory. However, an internal choice is provided in some questions.
A student is expected to attempt only one of these questions.
iii. Section A consists of 20 objective type questions carrying 1 mark each.
iv. Section B consists of 6 Very Short questions carrying 02 marks each. Answers to these
questions should be in the range of 30 to 50 words.
v. Section C consists of 7 Short Answer type questions carrying 03 marks each. Answers to
these questions should be in the range of 50 to 80 words.
vi. Section D consists of 3 Long Answer type questions carrying 05 marks each. Answer to
these questions should be in the range of 80 to 120 words.
vii. Section E consists of 3 source - based/case - based units of assessment of 04 marks
each with sub - parts

[1]
Section A 20 Marks

1 Which type of chemical reaction is shown in the given image?

a) Redox reaction

b) Combination reaction

c) Decomposition reaction

d) Displacement reaction

2 The taste and smell of food changes when kept for a long time in open. It is called
a) Rancidity

b) Corrosion

c) Oxidation

d) Reduction

3 Which of the following substances is an antacid?

a) NaCl

b) H2 SO 4

c) Mg(OH)2

d) HCl

4 pH (power of Hydrogen) value of black coffee is:

a) 5

b) 3

c) 7

d) 8
5. During deficiency of oxygen in tissues of human beings, pyruvic acid is converted into lactic acid in the

a) Golgi body

b) Mitochondria

c) Chloroplast

d) Cytoplasm

6 A student added zinc granules to copper sulphate solution taken in a test tube.
Out of the following, the correct observations made by the student will be

1. Zinc granules have no regular shape.

2. Zinc granules have silvery grey colour.

3. The colour of zinc granules changed to brownish - black.

a) C only

b) B only
c) A only

d) All of these

7 Seeds taken in the conical flask during respiration experiment must be

a) Dry gram seeds

b) Wet gram seeds

c) Boiled gram seeds

d) Germinating gram seeds

8 In the experiment to demonstrate that CO2 is given out during respiration, what
would you observe in the delivery tube dipped in water?

a) Water level rises in the delivery tube.

b) Water turns milky and rises in the delivery tube.

c) Water turns milky but does not rise in the delivery tube.

d) Water level in the delivery tube remains unchanged.


9 The nature of the image is not affected by the position of the object in

a) convex mirror

b) convexlenses

c) concave lenses

d) concave mirror

10 Which of the following is nastic movement?


a) Shoots moving towards light

b) Roots moving downwards

c) The growth of pollen tube towards a chemical

d) Opening of flower (evening primrose)

11 Which of the following hormone present in the newborn child that becomes
smaller with advancing years?

a) Thymus gland

b) Testis
c) Pineal gland

d) Ovary

12 A black strip of paper was clipped onto a destarched leaf in a potted plant to
cover a part of the leaf. The plant was then exposed to sunlight for four hours, the
paper strip was removed and the leaf was tested for starch. When iodine solution
was added:

a) The entire leaf turned blue - black.

b) The uncovered part of the leaf became blue - black.

c) The colour of the iodine solution remain unchanged.

d) The covered part of the leaf became blue - black.

13 Which of the following statements about the current in given circuit is correct?

a) R has maximum current.

b) Current at Q is greater than the current at R.

c) P has maximum current.

d) Current at R is greater than the current at P.


14 A current of 200mA flows through a 4 k𝛺 resistor. What is the potential
difference across the resistor?

a) 4000 volt

b) 900 volt

c) 800 volt

d) 3000 volt

15 Which of the following limits the number of trophic levels in a food chain?
a) Food supply

b) Decrease in energy at each trophic level

c) Polluted air
d) Water

16 Which of the following phenomena is not the result of total internal reflection?
a) Looming

b) Sparkles of the diamond

c) Mirage

d) Twinkling of stars

17 Assertion (A): Following is a balanced chemical equation for the action of steam
on iron:

3Fe + 4H 2 O → Fe 3 O 4 + 4H 2

Reason (R): The law of conservation of mass holds good for a chemical equation.

a) Both A and R are true and R is the correct explanation of A.

b) Both A and R are true but R is not the correct explanation of A.

c) A is true but R is false.

d) A is false but R is true.

18 Assertion (A) : Damage to the medulla oblongata causes death.


Reason (R) : Medulla oblongata controls involuntary functions of the body.

a) Both A and R are true and R is the correct explanation of A.

b) Both A and R are true but R is not the correct explanation of A.

c) A is true but R is false.

d) A is false but R is true.


19 Assertion (A): The refractive index of Kerosene is 1.44 which is optically denser
than water.

Reason (R): The mass density of kerosene is lesser than water.

a) Both A and R are true and R is the correct explanation of A.

b) Both A and R are true but R is not the correct explanation of A.

c) A is true but R is false.

d) A is false but R is true.

20 Assertion (A): Longer wires have greater resistance and the smaller wires have
lesser resistance.

Reason (R): Resistance is inversely proportional to the length of the wire.

a) Both A and R are true and R is the correct explanation of A.

b) Both A and R are true but R is not the correct explanation of A.

c) A is true but R is false.

d) A is false but R is true.

Section B 12 Marks
21 What change will you observe if you test soap with litmus paper (red or blue)?
22 What is the difference between reflex action and walking?
23 Only deoxygenated blood is pumped through a fish’s heart. Is it true? Justify your
answer.
OR

What are the different modes of excretion seen in plants?

24 A convex mirror used on a bus has a focal length of 200 cm. If a scooter is located
at 100 cm. from this mirror find the position, nature and magnification of the
image formed in the mirror.
25 Why is parallel arrangement used in domestic wiring?
OR

Explain the term lateral inversion.

26 A concave mirror of focal length 10 cm is placed at a distance of 35 cm from a


wall. How far from the wall an object be placed so that its image formed by
mirror falls on the wall?
Section C 21 Marks

27 A milkman adds a very small amount of baking soda to fresh milk.


1. Why does he shift the pH of the fresh milk from 6 to slightly alkaline?

2. Why does this milk take a long time to set as curd?

3. What do you expect to observe when milk comes to boil?

28 Write the balanced chemical equation for the following reaction:


1. Phosphorus burns in presence of chlorine to form phosphorus penta
chloride.
2. Burning of natural gas.

3. The process of respiration

OR

Does Tartaric acid helps in making cake or bread fluffy. Justify.

29 Observe the following table carefully and match the components of part I with
part II of the table. Write them in complete sentences.

30 Nervous and hormonal system together performs the functions of control and
coordination in human beings. Justify the statement.
31 Sudha finds out that the sharp image of window pane of her science laboratory is
formed at a distance of 15 cm from the lens. She now tries to focus the building
visible of her outside the window instead of the window pane without disturbing
the lens. In which direction will she move the screen to obtain a sharp image of
the building? What is the approximate focal length of this lens?
32 1. When a 12 V battery is connected across an unknown resistor, there is a
current of 2.5 mA in the circuit. Find the value of the resistance of the
resistor?

2. 320J of heat is produced in 10 s in a 2𝛺 resistor. Find the amount of


current flowing through the resistor.

33 If in the figure R1 = 10 𝛺 , R 2 = 40 𝛺 , R 3 = 30 𝛺 , R 4 = 20 𝛺 , R 5 = 60 𝛺 , and a 12


V battery is connected to the arrangement.Calculate
1. the total resistance in the circuit ,and

2. the total current flowing in the circuit.


Section D 15 Marks
34 What are hydrated salts and water of crystallization ?
OR

A cloth strip dipped in onion juice is used for testing a liquid X. The liquid X
changes its odour. Which type of an indicator is onion juice? The liquid X turns
blue litmus red. List the observations the liquid X will show on reacting with the
following:

1. Zinc granules

2. Solid sodium carbonate

Write the chemical equations for the reactions involved.

35 What are the different types of nervous system present in our body and what are
its components? How are the components of nervous system protected?

OR

What constitutes the central and peripheral nervous systems? How are the
components of central nervous system protected? Which signals will get
disrupted in case of a spinal cord injury?

36 An object is placed at a distance of 30 cm from a concave lens of focal length 30


cm.

1. Use lens formula to determine the distance of the image from the lens.

2. List four characteristics of the image (nature position, size,


erect/inverted) in this case.
3. Draw a labelled diagram to justify your answer of part (ii)

OR

1. An object is placed in front of a convex lens of focal length f. If the distance


of the object from the lens is 2f, draw a ray diagram to show the formation
of the image. State two characteristic of image formed.
2. A student has focussed the image of a candle flame on a white screen
using a convex lens. The situation is as given below:

Length of the flame = 1.5 cm

Focal length of the lens = 12 cm

Distance of the flame from the lens = 18 cm

If the flame is perpendicular to the principal axis of the lens, calculate the
values of the following:

a. Distance of the image from the lens


b. Length of the image formed

Section E 12 Marks

37 Read the text carefully and answer the questions:


The teacher while conducting practicals in the laboratory divided the students
into three groups and gave them various solutions to find out their pH and
classify them into acidic, basic and neutral solutions.

Group A - Lemon juice, vinegar, colourless aerated drink

Group B - Tomato juice, coffee, ginger juice

Group C - Sodium hydroxide, sodium chloride, lime water

1. For the solutions provided, which group is/are likely to have pH value (i)
less than 7, and (ii) greater than 7?

2. List two ways of determining pH of a solution.

3. How does Rohan's body react to the incoming ball without him
consciously deciding to move? Explain the role of the nervous system in
this reflex action.
OR

Explain, why the sour substances such as lemon juice are effective in cleaning
the tarnished copper vessels.
38 Read the text carefully and answer the questions
Environmental triggers such as light, or gravity will change the directions that
plant parts grow in. These directional, or tropic, movements can be either
towards the stimulus or away from it. So, in two different kinds of phototropic
movement, shoots respond by bending towards light while roots respond by
bending away from it. How does this help the plant? Plants show tropism in
response to other stimuli as well. The roots of a plant always grow downwards
while the shoots usually grow upwards and away from the earth. This upward
and downward growth of shoots and roots, respectively, in response to the pull
of earth or gravity, is, obviously, geotropism. If ‘hydro’ means water and ‘chemo’
refers to chemicals, what would ‘hydrotropism’ and ‘chemotropism’ mean? Can
we think of examples of these kinds of directional growth movements? One
example of chemotropism is the growth of pollen tubes towards ovules, about
which we will learn more when we examine the reproductive processes of living
organisms.

1. Where does negative phototropism occur in plants?

2. Phototropism in shoots is attributed due to which plant hormone?

3. Tendrils exhibit/ twining of tendrils show which type of tropic


movement?

OR

If the stem grows towards sunlight and the root grows just opposite to it, then
what type of movement of the stem is it?

39 Read the text carefully and answer the questions:


Study the following electric circuit in which the resistors are arranged in three
arms A, B and C :
1. Find the equivalent resistance of arm C.

2. Calculate the equivalent resistance of the parallel combination of the arms


B and C.

3. Determine the current that flows through the ammeter.

OR

4. Determine the current that flows in the ammeter when the arm B is
withdrawn from the circuit.
Series SCI/10/6 Set ~ 6
Roll No. Q.P Code 10/6/6
Candidates must write the Q.P Code
on the title page of the answer-book.

 Please check that this question paper contains 8 printed pages.


 Q.P. Code given on the right hand side of the question paper should be written on the title
page of the answer-book by the candidate.
 Please check that this question paper contains 39 questions.
 Please write down the serial number of the question in the answer-book before
attempting it.
 15 Minute time has been allotted to read this question paper. The question paper will be
distributed at 10:15 a.m From 10.15a.m to 10.30 a.m, the students will read the question
paper only and will not write any answer on the answer –book during this period

SCIENCE
Time Allowed : 3 hours Maximum Marks : 80

General Instructions:

i. This question paper consists of 39 questions in 5 sections.


ii. All questions are compulsory. However, an internal choice is provided in some questions.
A student is expected to attempt only one of these questions.
iii. Section A consists of 20 objective type questions carrying 1 mark each.
iv. Section B consists of 6 Very Short questions carrying 02 marks each. Answers to these
questions should be in the range of 30 to 50 words.
v. Section C consists of 7 Short Answer type questions carrying 03 marks each. Answers to
these questions should be in the range of 50 to 80 words.
vi. Section D consists of 3 Long Answer type questions carrying 05 marks each. Answer to
these questions should be in the range of 80 to 120 words.
vii. Section E consists of 3 source - based/case - based units of assessment of 04 marks
each with sub - parts

[1]
Section A 20 Marks
1. What happens in the test tube shown here?

a) H2O will produce b) SO2 will produce

c) No reaction d) FeO will produce


2. The colour of the ferrous sulphate solution is

a) Blue b) Colourless

c) Pale green d) Reddish-brown


3. When a student added zinc granules to dilute HCl, a colourless and odourless gas was evolved, which was tested
with a burning matchstick, it was observed that

a) The matchstick continued to burn brilliantly. b) The matchstick extinguished and the gas
burnt with pop sound.

c) The matchstick burnt slowly with a blue d) The matchstick extinguished and the gas
flame. burnt with no sound.
4. The raw materials that are required for the manufacturing of washing soda by the Solvay process are:

a) NH4OH, CaCO3, NaCl b) NaCl, CaCO3, NH3

c) NaCl, NH4OH, CaO d) NH3, CaCl2, CaCO3

5. The reaction between CaO and H2O is

a) Highly exothermic with a hissing sound b) Endothermic with a hissing sound

c) Neither endothermic nor exothermic and no d) Exothermic with no sound


sound is produced
6. Which metal is displaced when zinc metal is put in the solution of copper sulphate?

a) Zinc b) All of these

c) Copper d) Sulphate
7. A portion of each of four destarched leaves of a plant was covered with paper strips of various kinds. The plant
was exposed to sunlight for 5 hours. There after the strips were removed and the leaves tested for starch in the
covered portion.
Which one out of the four leaves gave the starch test in the covered portion?

a) That covered with white paper strip. b) That covered with green paper strip.

c) That covered with a transparent paper strip. d) That covered with black paper strip.
8. To prepare a good temporary mount of the petunia leaf peel showing many stomata, the student has to get the
peel from the

a) lower surface of the leaf b) tip of the leaf

c) point of attachment of the leaf to its petiole. d) upper surface of the leaf
9. Four lobed endocrine glands is

a) Pituitary gland b) Adrenal gland

c) Parathyroid gland d) Mucus gland


10. One of the events that does not occur during photosynthesis is:

a) Chlorophyll absorbs solar energy. b) Carbon dioxide is released during the


process.

c) Oxygen is released during the process. d) Carbon dioxide is absorbed during the
process.
11. How much resistance does a superconductor have?

a) Considerable resistance b) Low resistance

c) High resistance d) Zero resistance


12. Single circulation i.e., blood flows through the heart only once during one cycle of passage through the body, is
exhibited by

a) Hippocampus, Exocoetus, Anabas b) Whale, Dolphin, Turtle

c) Labeo, Chameleon, Salamander d) Hyla, Rana, Draco


13. Which of the following is/are good conductor(s) of electricity?

a) Mica, Quartz b) Metals, Mica

c) Metals, Quartz d) Metals, Rubber


14. Which of the following represents voltage?

a) Work done

Current × Time
b) Work done × Time

Current

c) Work done × Charge × Time d) Work done × Charge


15. Which of the following instruments does not have plus (+) or minus (-) sign marked on it while representing in a
circuit diagram?

a) Galvanometer b) Ammeter

c) Cell d) Rheostat
16. Which of the following lenses would you prefer to use while reading small letters found in a dictionary?

a) A convex lens of focal length 50 cm b) A convex lens of focal length 5 cm

c) A concave lens of focal length 50 cm d) A concave lens of focal length 5 cm


17. Assertion (A): Reaction of Quicklime with water is an exothermic reaction.
Reason (R): Quicklime reacts vigorously with water releasing a large amount of heat.

a) Both A and R are true and R is the correct b) Both A and R are true but R is not the
explanation of A. correct explanation of A.

c) A is true but R is false. d) A is false but R is true.


18. Assertion (A): Cytokinins are present, in greater concentration in areas of rapid cell division such as fruits and
seeds.
Reason (R): Cytokinins promote cell division.

a) Both Assertion (A) and Reason (R) are true b) Both Assertion (A) and Reason (R) are true,
and Reason (R) is the correct explanation of but Reason (R) is not the correct
the Assertion (A). explanation of the Assertion (A).

c) Assertion (A) is true, but Reason (R) is d) Assertion (A) is false, but Reason (R) is
false. true.
19. Assertion (A): Alloys are commonly used in electrical heating devices, like electrical iron, toasters etc.
Reason (R): Alloys do not oxidize (burn) readily at high temperatures.

a) Both A and R are true and R is the correct b) Both A and R are true but R is not the
explanation of A. correct explanation of A

c) A is true but R is false. d) A is false but R is true.


20. Assertion (A): As light travels from one medium to another, the frequency of light does not change.
Reason (R): Because frequency is the characteristic of source.

a) Both A and R are true and R is the correct b) Both A and R are true but R is not the
explanation of A. correct explanation of A.

c) A is true but R is false. d) A is false but R is true.

Section B 12 Marks

21. A substance X is used as a building material and insoluble in water. When reacts with dil. HCl, it produces a gas
which turns lime water milky. Predict the substance and write the chemical equation involved.

22. When our finger gets pricked by a sharp nail, we immediately take away our finger. Explain the pathway from
stimulus to response?
23. Given below is a sketch of a leaf partially covered with black paper and which is to be used in the experiment to
show that light is compulsory for the process of photosynthesis. At the end of the experiment, which one of the
leaf parts labeled I, II and III will become blue black when dipped in iodine solution?
OR

Differentiate between Respiration and Photosynthesis.


24. In the figure given below, a narrow beam of white light is shown to pass through a triangular glass prism. After
passing through the prism, it produces a spectrum XY on the screen.

i. Name the phenomenon.


ii. State the colours seen at X and Y.
iii. Why do different colours of white light bend at different angles through a prism?
25. Write two points of difference between electric energy and electric power.

OR
Why is an ammeter likely to be burnt out if it is connected in parallel in a circuit?
26. A concave mirror produces three times magnified (enlarged) real image of an object 10 cm in front of it. Where
is the image located?

Section C 21 Marks

27. Describe an activity to show that the effect of base is nullified by addition of an acid and vice versa. What is the
name given to this reaction? Define it.
28. Justify with the help of an example that displacement reaction is also a redox reaction.
OR
Name the three products of the 'chlor-alkali' process. Write one commercially or industrially important material each
that can be prepared from each of these products.
29. Name the following
i. The process in plants that links light energy with chemical energy.
ii. Organisms that can prepare their own food.
iii. The cell organelle where photosynthesis occurs.
iv. Cells that surround a stomatal pore.
v. Organisms that cannot prepare their own food.
vi. An enzyme secreted from gastric glands in the stomach that act on proteins.
30. During respiration in an organism A, one molecule of glucose produces 2 ATP molecules whereas in respiration
of another organism B, one molecule of glucose produces 38 ATP molecules.

i. Which organism is undergoing aerobic respiration?


ii. Which organism is undergoing anaerobic respiration?
iii. Which type of organism A or B can convert glucose into alcohol?
iv. Name one organism which behaves like A.
v. Name one organism which behaves like B.
31. What is atmospheric refraction? Explain with the help of a labelled diagram that the position of a star as seen by
us is not its true position.
32. Two lamps, one rated 100 W at 220 V and the other 60 W at 220 V, are connected in parallel to electric mains
supply of 220 V. Draw a circuit diagram to show this arrangement and calculate the current drawn by the two
lamps from the mains.
33. i. It would cost a man ₹3.50 to buy 1.0 kW h of electrical energy from the Main Electricity Board. His
generator has a maximum power of 2.0 kW. The generator produces energy at this maximum power for 3
hours. Calculate how much it would cost to buy the same amount of energy from the Main Electricity Board.
ii. A student boils water in an electric kettle for 20 minutes. Using the same mains supply he wants to reduce
the boiling time of water. To do so should he increase or decrease the length of the heating element? Justify
your answer.
Section D 15 Marks
34. What will be the action of the following substances on litmus paper?
Dry HCl gas, Moistened NH3 gas, Lemon juice, Carbonated soft drink, Curd, Soap solution.

OR
What are oxidizing and reducing agents ? Illustrate with suitable examples.
35. Why do we call pituitary gland as the master gland? Where is it located and what are its functions?
OR
Explain the importance of soil for plant growth
36. i. Draw a ray diagram to show the formation of image by a convex lens when an object is placed in front of the
lens between its optical centre and principal focus.
ii. In the above ray diagram mark the object distance (u) and the image distance (v) with their proper signs (+ ve
or - ve as per the new cartesian sign convention) and state how these distances are related to the focal length
(f) of the convex lens in this case.
iii. Find the power of convex lens which forms a real and inverted image of magnification -1 of an object placed
at a distance of 20 cm from its optical centre.
OR
i. Define optical centre of spherical lens.
ii. A divergent lens has a focal length of 20 cm. At what distance should an object of height 4 cm from the optical
centre of the lens be placed, so that its image is formed 10 cm away from the lens. Find the size of the image also.
iii. Draw a ray diagram to show the formation of image in above situation.
Section E
12 Marks
37. Read the text carefully and answer the questions:
When the fats and oil present in the food material get oxidized by the oxygen (of air), their oxidation products
have unpleasant smells and tastes. Due to this taste of food material containing fats and oil change and become
very unpleasant. The condition produced by aerial oxidation of fats and oils in food marked by unpleasant smell
and taste is called rancidity. Rancidity spoils the food material prepared in the fats and oils which have been kept
for a considerable time and makes them unfit for eating.
The development of rancidity in food can be prevented in the following ways-
i. Rancidity can be prevented by adding an antioxidant to foods containing fats and oils.
ii. Rancidity can be prevented by packaging fat and oil-containing food in Nitrogen gas.
iii. Rancidity can be prevented by keeping food in a refrigerator.
(a) What do you understand by oxidation?
(b) How does the food become rancid?
(c) How can we prevent the rancidity of food?
OR
Which type of food material gets spoiled by the phenomenon of rancidity?
38. Read the text carefully and answer the questions:
In animals, control and coordination are provided by nervous and muscular tissues. Touching a hot object is an
urgent and dangerous situation for us. We need to detect it and respond to it. How do we detect that we are
touching a hot object? All information from our environment is detected by the specialised tips of some nerve
cells. These receptors are usually located in our sense organs, such as the inner ear, the nose, the tongue, and so
on. So gustatory receptors will detect taste while olfactory receptors will detect the smell. This information,
acquired at the end of the dendritic tip of a nerve cell sets off a chemical reaction that creates an electrical
impulse. This impulse travels from the dendrite to the cell body, and then along the axon to its end. At the end of
the axon, the electrical impulse sets off the release of some chemicals. These chemicals cross the gap, or
synapse, and start a similar electrical impulse in the dendrite of the next neuron. This is a general scheme of how
nervous impulses travel in the body. A similar synapse finally allows the delivery of such impulses from neurons
to other cells, such as muscles cells or glands.

(a) Why does the flow of signals in a synapse from axonal end of one neuron to dendritic end of another
neuron take place but not in the reverse direction?
(b) From where the electrical impulse travels?
(c) Name the chemical which released at the end of axon to transmit the signal to the other neuron.
OR
What happens at the synapse between 2 neurons?
39. Read the text carefully and answer the questions:
Study the following electric circuit in which the resistors are arranged in three arms A, B and C:

(a) Find the equivalent resistance of arm A.


(b) Calculate the equivalent resistance of the parallel combination of the arms B and C.
(c) Determine the current that flows through the ammeter.
OR
Determine the current that flows in the ammeter when the arm B is withdrawn from the circuit.
Series SCI/10/7 Set ~ 7
Roll No. Q.P Code 10/7/7
Candidates must write the Q.P Code
on the title page of the answer-book.

 Please check that this question paper contains 8 printed pages.


 Q.P. Code given on the right hand side of the question paper should be written on the title
page of the answer-book by the candidate.
 Please check that this question paper contains 39 questions.
 Please write down the serial number of the question in the answer-book before
attempting it.
 15 Minute time has been allotted to read this question paper. The question paper will be
distributed at 10:15 a.m From 10.15a.m to 10.30 a.m, the students will read the question
paper only and will not write any answer on the answer –book during this period

SCIENCE
Time Allowed : 3 hours Maximum Marks : 80

General Instructions:

i. This question paper consists of 39 questions in 5 sections.


ii. All questions are compulsory. However, an internal choice is provided in some questions.
A student is expected to attempt only one of these questions.
iii. Section A consists of 20 objective type questions carrying 1 mark each.
iv. Section B consists of 6 Very Short questions carrying 02 marks each. Answers to these
questions should be in the range of 30 to 50 words.
v. Section C consists of 7 Short Answer type questions carrying 03 marks each. Answers to
these questions should be in the range of 50 to 80 words.
vi. Section D consists of 3 Long Answer type questions carrying 05 marks each. Answer to
these questions should be in the range of 80 to 120 words.
vii. Section E consists of 3 source - based/case - based units of assessment of 04 marks
each with sub - parts

[1]
Section A 20
Marks
1. The copper articles turn green when kept for long due to

a) Corrosion b) Rusting

c) Precipitation d) Rancidity
2. BaCl2 + Na2SO4 →
​ BaSO4 + 2NaCl. It is type of:

a) Both Precipitation reaction and Double b) Double displacement reaction


displacement reaction

c) Decomposition reaction d) Precipitation reaction


3. Select the correct option(s) in the following table:

Option Natural Sources Acid present

(i) Tamarind Tartaric acid

(ii) Tomato Acetic acid

(iii) Ant sting Oxalic acid

(iv) Nettle sting Methanoic acid

a) (i), (ii) and (iv) b) (iii) only

c) (i) only d) (i) and (iv)


4. The chemical formula for plaster of Paris is:

a) 2CaSO4⋅ H2O b) CaSO4⋅ H2O


1

c) CaSO4⋅ 2H2O d) CaSO4⋅ H2O


5. Which type of chemical reaction is shown in the given image?

a) Redox reaction b) Combination reaction

c) Decomposition reaction d) Displacement reaction


6. Given below are certain chemical properties of substances.
A. It turns blue litmus red.
B. It turns red litmus blue.
C. It reacts with zinc and a gas evolves.
D. It reacts with solid sodium carbonate to give brisk effervescence.
Which out of these properties are shown by dilute hydrochloric acid?

a) A, C and D only b) A and C only

c) B, C and D only d) A and B only


7. Bleaching powder is produced by the action of chlorine on

a) calcium chloride b) calcium hydroxide

c) dry slaked lime d) moist slaked lime


8. Which part of alimentary canal receives bile from the liver?

a) Stomach b) Small intestine

c) Large intestine d) Oesophagus


9. Which is the correct sequence of air passage during inhalation?

a) Nasal passage → trachea → pharynx → b) Nostrils → larynx → pharynx → trachea


larynx → alveoli → lungs

c) Nostrils → pharynx → larynx → trachea d) Larynx → nostrils → pharynx → lungs


→ alveoli
10. Diabetes mellitus is caused by a deficiency of:

a) Glucagon b) Insulin

c) Thyroxin d) Adrenaline
11. What is the function of glucagon?

a) Decrease the blood sugar b) All of these

c) Regulate the metabolism of fat d) Increase the blood sugar


12. Which of the following does not respire through the lungs?

a) Duck b) Frog

c) Whale d) Tadpole
13. In a slide projector, light from a lamp is incident on to a slide MN and a lens is used to form an image of the
slide on a screen as shown in figure. What kind of image is formed on the screen?

a) Real, magnified, inverted b) Virtual, diminished, erect

c) Virtual, magnified, erect d) Real, diminished, inverted


14. The maximum resistance which can be made using four resistors each of 2Ω is

a) 2Ω b) 16Ω

c) 8Ω d) 4Ω
15. The potential difference in a circuit in which components are connected in parallel:

a) Gets divided across each component b) Potential difference does not appear

c) Gets distributed equally d) Remains the same across each component


16. Which of the following hormone present in the newborn child that becomes smaller with advancing years?

a) Thymus gland b) Testis

c) Pineal gland d) Ovary


17. Assertion: The following chemical equation, is a balanced chemical equation.
2C2 H6 + 7O2 ⟶ 4C O2 + 6H2 O

Reason: In a balanced chemical equation, the total number of atoms of each element may or may not equal on
both side of the equation.

a) Assertion and reason both are correct b) Assertion and reason both are correct
statements and reason is correct explanation statements but reason is not correct
for assertion. explanation for assertion.

c) Assertion is correct statement but reason is d) Assertion is wrong statement but reason is
wrong statement. correct statement.
18. Assertion (A): The accumulation of lactic acid in the muscles causes muscle cramps.
Reason (R): During vigorous physical exercise leg muscles respire anaerobically.

a) Both A and R are true and R is the correct b) Both A and R are true but R is not the
explanation of A. correct explanation of A.

c) A is true but R is false. d) A is false but R is true.


19. Assertion (A): Resistance of 50 W bulb is greater than that of 100 W.
Reason (R): Resistance of bulb is inversely proportional to rated power.

a) Both A and R are true and R is the correct b) Both A and R are true but R is not the
explanation of A. correct explanation of A.

c) A is true but R is false. d) A is false but R is true.


20. Assertion (A): Humans are not truly aerobic.
Reason (R): They produce lactic acid anaerobically.

a) Both A and R are true and R is the correct b) Both A and R are true but R is not the
explanation of A. correct explanation of A.

c) A is true but R is false. d) A is false but R is true.


Section B 12 Marks
21. Write a balanced chemical equation for each of the following reactions and also classify them.
i. Lead acetate solution is treated with dilute hydrochloric acid to form lead chloride and acetic acid solution.
ii. A piece of sodium metal is added to absolute ethanol to form sodium ethoxide and hydrogen gas.
iii. Hydrogen sulphide gas reacts with oxygen gas to form solid sulphur and liquid water.
22. Classify the following plant movements as tropic or nastic.
i. turning of flower towards light
ii. bending of tentacles in plants
iii. growth of stem away from earth's surface
iv. twirling of a tendril
23. In case of diarrhoea name the major process which occurs in large intestine which is no longer occurring
normally.
OR
Delete the word or words which are inappropriate from the following reactions :
salivary amylase

i. starch + proteins −−−−−−−−−→ Maltose


HCl

ii. Pepsinogen −−→ Peptides + Pepsin


24. A student has focussed the image of an object of height 3 cm on a white screen using a concave mirror of focal
length 12 cm. If the distance of the object from the mirror is 18 cm, find the values of the following:
i. Distance of the image from the mirror
ii. Height of the image
25. Why do veins have thin walls as compared to arteries?
OR
Which part of the brain is more important? Why?
26. Write in tabular form the location and function of the hormones secreted by each of the following glands present
in the human body:
a. Pituitary gland
b. Thyroid gland
c. Pancreas
Section C 21 Marks
27. A metal salt MX when exposed to light, split up to form metal M and a gas X2. Metal M is used in making
ornaments whereas gas X2 is used in making bleaching powder. The salt MX is itself used in black and white
photography.
i. Identify metal M and gas X2.
ii. Mention the type of chemical reaction involved when salt MX is exposed to light.
28. When zinc metal is treated with a dilute solution of a strong acid, a gas is evolved, which is utilised in the
hydrogenation of oil. Name the gas evolved. Write the chemical equation of the reaction and also write a test to
detect the gas formed.
OR
Why do fire flies glow at night?
29. In each of the following situations what happens to the rate of photosynthesis?
i. Cloudy days
ii. No rainfall in the area
iii. Good manuring in the area
iv. Stomata get blocked due to dust
30. The diagram given below is the experiment set-up of show that carbon dioxide is given out during respiration. In
this set-up what does test tube marked (A) contain? What are its role in the experiment

31. How can you distinguish between plane mirror, convex mirror and concave mirror by merely looking at the
image formed in each case?
32. In the circuit diagram given in figure, suppose the resistors R1, R2 and R3 have the values 5Ω , 10Ω , 30Ω ,
respectively, which have been connected to a battery of 12 V. Calculate:

a. the current through each resistor,


b. the total current in the circuit, and the total circuit resistance.
33. A battery E is connected to three identical lamps P, Q and R as shown in figure:

Initially the switch S is kept open and the lamp P and Q are observed to glow with same brightness. Then switch
S is closed. How will the brightness of the glow of bulbs P and Q will change? Justify your answer.
Section D 15 Marks
34. A solid substance P which is very hard and is used in the construction of many buildings, especially flooring.
When substance P is heated strongly, it decomposes to form another solid Q and a gas R is given out. Solid Q
reacts with water with the release of a lot of heat to form a substance S. When gas R is passed into a clear
solution of substance S, then a white precipitate of substance T is formed. The substance T has the same
chemical composition as starting substance P.
i. What is substance P? Write its common name as well as chemical formula.
ii. What is substance Q?
iii. What is gas R?
iv. What is substance S? What is its clear solution known as?
v. What is substance T? Name any two natural forms in which substance T occurs in nature.
OR
Define water of crystallisation. Give the chemical formula for two compounds as examples. How can it be proved
that the water of crystallisation makes a difference in the state and colour of the compounds?
35. What is the difference between an intercellular and intracellular digestion?
OR
Some situations in our day to day life require quick response from our body. Illustrate the sentence with the help of
suitable diagram and common examples.
36. We wish to obtain an erect image of an object, using a concave mirror of focal length 15 cm. What should be the
range of distance of the object from the mirror? What is the nature of the image? Is the image larger or smaller
than the object? Draw a ray diagram to show the image formation in this case.
OR
An object 6 cm in size is placed at 50 cm in front of a convex lens of focal length 30 cm. At what distance from the
lens should a screen be placed in order to obtain a sharp image of the object? Find the nature and size of the image.
Also draw labelled ray diagram to show the image formation in this case.
Section E 12 Marks
37. Read the text carefully and answer the questions:
Copper sulphate crystal contains water of crystallisation when the crystal is heated the water is removed and salt
turns white. The crystal can be moistened again with water. The water of crystallisation is the fixed number of
water molecules present in 1 formula unit of copper sulphate. On heating gypsum at 373K, it loses water
molecules and became calcium sulphate hemihydrate.

(a) If the crystal is moistened with water, then which colour of the crystal reappears?
(b) What is the commercial name of calcium sulphate hemihydrate?
(c) How many water molecules are present in one formula unit of copper sulphate?
OR
What is obtained when gypsum is heated at 373K?
38. Read the text carefully and answer the questions:
Animals have a nervous system for controlling and coordinating the activities of the body. But plants have
neither a nervous system nor muscles. So, how do they respond to stimuli? When we touch the leaves of a chui-
mui (the ‘sensitive’ or ‘touch-me-not’ plant of the Mimosa family), they begin to fold up and droop. When a
seed germinates, the root goes down, the stem comes up into the air. What happens? Firstly, the leaves of the
sensitive plant move very quickly in response to touch.
There is no growth involved in this movement. On the other hand, the directional movement of a seedling is
caused by growth. If it is prevented from growing, it will not show any movement.

(a) Write the types of movement.


(b) Give an example of a plant hormone that promotes growth.
(c) What is the function of the nervous system?
OR
How is the movement of leaves of the sensitive plant different from the movement of a shoot towards light
?
39. Read the text carefully and answer the questions:
The ability of a medium to refract light is expressed in terms of its optical density. Optical density has a definite
connotation. It is not the same as mass density. On comparing two media, the one with the large refractive
index is optically denser medium than the other. The other medium with a lower refractive index is optically
rarer. Also the speed of light through a given medium is inversely proportional to its optical density.
(a) Determine the speed of light in diamond if the refractive index of diamond with respect to vacuum is 2.42.
Speed of light in vacuum is 3 × 108 m/s.
(b) Refractive indices of glass, water and carbon disulphide are 1.5, 1.33 and 1.62 respectively. If a ray of
light is incident in these media at the same angle (say θ ), then write the increasing order of the angle of
refraction in these media.
(c) The speed of light in glass is 2 × 108 m/s and in water is 2.25 × 108 m/s.
a. Which one of the two is optically denser and why?
b. A ray of light is incident normally at the water-glass interface when it enters a thick glass container
filled with water. What will happen to the path of the ray after entering the glass? Give reason.
OR
The absolute refractive indices of water and glass are 4

3
and 3

2
respectively. If the speed of light in glass is

2 × 108 m/s, find the speed of light in (i) vacuum and (ii) water.
Series SCI/10/8 Set ~ 8
Roll No. Q.P Code 10/8/8
Candidates must write the Q.P Code
on the title page of the answer-book.

 Please check that this question paper contains 8 printed pages.


 Q.P. Code given on the right hand side of the question paper should be written on the title
page of the answer-book by the candidate.
 Please check that this question paper contains 39 questions.
 Please write down the serial number of the question in the answer-book before
attempting it.
 15 Minute time has been allotted to read this question paper. The question paper will be
distributed at 10:15 a.m From 10.15a.m to 10.30 a.m, the students will read the question
paper only and will not write any answer on the answer –book during this period

SCIENCE
Time Allowed : 3 hours Maximum Marks : 80

General Instructions:

i. This question paper consists of 39 questions in 5 sections.


ii. All questions are compulsory. However, an internal choice is provided in some questions.
A student is expected to attempt only one of these questions.
iii. Section A consists of 20 objective type questions carrying 1 mark each.
iv. Section B consists of 6 Very Short questions carrying 02 marks each. Answers to these
questions should be in the range of 30 to 50 words.
v. Section C consists of 7 Short Answer type questions carrying 03 marks each. Answers to
these questions should be in the range of 50 to 80 words.
vi. Section D consists of 3 Long Answer type questions carrying 05 marks each. Answer to
these questions should be in the range of 80 to 120 words.
vii. Section E consists of 3 source - based/case - based units of assessment of 04 marks
each with sub - parts

[1]
Section A
20 Marks
1. Which characteristic is observed by the reaction shown in the given image?

a) Formation of a precipitate b) Change in temperature

c) Evolution of a gas d) Both change in temperature and evolution


of gas
2. Which one of the following molecules is produced initially when glucose breaks down in the cytoplasm of a cell
in aerobic as well as anaerobic respiration?

a) Carbon dioxide b) Ethanol

c) Pyruvate d) Lactic acid


3. What is CaOCl2 commonly called?

a) Chloride of lime b) Bleaching powder


c) Both Chloride of lime and Bleaching d) Calcium chloride
powder
4. As the pH value of solution increases from 7 to 14 , it represents

a) a decrease on the concentration of OH– ions b) an increase in the concentration in OH– ion

c) no change in the concentration of OH– ions d) an increase in the concentration of H3O+

ions.
5. A student took Cu, Al, Fe and Zn strips separately in four test tubes labelled I, II, III and IV. He added 10 mL of
freshly prepared ferrous sulphate solution to each test tube as shown below:
Black residue would be obtained in test tubes

a) II and IV b) I, II and IV

c) II and III d) III and IV


6. Metal X is found in earth’s crust. This metal forms a reddish brown substance when exposed to moist air. When
a blue coloured solution Y is stored in a container made of X, the solution turns green and a reddish brown metal
Z gets deposited on the container. X, Y and Z are respectively

a) Fe, CuSO4, Cu b) Zn, ZnSO4, Fe

c) Cu, FeSO4, Fe d) Cu, CuSO4, Fe

7. Fresh milk has a pH of 6. To delay its curdling, a chemical substance is added to it, which is:

a) Baking soda (Sodium hydrogen carbonate) b) Sodium carbonate

c) Sodium hydroxide (Caustic soda) d) Baking powder


8. The xylem in plants are responsible for:

a) transport of amino acids b) transport of oxygen

c) transport of food d) transport of water


9. Which hormone causes uterine contraction that leads to labour pain?

a) Vasopressin b) Thyroxin

c) Estrogen d) Progesterone
10. The energy-rich compound produced through respiration is

a) ADP b) Pyruvic acid

c) AMP d) ATP
11. Progesterone and relaxin bring about changes in the body of a

a) Teenaged girl b) Old woman

c) Pregnant woman d) Young girl


12. When students observed a stained epidermal peel of a leaf under the microscope, it appeared pinkish red. The

stain used was

a) iodine b) colchicine

c) safranin d) acetocarmine
13. To determine the focal length of a concave mirror, a student focuses a distant object using the concave mirror.
The best object can be:

a) a distant tree b) All of these

c) classroom window d) sun


14. In a house, two 60W electric bulbs are lighted for 4 hours and three 100W bulbs for 5 hours every day. The
electric energy is consumed in 30 days:
a) 59.4 kWh b) 79.4 kWh

c) 100 kWh d) 45 kWh


15. A well-stained leaf peel preparation when focused under the high power of the microscope would show
A. epidermal cells, stomata, guard cells each with one nucleus and many chloroplasts
B. epidermal cells, stomata, guard cells with many nuclei and one chloroplast each
C. stomata and guard cells without nuclei or chloroplast
D. stomata but no guard cells or epidermal cells

a) (B) b) (A)

c) (C) d) (D)
16. Spinal cord originates from:

a) cerebrum b) pons

c) cerebellum d) medulla
17. Assertion (A): Photosynthesis is considered as an endothermic reaction.
Reason (R): Energy gets released in the process of photosynthesis.

a) Both A and R are true and R is the correct b) Both A and R are true but R is not the
explanation of A. correct explanation of A.

c) A is true but R is false. d) A is false but R is true.


18. Assertion (A): Phototropism is caused by auxin.
Reason (R): When light is coming from one side of the plant, auxin diffuses towards the shady side of the shoot.

a) Both A and R are true and R is the correct b) Both A and R are true but R is not the
explanation of A. correct explanation of A.

c) A is true but R is false. d) A is false but R is true.


19. Assertion (A): In a circuit which is having 3 series resistors of R ohm each, the total resistance of the circuit
will be 3R.
Reason (R): As in parallel circuit the resultant resistance will be 1

R
=
1

R1
+
1

R2
+
1

R3
.

a) Both A and R are true and R is the correct b) Both A and R are true but R is not the
explanation of A. correct explanation of A.

c) A is true but R is false. d) A is false but R is true.

20. Assertion (A): A person has lost most of its intelligence memory and judgment.
Reason (R): A person has operated a tumour located in the cerebrum.

a) Both A and R are true and R is the correct b) Both A and R are true but R is not the
explanation of A. correct explanation of A.

c) A is true but R is false. d) A is false but R is true.


Section B 12 Marks
21. Why is respiration considered as exothermic reaction ? Explain.
22. Differentiate between single and double circulation found in vertebrates.
23. What is haemoglobin? State the consequences of deficiency of haemoglobin in our bodies.
OR
What is the role of saliva in the digestion of food?
24. In what way is the word AMBULANCE printed in front of the hospital vans? Why is it printed this way?
25. How is an electric impulse created in human nervous system? Identify the parts of a neuron which helps the
nerve impulse to travel
i. towards the cell body
ii. away from the cell body
OR
Answer the following:
a. Which hormone is responsible for the changes noticed in females at puberty?
b. Dwarfism results due to deficiency of which hormone?
c. Blood sugar level rises due to deficiency of which hormone?
d. Iodine is necessary for the synthesis of which hormone?
26. Ravi was diagnosed with kidney failure. He was advised to undergo dialysis for twice in a week. After
undergoing dialysis for a month he was unable to continue the treatment as it is very painful. What alternative
treatment solution can you suggest for Ravi other than dialysis.
Section C 21 Marks
27. Write the balanced chemical equations for the following reactions and identify the type of reaction in each case.
i. In thermite reaction, iron (III) oxide reacts with aluminium and gives molten iron and aluminium oxide.
ii. Magnesium ribbon is burnt in an atmosphere of nitrogen gas to form solid magnesium nitride.
28. Sugandha prepares HCl gas in her school laboratory using certain chemicals. She puts both dry and wet litmus
paper in contact with the gas.
i. Name the reagent used by Sugandha to prepare HCl gas.
ii. State the colour change observed with dry and wet litmus paper.
iii. Show the formation of ions when HCl gas combines with water.
OR
A student adds water to a substance X taken in beaker. He feels the beaker turning hot and a hissing sound is
produced.Why does this happen? Write a chemical equation for the reaction. State the type of this reaction.
29. What do you understand by parasitic nutrition?
30. Differentiate between exocrine gland and endocrine gland.
31. A pencil when dipped in water in a glass tumbler appears to be bent at the interface of air and water. Will the
pencil to be bent to the same extent, if instead of water we use liquids like, kerosene or turpentine? Support your
answer with reasons.
32. Given in fig. is the circuit diagram in which three resistors of 1Ω, 2Ω and 3Ω are connected to cell of e.m.f. 2V
and internal resistance 0.5 Ω.
i. Calculate the total resistance of the circuit.
ii. What is the reading of ammeter and What will be ammeter reading if an exactly similar cell is connected in

series with the given cell ?

33. For the circuit shown in fig. what is the value of


a. total resistance and current through 6W resistor
b. potential difference across 12 Wresistor?

Section D 15 Marks
34. a. Complete and balance the following chemical equations:
i. Al2O3 + HCl ⟶
ii. K2O + H2O ⟶
iii. Fe + H2O ⟶

b. An element X displaces iron from the aqueous solution of iron sulphate. List your observations if the element
X is treated with the aqueous solutions of copper sulphate, zinc sulphate and silver nitrate. Based on the
observations arrange X, Zn, Cu and Ag in increasing order of their reactivities.
OR
(a) The PH of rain water collected from two cities A and B was found to be 6 and 5 respectively. Water of which city
is more acidic? Find out the ratio of hydrogen ion concentration in the two samples of rain water?
(b) Arrange the following in order (ascending) of their PH values.
N aOH solution, Blood, lemon Juice.
35. What are reflex actions? Give two examples. Explain a reflex arc.
OR
Describe the central nervous system in human beings.
36. A student focussed the image of a candle flame on a white screen using a convex lens. He noted down the
position of the candle screen and the lens as under Position of candle = 12.0 cm Position of convex lens = 50.0
cm Position of the screen = 88.0 cm
i. What is the focal length of the convex lens?
ii. Where will the image be formed if he shifts the candle towards the lens at a position of 31.0 cm?
iii. What will be the nature of the image formed if he further shifts the candle towards the lens?
iv. Draw a ray diagram to show the formation of the image in case (iii) as said above.
OR
One-half of a convex lens is covered with a black paper. Will this lens produce a complete image of the object?
Verify your answers experimentally. Explain your observations.
Section E 12 Marks
37. Read the text carefully and answer the questions:
A scale for measuring hydronium ion in a solution is called the pH scale. The pH of a neutral solution is 7. A
value of less than 7 on the pH scale represents an acidic solution. As the pH value, increases from 7 to 14 it
represents OH- ion concentration in solution i.e a basic solution.

(a) What is the pH range of the Human Body?


(b) The strength of acid and bases depends on which factor?
(c) If the pH of soil X is 7.5 while that of soil Y is 4.5, then which soil should be treated with powdered chalk
to adjust its pH?
OR
Tooth decay starts when the pH of the mouth is lower than which pH?
38. Read the text carefully and answer the questions:
Animals have a nervous system for controlling and coordinating the activities of the body. But plants have
neither a nervous system nor muscles. So, how do they respond to stimuli? When we touch the leaves of a chhui-
mui (the ‘sensitive’ or ‘touch-me-not’ plant of the Mimosa family), they begin to fold up and droop. When a
seed germinates, the root goes down, the stem comes up into the air. What happens? Firstly, the leaves of the
sensitive plant move very quickly in response to touch. There is no growth involved in this movement. On the
other hand, the directional movement of a seedling is caused by growth. If it is prevented from growing, it will
not show any movement. So plants show two different types of movement - one dependent on growth and the
other independent of growth.

(a) Plants neither have nervous system nor muscles, then how does chemical coordination occur in plants?
(b) Why Mimossa pudica leaves drop down when we touched? Write its another name also.
(c) What is turgor movement?
OR
What is a tropic movement? Explain with an example
39. Read the text carefully and answer the questions:
We know that a battery or a cell is a source of electrical energy. The chemical reaction within the cell generates
the potential difference between its two terminals that sets the electrons in motion to flow the current through a
resistor or a system of resistors connected to the battery. To maintain the current, the source has to keep
expanding its energy. Where does this energy go? A part of the source energy in maintaining the current may be
consumed for useful work (like in rotating the blades of an electric fan). The rest of the source energy may be
expended in heat to raise the temperature of the gadget. We often observe this in our everyday life. For example,
an electric fan becomes warm if used continuously for a long time, etc. On the other hand, if the electric circuit
is purely resistive, that is, a configuration of resistors only connected to a battery; the source energy continually
gets dissipated entirely in the form of heat. This is known as the heating effect of electric current. This effect is
utilized in devices such as an electric heater, electric iron, etc.

(a) Explain Joule's heating law.


(b) In practical situations, when an electric appliance is connected to a known voltage source, then how does
the heating effect of electric current can be calculated?
(c) Write the relation between heat energy produced in a conductor when a potential difference V is applied
across its terminals and a current I flows through for t.
OR
Two identical wires one of nichrome and the other of copper are connected in series and a current (I) is
passed through them. State the change observed in the temperatures of the two wires.
Series SCI/10/9 Set ~ 9
Roll No. Q.P Code 10/9/9
Candidates must write the Q.P Code
on the title page of the answer-book.

 Please check that this question paper contains 13 printed pages.


 Q.P. Code given on the right hand side of the question paper should be written on the title
page of the answer-book by the candidate.
 Please check that this question paper contains 39 questions.
 Please write down the serial number of the question in the answer-book before
attempting it.
 15 Minute time has been allotted to read this question paper. The question paper will be
distributed at 10:15 a.m From 10.15a.m to 10.30 a.m, the students will read the question
paper only and will not write any answer on the answer –book during this period

SCIENCE
Time Allowed : 3 hours Maximum Marks : 80

General Instructions:

i. This question paper consists of 39 questions in 5 sections.


ii. All questions are compulsory. However, an internal choice is provided in some questions.
A student is expected to attempt only one of these questions.
iii. Section A consists of 20 objective type questions carrying 1 mark each.
iv. Section B consists of 6 Very Short questions carrying 02 marks each. Answers to these
questions should be in the range of 30 to 50 words.
v. Section C consists of 7 Short Answer type questions carrying 03 marks each. Answers to
these questions should be in the range of 50 to 80 words.
vi. Section D consists of 3 Long Answer type questions carrying 05 marks each. Answer to
these questions should be in the range of 80 to 120 words.
vii. Section E consists of 3 source - based/case - based units of assessment of 04 marks
each with sub - parts

[1]
Section A 20 Marks

1 The food items like cheese that is shown in the given below image become unfit
for eating. This happens due to:

a) Corrosion

b) Rusting

c) Dusting
d) Rancidity

2 Which among the following is(are) double displacement reaction(s)?

1. Pb + CuCl2 → PbCl 2 + Cu

2. Na2 SO 4 + BaCl 2 → BaSO 4 + 2NaCl

3. C + O2 → CO 2

4. CH4 + 2O 2 → CO 2 + 2H 2 O

a) (iii) and (iv)

b)) i) only

c) (i) and (ii)

d) ( ) and (iv)

3 The pH of the gastric juices released during digestion is

a) equal to 0

b)) qual to 7

c) more than 7

d) less than 7
4 A student wrote three statements about rancidity :

1. When fats and oils are reduced, they become rancid.

2. Chips manufacturers usually flush chips bags with oxygen to prevent


rancidity.

3. Rancidity is prevented by adding substances called antioxidants to food.

a) Statement (i), (ii) and (iii)

b) Statement (i) and (iii) only

c) Statement (i) only

d) Statement (iii) only

5 On adding a few drops of universal indicator to three unknown colourless


solution (P), (Q) and (R) taken separately in three test tubes shown in the
following diagrams, a student observed the changes in colour as green in (P), red
in (Q) and violet in (R).

The decreasing order of pH of the solutions taken is :

a) R > Q > P

b)) >P>R

c) R > P > Q

d) P > Q > R

6 Acid used for manufacture of fertilizers and explosives is:


a) Hydrochloric acid

b) Nitric acid

c) Phosphoric acid

d) Sulphuric acid
7 The name was given to a reaction in which one element, molecule or radical is
removed by anotheris:

a) Displacement reaction

b) Decomposition reaction

c) Combination reaction

d) Double displacement reaction

8 On complete digestion of fats, the end products are

a) Fatty acid and amino acids

b) Fatty acids and glycerol


c) Glucose and amino acids

d) Glucose and glycerol

9 People suffering from diabetes mellitus are unable to secrete

a) Insulin

b) Adrenaline

c) Thyroxine

d) Vasopressine

10 Which of the following precautions are to be taken for a successful run of the
experiment to show that carbon dioxide is given out during respiration?

1. Cork should be airtight.

2. Seeds in the flask should be totally dry.

3. A small tube with a freshly prepared KOH solution should be placed in the
flask.

4. The end of the delivery tube should be above water level.

The correct answer is

a) A, B and C

b) A and C

c) A and B

d) A, B and D
11 Which statement is not true about thyroxin?
a) Iron is essential for the synthesis of thyroxin

b) Thyroxin is also called thyroid hormone

c) It regulates carbohydrates, protein and fat metabolism in the body

d) Thyroid gland requires iodine to synthesise thyroxin

12 Cramps are caused by heavy exercise resulting in the accumulation of


a) Heat

b) Ethanol
c) arbon dioxide

d) Lactic acid

13 Which of the following is likely to be the correct wattage for an electric iron used
in our homes?

a) 60 W

b) 2000 W

c) 850 W

d) 250 W

14 Consider the room temperature is 24°C in summer, the electrical resistance of


thermocoil which is used in the AC unit is 150𝛺 . Then calculate the temperature
of the thermocoil if the electrical resistance is 175 𝛺 . Given the temperature
coefficient of the thermocoil is 2.98 × 10 −4 °C −1

a) 583 °C

b) 512 °C

c) 597 °C

d) 546 °C
15 Which of the following equations is the summary of photosynthesis?
a) 6CO2 + 12H 2 O + Chlorophyll + Sunlight → C 6 H 12 O 6 + 6O 2 + 6H 2 O

b) 6CO2 + 12H 2 O + Chlorophyll + Sunlight → C 6 H 12 O 6 + 6CO 2 + 6H 2 O

c) 6CO2 + 12H 2 O → C 6 H 12 O 6 + 6O 2 + 6H 2 O

d) 6CO2 + H 2 O + Sunlight → C 6 H 12 O 6 + O 2 + 6H 2 O

16 The hormone secreted by the thyroid gland is


1. Thyroxin

2. Calcitonin

3. Adrenaline

4. Insulin

a) B and C

b) A and B
c) A, B and D

d) All of these

17 Assertion (A): White silver chloride turns grey insunlight.


Reason (R): Decomposition of silver chloride in the presence of sunlight takes
place to form silver metal and chlorine gas.

a) Both A and R are true and R is the correct explanation of A.

b)) oth A and R are true but R is not the correct explanation of A.

c) A is true but R is false.

d) A is false but R is true.

18 Assertion (A): The release of energy in the aerobic process is much more than in
the anaerobic process.

Reason (R): Each glucose molecule produces 2 molecules of ATP and 38


molecules of ATP in aerobic and anaerobic respiration, respectively.

a) Both A and R are true and R is the correct explanation of A.

b) Both A and R are true but R is not the correct explanation of A.

c) A is true but R is false.

d) A is false but R is true.


19 Assertion (A): It is impossible to see a virtual image.
Reason (R): The rays that seen to emanate from a virtual image don’t, in fact,
emanates from the image.

a) Both A and R are true and R is the correct explanation of the assertion.

b) Both A and R are true but R is not the correct explanation of A.

c) A is true but R is false.

d) A is false but R is true.

20 Assertion (A): Our body maintains blood sugar levels.


Reason (R): Pancreas secretes insulin which helps to regulate blood sugar levels
in the body.

a) Both A and R are true and R is the correct explanation of A.

b) Both A and R are true but R is not the correct explanation of A.


c) A is true but R is false.

d) A is false but R is true.

Section B 12 Marks
21 Test tube A contains lead (II) nitrate and test tube B contains potassium iodide
solution.

1. What happens when solution A is added to solution B?

2. What happens when the order of addition is reversed for the above two
reactants?

22 The figure represents human brain. Study the above figure and answer the
following:

1. Name the parts labeled (a), (b) and (c)

2. Give one function of parts (a), (b) and (c)


23 What is meant by circulation?
OR

State the role played by transpirational pull in transportation?

24 On the way from Kanpur to Delhi there were four friends. Sunil was driving the
car and saw from his side mirror that the car which was behind their car had met
an accident. He suddenly applied the brake even after his friends asked him to
leave the situation as it is. But Sunil did not agree and get down of car and
persuaded his friends to help the injured. All of them took the injured person to
the nearest hospital. After taking first aid from hospital the victim thanked and
pleased them for saving his life. Read the above passage and answer the following
questions:

1. Name the type of mirror from which Sunil saw the accident.

2. Why this mirror is used as a side mirror in vehicles?

3. What can you learn from the Sunil’s character?

25 Where do plants get each of the raw materials required for photosynthesis?
OR

During a street fight between two individuals, mention the effects on the
following organs by the autonomic nervous system, in the table given below:

26 What are hypertension and hypotension?


Section C 21 Marks

27 A student burnsa metal A found in the form of ribbon.The ribbon burnswith a


dazzling flame & a white powder B is formed which is basic in nature. Identify A
& B. Write the balanced chemical equation for the reaction involved.
28 State reason for the following:
1. Lemon is used for restoring the shine of tarnished copper vessels.

2. A metal sulphide is converted into its oxide to extract the metal from the
sulphide ore.

3. Copper wires are used in electrical appliances.


OR

A silvery - white metal X taken in the form of ribbon, when ignited, burns in air
with a dazzling white flame to form a white powder Y. When water is added to
powder Y, it dissolves partially to form another substance Z.

1. What could metal X be?

2. What is powder Y?

3. With which substance metal X combines to form powder Y?

4. What is substance Z? Name one domestic use of substance Z.

5. Write a balanced chemical equation of the reaction which takes place


when metal X burns in air to form powder Y.

healthy for long? Give reasons for your answer.


29 Leaves of healthy potted plant were coated with vaseline. Will this plant remain
30 Why the leaf is boiled in alcohol for a few minutes using a water bath in an
experiment to show that sunlight is necessary for photosynthesis?
31 Observe the following incomplete ray diagram of an object where the image A’B’
is formed after refraction from a convex lens.

On the basic of above information fill in the blanks.

1. The position of object AB would have been...

2. Size of the object would have been ... than the size of image.

32 Find out the following in the electric circuit given in Figure

1. The potential difference across 4𝛺 resistance

2. The power dissipated in 4𝛺 resistor

33 Should the resistance of an ammeter be low or high? Give reason.


Section D 15 Marks

34 The white solid compound A decomposes quite rapidly on heating in the


presence of a black substance X to form a solid compound B and a gas C. When an
aqueous solution of compound B is reacted with silver nitrate solution, then a
white precipitate of silver chloride is obtained along with potassium nitrate
solution. Gas C does not burn itself but helps to burn other things.

1. What is compound A?

2. What is compound B?

3. What is gas C?

4. What do you think is the black substance X? What is its function?

5. What is the general name of substances like X?

OR

Solution A turns the universal indicator blue to purple whereas solution B turns
the universal indicator orange to red.
1. What will be the action of solution A on litmus?

2. What will be the action of solution B on litmus?

3. Name any two substances which can give solutions like A.

4. Name any two substances which can give solutions like B.

5. What sort of reaction takes place when solution A reacts with solution B?

35 1. Why is nutrition necessary for the human body?

2. What causes the movement of food inside the alimentary canal?

3. Why is the small intestine in herbivores longer than in carnivores?

4. What will happen if the mucus is not secreted by the gastric glands?

OR

Explain the reflex action by means of reflex arc with diagram.

36 Rishi went to a palmist to show his palm. The palmist used a special lens for this
purpose.
1. State the nature of the lens and the reason for its use.

2. Where should the palmist place/hold the lens so as to have a real and
magnified image of an object?

3. If the focal length of this lens is 10 cm and the lens is held at a distance of
5 cm from the palm, use lens formula to find the position and size of the
image.

OR

An object 4.0 cm in size, is placed 25.0 cm in front of a concave mirror of focal


length 15.0 cm.

1. At what distance from the mirror should a screen be placed in order to


obtain a sharp image?

2. Find the size of the image.

3. Draw a ray diagram to show the formation of image in this case.

Section E 12 Marks

37 Read the text carefully and answer the questions: The dissolving of an acid or
a base in water is a highly exothermic reaction. Care must be taken while mixing
concentrated nitric acid or sulphuric acid with water. The acid must always be
added slowly to water with constant stirring. If water is added to a concentrated

acid, the heat generated may cause the mixture to splash out and cause burns.
The glass container may also break due to excessive local heating. Look out for
the warning sign on the can of concentrated sulphuric acid and on the bottle of
sodium hydroxide pellets.

1. What is the exothermic reaction?

2. Write an example of an exothermic reaction.

3. Why is caution necessary when mixing concentrated acid with water?


OR

How will you obtainsulphuric acid from an acidic oxide?

38 Read the text carefully and answer the questions: Environmental triggers
such as light, or gravity will change the directions that plant parts grow in. These
directional, or tropic, movements can be either towards the stimulus or away
from it. So, in two different kinds of phototropic movement, shoots respond by
bending towards light while roots respond by bending away from it. How does
this help the plant? Plants show tropism in response to other stimuli as well. The
roots of a plant always grow downwards while the shoots usually grow upwards
and away from the earth. This upward and downward growth of shoots and
roots, respectively, in response to the pull of earth or gravity, is, obviously,
geotropism. If ‘hydro’ means water and ‘chemo’ refers to chemicals, what would
‘hydrotropism’ and ‘chemotropism’ mean? Can we think of examples of these
kinds of directional growth movements? One example of chemotropism is the
growth of pollen tubes towards ovules, about which we will learn more when we
examine the reproductive processes of living organisms.

1. Where does negative phototropism occur in plants?


2. Phototropism in shoots is attributed due to which plant hormone?

3. Tendrils exhibit/ twining of tendrils show which type of tropic


movement?

OR

If the stem grows towards sunlight and the root grows just opposite to it,
then what type of movement of the stem is it?
39 Read the text carefully and answer the questions: A student took three
concave mirrors of different focal lengths and performed the experiment to see
the image formation by placing an object at different distances with these
mirrors as shown in the following table.

1. List two properties of the image formed in Case I.

2. In which one of the cases given in the table, the mirror will form real
image of same and why?

3. Name the type of mirror used by dentists. Give reason why do they use
such type of mirrors.

4. Look at the table and identify the situation (object distance and focal
length) which resembles the situation in which concave mirrors are used
as shaving mirrors? Draw a ray diagram to show the image formation in
this case.

OR

How is a virtual image formed by a convex lens different from that formed by a
concave lens? Under what conditions do a convex and a concave lens form virtual
images
Series SCI/10/10 Set ~ 10
Roll No. Q.P Code 10/10/10
Candidates must write the Q.P Code
on the title page of the answer-book.

 Please check that this question paper contains 13 printed pages.


 Q.P. Code given on the right hand side of the question paper should be written on the title
page of the answer-book by the candidate.
 Please check that this question paper contains 39 questions.
 Please write down the serial number of the question in the answer-book before
attempting it.
 15 Minute time has been allotted to read this question paper. The question paper will be
distributed at 10:15 a.m From 10.15a.m to 10.30 a.m, the students will read the question
paper only and will not write any answer on the answer –book during this period

SCIENCE
Time Allowed : 3 hours Maximum Marks : 80

General Instructions:

i. This question paper consists of 39 questions in 5 sections.


ii. All questions are compulsory. However, an internal choice is provided in some questions.
A student is expected to attempt only one of these questions.
iii. Section A consists of 20 objective type questions carrying 1 mark each.
iv. Section B consists of 6 Very Short questions carrying 02 marks each. Answers to these
questions should be in the range of 30 to 50 words.
v. Section C consists of 7 Short Answer type questions carrying 03 marks each. Answers to
these questions should be in the range of 50 to 80 words.
vi. Section D consists of 3 Long Answer type questions carrying 05 marks each. Answer to
these questions should be in the range of 80 to 120 words.
vii. Section E consists of 3 source - based/case - based units of assessment of 04 marks
each with sub - parts

[1]
Section A 20 Marks

1 The image shows a substance X formed when potassium iodide and lead nitrate
react together. What is X ?

a) Lead iodide, Precipitate

b) No change
c) Lead sulphate, Gas

d) Potassium nitrate, Precipitate

2 Which metal is displaced when lead is put in the solution of copper chloride?

a) Chlorine

b) Copper

c) Lead

d) All of these

3 Certain chemical properties of a substance are given below:

1. It turns methyl orange red.

2. It turns turmeric paper red.

3. It reacts with zinc and a gas is evolved.

4. It reacts with solid sodium carbonate to give brisk effervescence.


Which of the above properties are shown by dilute sodium hydroxide?

a) (ii) and (iii) only

b) (i) and (ii) only

c) (i), (iii) and (iv) only

d) (iii) and (iv) only

4 Strong heating of ferrous sulphate leads to the formation of a brown solid and
two gases. This reaction can be categorised as:

a) displacement and redox

b) displacement and endothermic

c) decomposition and exothermic

d) decomposition and redox

5 Which is a base and not an alkali?

a) Fe(OH)3

b) NH4 OH

c) NaOH
d) KOH

6 Decomposition of vegetable matter into compost is an example of:

a) Exothermic reaction

b) Endothermic reaction

c) Redox reaction

d) Combination reaction
7 WhenHCl reacts with granulated zinc

a) odourof chlorine is observed

b) the surface of the metal turns shining

c) the reaction mixture turns milky

d) a colorless and odorless gas evolves with bubbles.


8 Refer to the given figure and select the incorrect statement regarding it.

a) It is the longest cell in human body.

b) It transmits messages in the form of nerve impulses.

c) It uses both amino acids and fatty acids as a respiratory substrate.

d) It is not repaired, when injured.

9 Which of the following is due to the parasympathetic system:

1. Dilation of blood vessels

2. Contraction of urinary blood vessels

3. Inhibits gastric secretion

4. Constrict the eye pupil

a) A, B and D
b) A and C

c) All of these

d) A and B

10 Which of the following precautions are to be taken for a successful run of the
experiment to show that carbon dioxide is given out during respiration?

1. Cork should be airtight.

2. Seeds in the flask should be totally dry.

3. A small tube with a freshly prepared KOH solution should be placed in the
flask.

4. The end of the delivery tube should be above water level.

The correct answer is

a) A, B and C

b) A and C

c) A and B

d) A, B and D
11 During respiration exchange of gases take place in
a) alveoli of lungs

b) throat and larynx

c) alveoli and throat

d) trachea and larynx

12 Refer to the following figure of phloem tissue. Which of the following statements
correctly describes the relationship between cells Y and Z?

a) Ycontains ribosomes that synthesise proteins required by Z.

b) Y provides the carbohydrates that are transported by Z.


c) Y transports sucrose, while Z transports water and minerals.

d) Y helps to filter the liquid transported in Z to facilitate translocation.

13 The lamps in a household circuit are connected in parallel because:


a) this way they required less current

b) if one lamp fails the others also fail

c) if one lamp fails the others remain lit

d) this way they require less power

14 If the potential difference between the ends of a fixed resistor is halved, the
electric power will becomes:

a) half

b) double

c) one - fourth

d) four times
15 The hormone secreted by the thyroid gland is
1. Thyroxin
2. Calcitonin

3. Adrenaline

4. Insulin

a) B and C

b) A and B

c) A, B and D

d) All of these

16 Which of the following is secreted by the posterior lobe of Pituitary?


1. Growth hormone

2. Thyroid - stimulating hormone

3. Oxytocin

4. Vasopressin
a) B and C

b) A and C

c) C and D

d) All of these

17 Assertion (A): In a reaction,Zn(s) + CuSO 4 (aq) → ZnSO 4 (aq) +Cu(s)


Zn is a reductant but itself get oxidized

Reason (R): In a redox reaction, the oxidant is reduced by accepting electrons


and reductant is oxidized by losing electrons.

a) Both A and R are true and R is the correct explanation of A.

b) Both A and R are true but R is not the correct explanation of A.

c) A is true but R is false.

d) A is false but R is true.

18 Assertion (A): Phototropism is a directional growth movement.


Reason (R): It occurs in the direction of light.

a) Both A and R are true and R is the correct explanation of A.

b) Both A and R are true but R is not the correct explanation of A.


c) A is true but R is false.

d) A is false but R is true.

19 Assertion (A): The height of an object is always considered positive.


Reason (R): An object is always placed above the principal axis in the upward
direction.

a) Both A and R are true and R is the correct explanation of the assertion.

b) Both A and R are true but R is not the correct explanation of A.

c) A is true but R is false.

d) A is false but R is true.

20 Assertion (A): Plants have low energy needs.


Reason (R): Plant bodies have large proportion of dead cells.

a) Both A and R are true and R is the correct explanation of A.

b) Both A and R are true but R is not the correct explanation of A.

c) A is true but R is false.

d) A is false but R is true.

Section B 12 Marks

21 P, Q and R are 3 elements which undergoes chemical reactions according to the


following equations:

1. P2 O 3 + 2Q → Q 2 O 3 + 2P

2. 2RSO4 + 2Q → Q 2 (SO 4 ) 3 + 3R

3. 3RO + 2P→ P 2 O 3 + 3R

Answer the following:

1. Which element is more reactive?

2. Which element is least reactive?

3. State the type of reaction listed above.

22 Differentiate between Auricles and Ventricles


23 What is the function of lymphatic system?
OR

Differentiate between Excretion and Egestion.

24 A doctor has prescribed a corrective lens of power +1.5 D. Find the focal length of
the lens. Is the prescribed lens diverging or converging?
25 Differentiate between photonastic and thigmonastic movements giving an
example?

OR

The above figure represents certain phenomenon pertaining to the nervous

system.

1. What does the figure represents?

2. Name the parts labelled a, b, c, d and e.

3. Give the function of parts labelled a, b, d.

26 What is the need for a system of control and coordination in an organism?


Section C 21 Marks

27 Give the characteristic tests for the following gases


1. CO2

2. SO2

3. O2

4. H2

28 Write uses of
1. Acids

2. Bases

3. Salts
OR

1. Why are the chips packets puffed when we buy them from market?

2. Paint is applied on articles made up of iron, why?

29 If a plant is releasing carbon dioxide and taking in oxygen during the day, does it
mean that there is no photosynthesis occurring? Justify your Answer.
30 During respiration in an organism A, one molecule of glucose produces 2 ATP
molecules whereas in respiration of another organism B, one molecule of glucose
produces 38 ATP molecules.

1. Which organism is undergoing aerobic respiration?

2. Which organism is undergoing anaerobic respiration?

3. Which type of organism A or B can convert glucose into alcohol?

4. Name one organism which behaves like A.

5. Name one organism which behaves like B.

31 An object is kept at a distance of 18 cm, 20 cm and 30 cm, from a lens of


power+5D. (i) In which case or cases would you get a magnified image? (ii)
Which of the magnified image can we get on a screen? (b) List two widely used
applications of a convex lens.
32 1. An electric heater is rated at 2kW. Electrical energy costs₹ 4 per kWh.
What is the cost of using the heater for 3 hours?

2. You have two electric lamps having rating 40W, 220V and 60W, 220V.
Which of the two has a higher resistance? Give reason for your answer. If
these two lamps are connected to a source of 220V, which will glow
brighter?

33 What is the current through each of the resistances in the following circuit ?

Section D 15 Marks

34 1. What happens when an aqueous solution of sodium sulphate reacts with


an aqueous solution of barium chloride?

2. Write the balanced chemical equation for the reaction which takes place.

3. State the physical conditions of reactants in which the reaction will not
take place.
4. Name the type of chemical reaction which occurs.

5. Give one example of another reaction which is of the same type as the
above reaction.

OR

Equal length of magnesium ribbon are taken in two test tubes Aand B. H2 SO 4 is
added to test tube Aand H 2 CO 3 in the test tube B in equal amounts:

1. Identify the test tube showing vigorous reaction.

2. Give reason to support your answer.

3. Name the gas liberated in both the tubes. How will you prove its
liberation?

4. Write chemical equations for both reactions.

5. Out of the two acids taken above.

a. Which one will have lower pH value?


b. Which one will have lower H+ concentration?

35 Following are the two examples of plant movement. One is drooping of leaves in
touch - me - not plant and second is attaching of pea plant to a support with the
help of tendrils.

1. What is the stimulus which is common for movement in both the cases?

2. What is the difference in movement in both the plants? Explain.

3. Give appropriate scientific terms for the movements described in both


cases.

OR

Given below is a well - labelled diagram showing synapse between the two
neurons.
Using the given diagram, answer the following questions:

1. What is the sequence in which nerve impulse travels?

2. How synapse between two neurons acts as a one - way valve?

3. Which chemical substance is released when an electrical impulse coming


from the receptor reaches the end of the axon of a sensory neuron?

4. How a neurotransmitter starts an electrical impulse in the next neuron?

5. Which part of the neuron has a synaptic knob?

36 Under what condition in an arrangement of two plane mirrors, incident ray and
reflected ray will always be parallel to each other, whatever may be angle of
incidence. Show the same with the help of diagram.

OR

1. Define the following terms :

a. Power of a lens

b. Principal focus of a concave mirror

2. Write the relationship among the object distance (u), image distance (v)
and the focal length (f) of a

a. Spherical lens

b. Spherical mirror

3. An object is placed at a distance of 10 cm from optical centre of a convex


lens of focal length 15 cm. Draw a labelled ray diagram to show the
formation of image in this case.
12 Marks
Section E

37 Read the text carefully and answer the questions: A scale for measuring
hydronium ion in a solution is called the pH scale. The pH of a neutral solution is
7. A value of less than 7 on the pH scale represents an acidic solution. As the pH
value, increases from 7 to 14 it represents OH - ion concentration in solution i.e
a basic solution.

1. What is the pH range of the Human Body?

2. The strength of acid and bases depends on which factor?

3. What is the pH value of a neutral solution, and what does it signify?


OR

If the pH of soil X is 7.5 while that of soil Y is 4.5, then which soil should be
treated with powdered chalk to adjust its pH?

38 Read the text carefully and answer the questions: Animals have a nervous
system for controlling and coordinating the activities of the body. But plants have
neither a nervous system nor muscles. So, how do they respond to stimuli? When
we touch the leaves of a chui - mui (the ‘sensitive’ or ‘touch - me - not’ plant of
the Mimosa family), they begin to fold up and droop. When a seed germinates, the
root goes down, the stem comes up into the air. What happens? Firstly, the leaves
of the sensitive plant move very quickly in response to touch.

There is no growth involved in this movement. On the other hand, the directional
movement of a seedling is caused by growth. If it is prevented from growing, it
will not show any movement.

1. Write the types of movement.


2. Give an example of a plant hormone that promotes growth.

3. What is the function of the nervous system?

OR

How is the movement of leaves of the sensitive plant different from the
movement of a shoot towards light ?

39 Read the text carefully and answer the questions:


The relationship between the distance of the object from the lens (u), a distance
of the image from the lens (v) and the focal length (f) of the lens is called the lens
1 1 1
formula. It can be written as = − .
𝑓 𝑣 𝑢

The size of the image formed by a lens depends on the position of the object from
the lens. A lens of short focal length has more power whereas a lens of long focal
length has less power. When the lens is convex, the power is positive and for the
concave lens, the power is negative.

The magnification produced by a lens is the ratio of the height of the image to the
height of the object as the size of the image relative to the object is given by linear
magnification (m).

When m is negative, an image formed is real and when m is positive, an image


formed is virtual. If m < 1, the size of the image is smaller than the object. If m >1,
the size of the image is larger than the object.
1. An object with a height of 4 cm is positioned 10 cm away from a convex
lens with a focal length of 20 cm. Determine the position of the image
formed by the lens.

2. Given an object of height 4 cm positioned at a distance of 10 cm from a


convex lens with a focal length of 20 cm, what is the height or size of the
resulting image formed by the lens?

3. If a convex lens forms an image with a magnification of - 2 and the height


of the image is 6 cm, what is the height of the object?

OR

What is the power of a concave lens with a focal length of 5 cm?


Series SCI/10/11 Set ~ 11
Roll No. Q.P Code 10/11/11
Candidates must write the Q.P Code
on the title page of the answer-book.

 Please check that this question paper contains 11 printed pages.


 Q.P. Code given on the right hand side of the question paper should be written on the title
page of the answer-book by the candidate.
 Please check that this question paper contains 39 questions.
 Please write down the serial number of the question in the answer-book before
attempting it.
 15 Minute time has been allotted to read this question paper. The question paper will be
distributed at 10:15 a.m From 10.15a.m to 10.30 a.m, the students will read the question
paper only and will not write any answer on the answer –book during this period

SCIENCE
Time Allowed : 3 hours Maximum Marks : 80

General Instructions:

i. This question paper consists of 39 questions in 5 sections.


ii. All questions are compulsory. However, an internal choice is provided in some questions.
A student is expected to attempt only one of these questions.
iii. Section A consists of 20 objective type questions carrying 1 mark each.
iv. Section B consists of 6 Very Short questions carrying 02 marks each. Answers to these
questions should be in the range of 30 to 50 words.
v. Section C consists of 7 Short Answer type questions carrying 03 marks each. Answers to
these questions should be in the range of 50 to 80 words.
vi. Section D consists of 3 Long Answer type questions carrying 05 marks each. Answer to
these questions should be in the range of 80 to 120 words.
vii. Section E consists of 3 source - based/case - based units of assessment of 04 marks
each with sub - parts

[1]
Section A 20 Marks
1 Four test tubes containing solutions (I), (II), (III) and (IV) are shown below along
with their colours. Zinc sulphate is contained in

a) II

b) IV

c) I

d) III

2 Which of the following is an exothermic reaction?

a) Reactants× heat → Products

b) Reactants - heat→ Products

c) Reactants→ Products - heat

d) Reactants→ Products + heat

3 Which one of the following natural sources contains Oxalic acid?

a) Ant sting

b) Tomato

c) Tamarind

d) Nettle sting
4 Metal oxides generally react with acids, but few oxides of metal also react with
bases. Such metallic oxides are:

1. MgO

2. ZnO

3. Al2 O 3
4. CaO

a) I and IV

b) II and III

c) III and IV

d) I and II

5 Identify the method shown in the image to prevent rancidity:

a) Packaging fat and oil - containing foods in nitrogen gas

b) Adding anti - oxidants to foods

c) Storing food in air - tight containers

d) Keeping food in a refrigerator

6 During the preparation of hydrogen chloride gas on a humid day, the gas is
usually passed through the guard tube containing calcium chloride. The role of
calcium chloride taken in the guard tube is to

a) Absorb moisture from the gas

b) Absorb the evolved gas

c) Absorb Cl - ions from the evolved gas

d) Moisten the gas

7 Two colourless solutions X and Y were mixed together. On mixing, a yellow


precipitate Z was formed. Which of the following statements is correct regarding
X, Y and Z?

a) X and Y were potassium hydroxide solution and nitric acid respectively and
the yellow precipitate Z was potassium nitrate.

b) X and Y were sodium hydroxide solution and hydrochloric acid respectively


and the yellow precipitate Z was sodium chloride.

c) X and Y were potassium chloride solution and water respectively. The yellow
precipitate Z was of chloride ions.

d) X and Y were lead nitrate and potassium iodide solutions respectively. The
yellow precipitate Z was lead iodide.
8 Why is some KOH placed in a small test tube in the flask with germinating seeds
in the experiment to demonstrate occurrence of respiration in germinating
seeds?

a) To make the pair present in the flask alkaline.

b) To absorb carbon dioxide and create partial vacuum in the flask.

c) To absorb water from the seeds to make them dry.

d) To provide oxygen required by the seeds for respiration.

9 In the experiment to show that carbon dioxide is released during respiration the
small test tube of KOH solution is suspended inside the conical flask to absorb
the:

a) Oxygen of the flask.

b) Moisture of the flask.

c) Air of the flask.

d) Carbon dioxide of the flask released by the seeds.

10 The production of which hormone in adults leads to a gorilla - likeappearance


called acromegaly?

a) Thyroxine

b) Testosterone

c) Growth hormone

d) Adrenaline

11 In plants the role of cytokinin is:


a) Wilting of leaves.

b) Promote the opening of stomatal pore.

c) Help in the growth of stem.

d) Promote cell division.

12 After completing the experiment three students A, B and C found that the
percentage of absorption of water is 10%, 20% and 25% respectively. Who had
put raisins more time in the water?

a) C

b) A and C

c) B
d) A

13 Consider a thin square sheet of side x and thickness y made of a material of


resistivity𝜌 . The resistance between two opposite faces, shown by the shaded
areas in the figure is

a) Directly proportional to x
b) Independent of x

c) Independent of y

d) Directly proportional to y

14 An electric heater of resistance 8𝛺 takes a current of 15 A from the mains supply


line. The rate at which heat is developed in the heater:

a) 1500 J/s

b) 1700 J/s

c) 1800 J/s

d) 1600 J/s

15 The growth of pollen tubes towards ovules is due to


a) geotropism

b) chemotropism

c) phototropism

d) hydrotropism

16 The process by which blood is cleared of metabolic wastes in case of kidney


failure is called:

a) dialysis

b) filtration

c) transplantation

d) artificial kidney
17 Assertion (A): Pungent smelling gas is produced whensulphur burns in air.
Reason (R): Sulphur trioxide is formed on reaction of sulphur with oxygen.

a) Both A and R are true and R is the correct explanation of A.

b) Both A and R are true but R is not the correct explanation of A.

c) A is true but R is false.

d) A is false but R is true.

18 Assertion (A): Cytokinins are present in highest concentration in fruits and


seeds.
Reason (R): Cytokinins are responsible for promoting cell division.

a) Both A and R are true and R is the correct explanation of A.

b) Both A and R are true but R is not the correct explanation of A.

c) A is true but R is false.

d) A is false but R is true.

19 Assertion (A): If the rays are diverging after emerging from a lens; the lens must
be concave.

Reason (R): The convex lens can give diverging rays.

a) Both A and R are true and R is the correct explanation of A.

b) Both A and R are true but R is not the correct explanation of A.

c) A is true but R is false.

d) A is false but R is true.

20 Assertion (A): The effect of auxin hormone on the growth of root is exactly
opposite to that on a stem.

Reason (R): Auxin hormone increases the rate of growth in root and decreases
the rate of growth in stem.

a) Both A and R are true and R is the correct explanation of A.

b) Both A and R are true but R is not the correct explanation of A.

c) A is true but R is false.

d) A is false but R is true.


Section B 12 Marks

21 What are reduction reactions ? Give examples. [2]


22 What are the waste materials eliminated by our body? How are they generated? [2]
23 Design a pathway showing the correct sequence of air passage during inhalation. [2]
OR

Give reason why ammonia is converted to uric acid crystals directly in uricotelic
organisms?

24 Why does a ray of light bend when it travels from one medium into another? [2]
25 What are the functions of IAA? [2]

OR

Imagine what would happen if your neurons were disc - shaped?

26 What are practical uses of application of gibberellins?


Section C 21 Marks

27 A brown substance X on heating in air forms a substance Y. When hydrogen gas is [3]
passed over heated Y, it again changes back into X. Name the substances X and Y.
28 Explain the following giving equation in each: [3]
1. Baking soda is heated.

2. Washing soda is heated.

3. Gypsum is heated at 373 K.

OR

Does Tartaric acid helps in making cake or bread fluffy. Justify.

29 If a plant is releasing carbon dioxide and taking in oxygen during the day, does it [3]
mean that there is no photosynthesis occurring? Justify your Answer.
30 Nervous and hormonal system together performs the functions of control and [3]
coordination in human beings. Justify the statement.
31 Differentiate between virtual image formed by a concave mirror and of a convex [3]
mirror.
32 An electrician puts a fuse of rating 5 A in that part of domestic electrical circuit in [3]
which an electric heater of rating 1.5 kW, 220 V is operating. What is likely to
happen in this case and why?
33 Three students X, Y and Z while performing the experiment to study the [3]
dependence of current on the potential difference across a resistor, connect the
ammeter (A), the battery (B), the key (k) and the resistor (R) in series, in the
following three different orders.

1. X→ B, K, R, A, B

2. Y→ B, A, K, R, B

3. Z→ B, R, K, A, B

Who has connected them in the correct order?

Section D 15 Marks

34 How is plaster of Paris prepared ? Why is temperature control necessary during


its preparation ? How does it react with water ?

OR

What is meant by displacement reaction and double displacement reaction?


Explain with the help of one example each.

35 What is digestive gland? Name the various digestive glands of man and their
secretions.

OR

The image shows phototropism in the plant stem.

Using the given image, answer the following questions:

1. Which plant hormone is responsible for the bending of the stem towards
the light?

2. Where is this hormone present in the plant?

3. What makes the stem bend towards the light?

4. Why such movement of plant stem is called positive tropism?

5. Which part of the plant shows negative phototropism?


36 1. Name the type of mirror that should be used to obtain thefollowing types [5]
of images:

a. A magnified and virtual image of an object

b. A diminished and virtual image of an object

2. Draw labelled ray diagrams to justify your answers in each case


mentioned above.

3. Which of these mirrors can also be used to obtain a magnified and real
image of an object? State the position of the object in this case.

OR

A student wants to project the image of a candle flame on the walls of school
laboratory by using a lens.

1. Which type of lens should he use and why?


2. At what distance in terms of focal length F of the lens should he place the
candle flame, so as to get

a. a magnified and

b. a diminished image respectively, on the wall?

3. Draw ray diagrams to show the formation of the image in each case.

Section E 12 Marks
37 Read the text carefully and answer the questions: Copper sulphate crystal
contains water of crystallisation when the crystal is heated the water is removed
and salt turns white. The crystalcan be moistened again with water. The water of
crystallisation is the fixed number of water molecules present in 1 formula unit of
copper sulphate. On heating gypsum at 373K, it loses water molecules and
became calcium sulphate hemihydrate.
1. If the crystal is moistened with water, then which colour of the crystal
reappears?

2. What is the commercial name of calcium sulphate hemihydrate?

3. What happens to copper sulphate crystals when heated?

OR

How many water molecules arepresent in one formula unit of copper


sulphate?

38 Read the text carefully and answer the questions: Following questions are
based on the two tables given below. Study these tables related to blood sugar
levels:

Table A (Blood glucose chart)

Table B(Blood Report of Patient X and Y)


1. Refer to Table B showing the blood report of the levels of glucose of
patients X and Y. Infer the disease which can be diagnosed from the given
data.

2. Identify the hormone whose level in the blood is responsible for the above
disease.

3. High/low sugar and a low/high - fat diet What would you recommend to
the affected patient?

OR

Refer to Table A and suggest the value of the mean blood glucose level
beyond which doctor’s advice is necessary.

39 Read the text carefully and answer the questions: Study the following electric
circuit in which the resistors are arranged in three arms A , B and C:

1. Find the equivalent resistance of arm B.

2. Calculate the equivalent resistance of the parallel combination of the arms


B and C.

3. Determine the current that flows through the ammeter.

OR

Determine the current that flows in the ammeter when the arm B is
withdrawn from the circuit.
Series SCI/10/12 Set ~ 12
Roll No. Q.P Code 10/12/12
Candidates must write the Q.P Code
on the title page of the answer-book.

 Please check that this question paper contains 8 printed pages.


 Q.P. Code given on the right hand side of the question paper should be written on the title
page of the answer-book by the candidate.
 Please check that this question paper contains 39 questions.
 Please write down the serial number of the question in the answer-book before
attempting it.
 15 Minute time has been allotted to read this question paper. The question paper will be
distributed at 10:15 a.m From 10.15a.m to 10.30 a.m, the students will read the question
paper only and will not write any answer on the answer –book during this period

SCIENCE
Time Allowed : 3 hours Maximum Marks : 80

General Instructions:

i. This question paper consists of 39 questions in 5 sections.


ii. All questions are compulsory. However, an internal choice is provided in some questions.
A student is expected to attempt only one of these questions.
iii. Section A consists of 20 objective type questions carrying 1 mark each.
iv. Section B consists of 6 Very Short questions carrying 02 marks each. Answers to these
questions should be in the range of 30 to 50 words.
v. Section C consists of 7 Short Answer type questions carrying 03 marks each. Answers to
these questions should be in the range of 50 to 80 words.
vi. Section D consists of 3 Long Answer type questions carrying 05 marks each. Answer to
these questions should be in the range of 80 to 120 words.
vii. Section E consists of 3 source - based/case - based units of assessment of 04 marks
each with sub - parts

[1]
Section A 20 Marks

1. A student took Cu, Al, Fe and Zn strips separately in four test tubes labelled I, II, III and IV. He added 10 mL of
freshly prepared ferrous sulphate solution to each test tube as shown below:

Black residue would be obtained in test tubes

a) II and IV b) I, II and IV

c) II and III d) III and IV


2. Study the given experimental set-up.

If in test tubes I and III, black residue was obtained and in test tubes II and IV, no change was observed, then
metals P, Q, R and S could be respectively

a) Al, Cu, Pb, Ag b) Pb, Cu, Ag, Al

c) Zn, Al, Cu, Ag d) Zn, Cu, Al, Ag


3. If the pH of a solution is 13, it means that it is

a) Weakly acidic b) Strongly Basic

c) Strongly acidic d) Weakly basic


4. A chemical reaction is characterised by:

a) Change in state b) Evolution and absorption of energy

c) Formation of new products d) All of these


5. A student added dilute HCl to Zn granules taken in a test tube. The correct observation would be
a) evolution of gas b) no change

c) Zn granules turned green d) formation of a precipitate


6. Which of the following will allow the electricity to flow through it/them?
i. Glucose
ii. Dry HCl gas
iii. Alcohol
iv. Dil. H2SO4
c) soaked in vinegar d) dried completely
a) (iv) only b) (ii), (iii) and (iv)

c) (ii) and (iv) only d) (i) and (iii) only


7. The drying agent used for ammonia gas is

a) P2O5 b) Slaked lime

c) Quick lime d) Conc. H2SO4

8. Study the given experimental set-up.

By measuring the movement of the oil drop in the apparatus, what can be investigated?

a) Heat is released during germination b) Water is produced during germination

c) Oxygen is used during germination d) Carbon dioxide is released during


germination
9. Which hormone causes uterine contraction that leads to labour pain?

a) Vasopressin b) Thyroxin

c) Estrogen d) Progesterone
10. Which part of the brain contains respiratory centre?

a) Mid brain b) Cerebrum

c) Pons d) Medulla oblongata


11. Before setting up an experiment to show that seeds release carbon dioxide during respiration, the seeds should
be

a) boiled to make them soft b) kept moist till they germinate


c) soaked in vinegar d) dried completely
12. The opening and closing of the stomatal pore depends upon

a) Oxygen b) Water in guard cells

c) Concentration of CO2 in stomatal d) Temperature

13. A potential difference of 20V is applied across the ends of a resistance of 5 ohms. What current will flow in the
resistance?

a) 20A b) 3A

c) 5A d) 4A
14. The resistivity of certain material is 0.6 Ω m. The material is most likely to be:

a) A conductor b) A semiconductor

c) A superconductor d) An insulator
15. Which one of the following is involved in reflex action?

a) Endocrine gland b) Exocrine glad

c) spinal cord d) Brain


16. The process complementary to respiration is ________.

a) Transpiration b) Circulation

c) Photosynthesis d) Osmoregulation
17. Assertion (A): Corrosion of iron is commonly known as rusting.
Reason (R): Corrosion of iron occurs in presence of moist air.

a) Both A and R are true and R is the correct b) Both A and R are true but R is not the
explanation of A. correct explanation of A.

c) A is true but R is false. d) A is false but R is true.


18. Assertion (A): Antheorozoids of Funaria show chemotropic movement.
Reason (R): This is a paratonic movement of locomotion.

a) Both A and R are true and R is the correct b) Both A and R are true but R is not the
explanation of A. correct explanation of A.

c) A is true but R is false. d) A is false but R is true.


19. Assertion (A): Light travels faster in air than in glass.
Reason (R): Air is denser than glass.

a) Both A and R are true and R is correct b) Both A and R are true but R is not the
explanation of the assertion. correct explanation of the assertion.

c) A is false but R is true. d) A is true but R is false.

20. Assertion (A): In a chain of bulbs, 50 bulbs are joined in series. One bulb is removed now and circuit is
completed again. If the remaining 49 bulbs are again connected in series across the same supply, then light gets
decreased in the room.
Reason (R): Net resistance of 49 bulbs will be less than 50 bulbs.
a) Both A and R are true and R is the correct b) Both A and R are true but R is not the
explanation of A. correct explanation of A.

c) A is true but R is false. d) A is false but R is true.


Section B 12 Marks
21. State reason for the following.
i. Small amount of acid is added to water during electrolysis of water.
ii. When ammonium chloride is dissolved in water in a test tube, The test tube becomes cold.
iii. Paint is applied on iron articles.
22. Why should we keep the germinating seeds moist while carrying out experiment on respiration?
23. In the human body the site of absorption of digested food is the small intestine. How is the process of absorption
carried out and why is absorption of digested food necessary?
OR
Name any three glands associated with digestion in humans. Write the name of enzymes secreted by them?
24. An object of size 7.0 cm is placed 27 cm in front of concave mirror of focal length 18 cm. At what distance
should the screen be placed so that a sharp focused image can be obtained ? Find the size and nature of the
image.
25. What is autonomic nervous system? Which are the divisions of autonomic nervous system?
OR
Create a flow chart of neuron through which information travels as an electrical impulse by filling up a and b.
Dendrite ⟶ a ⟶ b ⟶ end point of neuron.
26. How does phototropism occur in plants?
21 Marks
Section C
27. Identify the type of reactions taking place in each of the following cases and write the balanced chemical
equation for the reactions.
a. Zinc reacts with silver nitrate to produce zinc nitrate and silver.
b. Potassium iodide reacts with lead nitrate to produce potassium nitrate and lead iodide.
28. A dry pellet of a common base B, when kept in open absorbs moisture and turns sticky. The compound is also a
by-product of chlor-alkali process. Identify B. What type of reaction occurs when B is treated with an acidic
oxide? Write a balanced chemical equation for one such solution.
OR
Write the balanced chemical equations for the following reactions and identify the type of reaction in each case.
i. In thermite reaction, iron (III) oxide reacts with aluminium and gives molten iron and aluminium oxide.
ii. Magnesium ribbon is burnt in an atmosphere of nitrogen gas to form solid magnesium nitride.

29. a. Draw a diagram of human alimentary canal and label - gall bladder, pancreas, liver and small intestine on it.
b. Give two reasons to explain why absorption of digested food occurs mainly in the small intestine.

30. Nervous and hormonal system together performs the functions of control and coordination in human beings.
Justify the statement.
31. One-half of a convex lens is covered with a black paper. Will this lens produce a complete image of the object?
Verify your answer experimentally. Explain your observations.
32. How can three resistors of resistance 2Ω, 3Ω and 6Ω be connected to give a total resistance of (a) 4Ω; (b) 1Ω? [3]
33. The length of different metallic wires, but of the same area of cross-section and made of the same material are
given below:
Wire - Length
A-1m
B - 1.5 m
C - 2.0 m
(i) Out of these three wires, which wire has higher resistance?
(ii) Which wire has higher electrical resistivity? Justify your answer.
Section D 15 Marks
34. i. What is salt? Give the names and formulae of any two salts. Also, name the acids and bases from which
these salts may be obtained.
ii. What is meant by a family of salts? Explain with examples.
iii. What is meant by hydrated and anhydrous salts? Explain with examples.
iv. Write the names, formulae, and colours of any two hydrated salts.
v. What will be the colour of litmus in an aqueous solution of ammonium chloride salt?
OR
When the solution of substance A is added to a solution of potassium iodide, then a yellow solid separates out from
the solution.
i. What do you think substance A would be?
ii. Name the yellow substance which separates out.
iii. Write the characteristic of chemical reactions which is illustrated by this example.
iv. Write a balanced chemical equation for the reaction which takes place. Mention the physical states of all the
reactants and products involved in the chemical equation.
35. Given below is a well-labelled diagram of the human respiratory system.

Using the above diagram, answer the following questions:


i. Why nasal hair and mucus are present in the nasal chamber?
ii. Why trachea does not collapse even when there is no air?
iii. Where the exchange of respiratory gases take place?
iv. What happens when we breathe in?
v. Why the lungs always contain a residual volume of air?
OR
i. List three points of difference between nervous and hormonal mechanisms for control and coordination in
animals.
ii. How are auxins related with the bending of plant shoot towards unidirectional light? Explain.

36. a. List four characteristics of the image formed by a concave lens of focal length 20 cm when the object is
placed at a distance of 40 cm from its optical centre.
b. The size of image of an object by a convex lens of focal length 20 cm is observed to be reduced to 1 rd of its
3

size. Find the distance of the object from the optical centre of the lens.
OR
A student places a candle flame at a distance of about 60 cm from a convex lens of focal length 10 cm and focuses
the image of the flame on a screen. After that he gradually moves the flame towards the lens and each time focuses
the image on the screen.
i. In which direction: toward or away from the lens, does he move the screen to focus the image?
ii. How does the size of the image change?
iii. How does the intensity of the image change as the flame moves towards the lens?
iv. Approximately for what distance between the flame and the lens, the image formed on the screen is inverted and
of the same size?
Section E 12 Marks
37. Read the text carefully and answer the questions:
The teacher while conducting practicals in the laboratory divided the students into three groups and gave them
various solutions to find out their pH and classify them into acidic, basic and neutral solutions.
Group A - Lemon juice, vinegar, colourless aerated drink
Group B - Tomato juice, coffee, ginger juice
Group C - Sodium hydroxide, sodium chloride, lime water
(a) For the solutions provided, which group is/are likely to have pH value (i) less than 7, and (ii) greater than
7?
(b) List two ways of determining pH of a solution.
(c) Explain, why the sour substances such as lemon juice are effective in cleaning the tarnished copper
vessels.
OR
pH has great importance in our daily life. Justify this statement by giving two examples.
38. Read the text carefully and answer the questions:
You must have noticed many dramatic changes in your appearance as well as that of your friends as you
approached 10-12 years of age. These changes associated with puberty are because of the secretion of
testosterone in males and oestrogen in females. Do you know anyone in your family or friends who has been
advised by the doctor to take less sugar in their diet because they are suffering from diabetes? As a treatment,
they might be taking injections of insulin. This is a hormone that is produced by the pancreas.
(a) Why is pancreas a dual gland?
(b) Name the hormone which is secreted by males and females during adolescence.
(c) What happens if Insulin is not secreted in the proper amount?
OR
From which cells of pancreatic islets insulin and glucagon hormone are secreted?

39. Read the text carefully and answer the questions:


In 1827, a German physicist Georg Simon Ohm (1787-1854) found out the relationship between the current I,
flowing in metallic wire and the potential difference across its terminals. He stated that the electric current
flowing through a metallic wire is directly proportional to the potential difference V, across its ends provided its
temperature remains the same.
The resistance of a circuit is defined as the ratio between the voltage applied to the current flowing through it.
Rearranging the above relation,
V
R =
I

Electric charge flows easily through some materials than others. The electrical resistance measures how much
the flow of this electric charge is restricted within the circuit.

(a) What is the unit of electrical resistance?


(b) Define Ohm's law.
(c) From graph which resistance have high resistance?
OR
What does the slope of V-I graph at any point represent?
Series SCI/10/13 Set ~ 13
Roll No. Q.P Code 10/13/13
Candidates must write the Q.P Code
on the title page of the answer-book.

 Please check that this question paper contains 8 printed pages.


 Q.P. Code given on the right hand side of the question paper should be written on the title
page of the answer-book by the candidate.
 Please check that this question paper contains 39 questions.
 Please write down the serial number of the question in the answer-book before
attempting it.
 15 Minute time has been allotted to read this question paper. The question paper will be
distributed at 10:15 a.m From 10.15a.m to 10.30 a.m, the students will read the question
paper only and will not write any answer on the answer –book during this period

SCIENCE
Time Allowed : 3 hours Maximum Marks : 80

General Instructions:

i. This question paper consists of 39 questions in 5 sections.


ii. All questions are compulsory. However, an internal choice is provided in some questions.
A student is expected to attempt only one of these questions.
iii. Section A consists of 20 objective type questions carrying 1 mark each.
iv. Section B consists of 6 Very Short questions carrying 02 marks each. Answers to these
questions should be in the range of 30 to 50 words.
v. Section C consists of 7 Short Answer type questions carrying 03 marks each. Answers to
these questions should be in the range of 50 to 80 words.
vi. Section D consists of 3 Long Answer type questions carrying 05 marks each. Answer to
these questions should be in the range of 80 to 120 words.
vii. Section E consists of 3 source - based/case - based units of assessment of 04 marks
each with sub - parts

[1]
Section A 20 Marks
1. Zinc granules are placed in each of the four solutions A, B, C and D as shown below decolourisation would be
observed in

a) [B] and [D] b) [A] and [C]

c) [C] and [D] d) [A] and [B]


2. The reaction between calcium oxide and water is:

a) Displacement reaction b) Decomposition reaction

c) Combination reaction d) Double -decomposition reaction

3. The H+ ion concentration of a solution is 1.0 × 10-5 m. The solution is

a) Acidic b) Neutral

c) Alkaline d) Amphoteric
4. Soda-lime is a mixture of NaOH and CaO. What is the ratio of NaOH and CaO in soda-lime?

a) 0.0854166666667 b) 0.125694444444

c) 0.04375 d) 0.126388888889
5. A student performed an experiment using zinc granules and sodium carbonate with sodium hydroxide and
hydrochloric acid under different conditions as shown here.

He would observe that no gas is evolved in the set up :

a) II b) III

c) I d) IV
6. The name given to the expression of a reaction in terms of formulae of reactants and products is

a) Chemical reaction b) Redox reaction

c) Chemical equation d) Chemical composition


7. In which of the following, the identity of initial substance remains unchanged?

a) Formation of crystals by process of b) Curdling of milk


crystallisation

c) Fermentation of grapes d) Digestion of food


8. The product of fermentation is ________.

a) Methanol b) Formic acid

c) Citric acid d) Ethanol


9. Which plant hormone is essential for the cell elongation?

a) Cytokinin b) Auxin

c) Gibberellins d) Ethylene
10. When the cell is kept in a hypertonic solution, then water moves

a) Into the cell b) Out of the cell

c) movement of water d) No movement of water


11. The part in which gustatory receptors are present in our body is:

a) skin b) tongue

c) inner lining of nose d) inner ear


12. While preparing a temporary stained mount of a leaf epidermal peel, the extra stain is removed by

a) soaking with filter paper b) absorbing with cotton wool

c) washing with water d) washing with calcium chloride solution


13. In torch lights and head lights of vehicles, the bulb is placed

a) between the pole and the focus of the b) very near to the focus of the reflector.
reflector.

c) at the centre of curvature of the reflector. d) between the focus and centre of curvature of
the reflector.
14. A material used for making heating elements of electrical heating devices should have:

a) Low resistivity and low melting point. b) Low resistivity and high melting point.

c) High resistivity and low melting point. d) High resistivity and high melting point.
15. In an airtight experimental set-up which was used by you in the laboratory to study respiration in germinating
seeds, the seeds obtained the oxygen for respiration from

a) water in the germinating seeds b) water in the beaker

c) water used for soaking the seeds d) air in the flask


16. Which part of the brain is concerned with muscular coordination in the body?

a) Temporal lobe b) Cerebellum

c) Pons d) Parietal lobe


17. Assertion (A): Brown fumes are produced when lead nitrate is heated.
Reason (R): Nitrogen dioxide gas is produced as a byproduct due to the decomposition of lead nitrate.

a) Both A and R are true and R is the correct b) Both A and R are true but R is not the
explanation of A. correct explanation of A.

c) A is true but R is false. d) A is false but R is true.


18. Assertion (A): The large intestine is the largest part of the alimentary canal.
Reason (R): Tiger has a shorter small intestine than herbivores.

a) Both A and R are true and R is the correct b) Both A and R are true but R is not the
explanation of A. correct explanation of A.

c) A is true but R is false. d) A is false but R is true.


19. Assertion (A): Property of converging of a convergent lens does not remain same in all media.
Reason (R): Property of lens whether the ray is diverging or converging is independent of the surrounding
medium.

a) Both A and R are true and R is the correct b) Both A and R are true but R is not the
explanation of A. correct explanation of A.

c) A is true but R is false. d) A is false but R is true.


20. Assertion (A): Animals can react to stimuli in different ways.
Reason (R): All animals have a nervous system and an endocrine system involving hormones.

a) Both A and R are true and R is the correct b) Both A and R are true but R is not the
explanation of A. correct explanation of A.

c) A is true but R is false. d) A is false but R is true.


Section B 12 Marks
21. What is the difference between combination and decomposition reactions? Write an equation of each type.
22. i. Define olfactory indicators. Name two substances which can be used as olfactory indicators.
ii. Choose strong acids from the following: CH3COOH, H2SO4, H2CO3, HNO3

23. How the skin helps in excretion?


OR
Describe double circulation of blood in human beings. Why is it necessary?
24. A truck uses a convex mirror as view finder whose radius of curvature is 2.0 m. A maruti car is coming behind
the truck at a distance of 10 m. What will be the position of the image of the car and size of the image of the car
when observed by the driver of the truck through the convex mirror?
25. What happens to glucose which enters the nephron along with the filtrate?
OR
Name the hormones which stimulate the testis.
26. Define tropism? What are different types of tropism?
Section C 21 Marks
27. Translate the following statement into a chemical equation and then balance it :
Barium chloride reacts with aluminium sulphate to give aluminium chloride and a precipitate of barium
sulphate.
28. Justify HNO3 and KOH are respectively strong and weak acid and base.
OR
A compound X on heating with excess conc. sulphuric acid at 443 K gives an unsaturated compound Y. X also reacts
with sodium metal to evolve a colourless gas Z. Identify X, Y and Z. Write the equation of the chemical reaction of
formation of Y and also write the role of sulphuric acid in the reaction.
29. a. Why is there a difference in the rate of breathing between aquatic organisms and terrestrial organisms?
Explain.
b. Draw a diagram of human respiratory system and label - pharynx, trachea, lungs, diaphragm and alveolar sac
on it.
30. Nervous and hormonal system together performs the functions of control and coordination in human beings.
Justify the statement.
31. Sudha finds out that the sharp image of window pane of her science laboratory is formed at a distance of 15 cm
from the lens. She now tries to focus the building visible of her outside the window instead of the window pane
without disturbing the lens. In which direction will she move the screen to obtain a sharp image of the building?
What is the approximate focal length of this lens?
32. i. Write the formula for determining the equivalent resistance between A and B of the two combinations (I) and
(II) of three resistors R1, R2 and R3 arranged as follows:

ii. If the equivalent resistance of the arrangements (I) and (II) are Rs and Rp respectively, then which one of the
following VI graphs is correctly labelled? Justify your answer.
33. How many 176Ω resistors (in parallel) are required to carry 5A on a 220V line?
Section D
34. Compound 'A', when dissolved in water, gives compound 'B' and liberates heat. Compound 'A' is used in
whitewashing. Compound 'B' reacts with C O to form a white precipitate of compound 'C'. Identify compounds
2

'A', 'B' and 'C' Also write the equations involved.


OR
In the following schematic diagram for the preparation of hydrogen gas as shown in the figure, what would happen if
the following changes are made?

i. In place of zinc granules, same amount of zinc dust in taken in the test tube.
ii. Instead of dilute sulphuric acid, dilute hydrochloric acid is taken.
iii. In place of zinc, copper turnings are taken.
iv. Sodium hydroxide is taken in place of dilute sulphuric acid and the test tube is heated.
35. Explain the location, structure and functions of kidney in human beings. Also draw an appropriate diagram
supporting your answer.
OR
Give an account of any seven diseases caused by the excess and lesser secretions of various endocrine glands.
36. Find the size, nature and position of image formed when an object of size 1 is placed at a distance of 15 cm from
a concave mirror of focal length 10 cm.
OR
i. One half of a convex lens of focal length 10 cm is covered with a black paper. Can such a lens produce an image
of a complete object placed at a distance of 30 cm from the lens? Draw a ray diagram to justify your answer.
ii. A 4 cm tall object is placed perpendicular to principal axis of a convex lens of focal length 20 cm. The distance of
the object from the lens is 15 cm. Find the nature, position and the size of the image.
Section E 12 Marks
37. Read the text carefully and answer the questions:
Salt of a strong acid and strong base is neutral with a pH value of 7. NaCl common salt is formed by a
combination of hydrochloride and sodium hydroxide solution. This is the salt that is used in food. Some salt is
called rock salt, bed of rock salt was formed when seas of bygone ages dried up. The common salt thus obtained
is an important raw material for various materials of daily use, such as sodium hydroxide, baking soda, washing
soda, and bleaching powder.
(a) If given acids are phosphoric acid, carbonic acid, hydrochloric acid and sulphuric acid, then which acid
does not form an acidic salt?
(b) What is the formula of baking soda?
(c) Name the substance which on treatment with chlorine to obtain bleaching powder.
OR
Which salt is used for removing the permanent hardness of water?
38. Read the text carefully and answer the questions:
In animals, control and coordination are provided by nervous and muscular tissues. Touching a hot object is an
urgent and dangerous situation for us. We need to detect it and respond to it. How do we detect that we are
touching a hot object? All information from our environment is detected by the specialised tips of some nerve
cells. These receptors are usually located in our sense organs, such as the inner ear, the nose, the tongue, and so
on. So gustatory receptors will detect taste while olfactory receptors will detect the smell. This information,
acquired at the end of the dendritic tip of a nerve cell sets off a chemical reaction that creates an electrical
impulse. This impulse travels from the dendrite to the cell body, and then along the axon to its end. At the end of
the axon, the electrical impulse sets off the release of some chemicals. These chemicals cross the gap, or
synapse, and start a similar electrical impulse in the dendrite of the next neuron. This is a general scheme of how
nervous impulses travel in the body. A similar synapse finally allows the delivery of such impulses from neurons
to other cells, such as muscles cells or glands.

(a) Why does the flow of signals in a synapse from axonal end of one neuron to dendritic end of another
neuron take place but not in the reverse direction?
(b) From where the electrical impulse travels?
(c) Name the chemical which released at the end of axon to transmit the signal to the other neuron.
OR
What happens at the synapse between 2 neurons?
39. Read the text carefully and answer the questions:
The ability of a medium to refract light is expressed in terms of its optical density. Optical density has a definite
connotation. It is not the same as mass density. On comparing two media, the one with the large refractive
index is optically denser medium than the other. The other medium with a lower refractive index is optically
rarer. Also the speed of light through a given medium is inversely proportional to its optical density.
(a) Determine the speed of light in diamond if the refractive index of diamond with respect to vacuum is 2.42.

Speed of light in vacuum is 3 × 108 m/s.


(b) Refractive indices of glass, water and carbon disulphide are 1.5, 1.33 and 1.62 respectively. If a ray of
light is incident in these media at the same angle (say θ ), then write the increasing order of the angle of
refraction in these media.
(c) The speed of light in glass is 2 × 108 m/s and in water is 2.25 × 108 m/s.
a. Which one of the two is optically denser and why?
b. A ray of light is incident normally at the water-glass interface when it enters a thick glass container
filled with water. What will happen to the path of the ray after entering the glass? Give reason.
OR
The absolute refractive indices of water and glass are and respectively. If the speed of light in glass is
4 3

3 2

2 × 108 m/s, find the speed of light in (i) vacuum and (ii) water.
Series SCI/10/14 Set ~ 14
Roll No. Q.P Code 10/14/14
Candidates must write the Q.P Code
on the title page of the answer-book.

 Please check that this question paper contains 8 printed pages.


 Q.P. Code given on the right hand side of the question paper should be written on the title
page of the answer-book by the candidate.
 Please check that this question paper contains 39 questions.
 Please write down the serial number of the question in the answer-book before
attempting it.
 15 Minute time has been allotted to read this question paper. The question paper will be
distributed at 10:15 a.m From 10.15a.m to 10.30 a.m, the students will read the question
paper only and will not write any answer on the answer –book during this period

SCIENCE
Time Allowed : 3 hours Maximum Marks : 80

General Instructions:

i. This question paper consists of 39 questions in 5 sections.


ii. All questions are compulsory. However, an internal choice is provided in some questions.
A student is expected to attempt only one of these questions.
iii. Section A consists of 20 objective type questions carrying 1 mark each.
iv. Section B consists of 6 Very Short questions carrying 02 marks each. Answers to these
questions should be in the range of 30 to 50 words.
v. Section C consists of 7 Short Answer type questions carrying 03 marks each. Answers to
these questions should be in the range of 50 to 80 words.
vi. Section D consists of 3 Long Answer type questions carrying 05 marks each. Answer to
these questions should be in the range of 80 to 120 words.
vii. Section E consists of 3 source - based/case - based units of assessment of 04 marks
each with sub - parts

[1]
Section A 20 Marks
1. The copper articles turn green when kept for long due to

a) Corrosion b) Rusting

c) Precipitation d) Rancidity
2. What happens when dilute HCl is added to iron fillings? Select the correct answer.

a) Hydrogen gas and iron chloride are b) Iron salt and water are produced
produced

c) No reaction takes place d) chlorine gas and iron hydroxide are


produced
3. When steam is passed over red hot iron

a) No reaction takes place. b) H2 and Fe3O4 are formed.

c) H2 and Fe2O3 are formed. d) O2 and Fe(OH)3 are formed.

4. Calcium phosphate is present in tooth enamel. Its nature is

a) Acidic b) Neutral

c) Amphoteric d) Basic
5. On adding zinc granules to freshly prepared ferrous sulphate solution, a student observes that

a) a dull brown coating is formed b) a greyish black coating is formed

c) a white coating is formed d) no coating is formed

6. HCl is a strong acid since in solution it gives

a) more number of water molecules b) less number of H+ ions

c) more number of Cl– ions d) more number of H+ ions


7. In the reaction CuO + H2 → Cu + H2O, identify the oxidizing and reducing agent.

a) Cu, H2O b) CuO, H2

c) H2O, C d) H2O, Cu

8. When air is blown from mouth into a test - tube containing lime water, the lime water turned milky due to
presence of-

a) water vapours b) nitrogen

c) oxygen d) carbon dioxide


9. Spinal cord originates from:

a) cerebrum b) pons

c) cerebellum d) medulla
10. The given figure is a demonstration of an experiment to show that carbon dioxide is essential for photosynthesis.
What is the substance X, kept in watch-glass?

a) Sodium bicarbonate b) Sodium carbonate

c) Potassium sulphate d) Potassium hydroxide


11. Which hormone deficiency causes dwarfism?

a) Oxytocin b) Growth hormone


c) Thyroxin d) Vasopressin hormone
12. Opening and closing of stomata is due to

a) Movement of water in and out of the guard b) Stimulus of light in the guard cells.
cells.

c) Diffusion of CO2 in and out of the guard d) High pressure of gases inside the cells.
cells.
13. The image formed by a plane mirror is:

a) real, behind the mirror, and of the same size b) virtual, behind the mirror, and of the same
as the object size as the object

c) virtual, behind the mirror, and enlarged d) real, at the surface of the mirror, and
enlarged
14. If the area of cross-section of a resistance wire is halved, then its resistance becomes:

a) 4 times b) One-half
c) 2 times d) One-fourth
15. When a ray of light from rarer to denser it will:

a) Return into the same medium b) Bend away from the normal

c) Pass straight d) Bends towards the normal


16. The final product of glycolsis is

a) Glucose b) Acetyl COA

c) Malic acid d) Pyruvic acid


17. Assertion (A): Food materials are often packed in air tight container.
Reason (R): Oxidation, resulting in rancidity, is prevented.

a) Both A and R are true and R is the correct b) Both A and R are true but R is not the
explanation of A. correct explanation of A.

c) A is true but R is false. d) A is false but R is true.


18. Assertion (A): Auxins are in the growing tips of the plant.
Reason (R): Auxin concentration is highest at the tip of the root.

a) Both A and R are true and R is the correct b) Both A and R are true but R is not the
explanation of A. correct explanation of A.

c) A is true but R is false. d) A is false but R is true.


19. Assertion (A): At high temperatures, metal wires have a greater chance of short-circuiting.
Reason (R): Both resistance and resistivity of a material vary with temperature.

a) Both A and R are true and R is the correct b) Both A and R are true but R is not the
explanation of A. correct explanation of A.

c) A is true but R is false. d) A is false but R is true.


20. Assertion (A): The inner walls of the small intestine have finger like projections called villi which are rich in
blood.

Reason (R): These villi have a large surface area to help the small intestine in completing the digestion of food.

a) Both A and R are true and R is the correct b) Both A and R are true but R is not the
explanation of A. correct explanation of A.

c) A is true but R is false. d) A is false but R is true.

12 Marks
Section B
21. With the help of suitable chemical equations, list the two main differences between roasting and calcination.
How is metal reduced from the product obtained after roasting/calcination of the ore? Write the chemical
equation for the reaction involved.

22. A white powder is added while baking cakes to make it soft and spongy. Name its main ingredients. Explain the
function of each ingredient. Write the chemical reaction taking place when the powder is heated during baking.
23. How does respiration take place in plants?
OR
Compare the functioning of alveoli in the lungs and nephrons in the kidneys with respect to their structure and
functioning.
24. For which position of the object does a convex lens form a virtual and erect image?
25. Give functions of blood.
OR
Write a short note on Thymus gland.
26. Give reasons:
i. Ventricles have thicker muscular walls than atria.
ii. Transport system in plants is slow.
iii. Circulation of blood in aquatic vertebrates differs from that in terrestrial vertebrates.
iv. During the daytime, water and minerals travel faster through xylem as compared to the night.
v. Veins have valves whereas arteries do not.
Section C 21 Marks
27. A compound X on heating with excess conc. sulphuric acid at 443 K gives an unsaturated compound Y. X also
reacts with sodium metal to evolve a colourless gas Z. Identify X, Y and Z. Write the equation of the chemical
reaction of formation of Y and also write the role of sulphuric acid in the reaction.
28. A white shirt has a yellow stain of curry. When soap is rubbed on this shirt during washing, the yellow stain
turns reddish-brown. On rinsing the shirt with plenty of water, the reddish-brown stain turns yellow again.
i. Name the natural indicator present in curry stain.
ii. Explain the changes in colour of this indicator which take place during washing and rinsing the shirt.
iii. What is the nature of soap (acidic/basic) as shown by the indicator present in curry stain?
OR
Aluminium is a reactive metal but is still used for packing food articles. Why?
29. i. Why do the herbivores need longer small intestine as compared to that of the carnivores?
ii. List three types of substances secreted by the gastric glands and state the role of each in the digestion of food
in alimentary canal of human beings.
30. Nervous and hormonal system together performs the functions of control and coordination in human beings.
Justify the statement.
31. Distinguish between real image and virtual image.

32. a. How much current will an electric bulb draw from a 220 V source, if the resistance of the bulb filament is
1200 Ω ?
b. How much current will an electric heater coil draw from a 220 V source, if the resistance of the heater coil is
100 Ω ?

33. The values of current I flowing through a resistor for the corresponding values of potential difference V across it
are given below:

V(volts) 1.5 3.0 6.0 9.0

I(amperes) 0.5 1.0 2.0 3.0


a. Plot a graph between V and I.
b. Why should this graph pass through the origin?
c. Name and state the law which is represented by the graph.
Section D 15 Marks
34. What happens when zinc granules are treated with a dilute solution of H2SO4, HCl, HNO3, NaCl, and NaOH,
also write the chemical equations if a reaction occurs.
OR
i. What is meant by pH?
ii. Water is a neutral substance. What colour will you get when you add a few drops of universal indicator to a test
tube containing distilled water?
iii. Two solutions A and B have pH values of 3.0 and 9.5, respectively. Which of these will turn litmus solution from
blue to red and which will turn phenolphthalein from colourless to pink?
35. What is nutrition? Briefly explain the two major kinds of nutrition.
OR
Draw a labeled diagram of neuron and explain its function.
36. i. Two lenses have power of
a. + 2 D
b. - 4 D.
What is the nature and focal length of each lens?
ii. An object is kept at a distance of 100 cm from each of above lenses. Calculate
a. image distance and
b. magnification in each of the two cases.
OR
An object 1 m tall is placed on the principal axis of a convex lens and its 40 cm tall image is formed on the screen
placed at a distance of 70 cm from the object. What is the focal length of the lens?
Section E 12 Marks
37. Read the text carefully and answer the questions:
A scale for measuring hydronium ion in a solution is called the pH scale. The pH of a neutral solution is 7. A
value of less than 7 on the pH scale represents an acidic solution. As the pH value, increases from 7 to 14 it
represents OH- ion concentration in solution i.e a basic solution.

(a) What is the pH range of the Human Body?


(b) The strength of acid and bases depends on which factor?
(c) If the pH of soil X is 7.5 while that of soil Y is 4.5, then which soil should be treated with powdered chalk
to adjust its pH?
OR
Tooth decay starts when the pH of the mouth is lower than which pH?
38. Read the text carefully and answer the questions:
Animals have a nervous system for controlling and coordinating the activities of the body. But plants have
neither a nervous system nor muscles. So, how do they respond to stimuli? When we touch the leaves of a chhui-
mui (the ‘sensitive’ or ‘touch-me-not’ plant of the Mimosa family), they begin to fold up and droop. When a
seed germinates, the root goes down, the stem comes up into the air. What happens? Firstly, the leaves of the
sensitive plant move very quickly in response to touch. There is no growth involved in this movement. On the
other hand, the directional movement of a seedling is caused by growth. If it is prevented from growing, it will
not show any movement. So plants show two different types of movement - one dependent on growth and the
other independent of growth.

(a) Plants neither have nervous system nor muscles, then how does chemical coordination occur in plants?
(b) Why Mimossa pudica leaves drop down when we touched? Write its another name also.
(c) What is turgor movement?
OR
What is a tropic movement? Explain with an example
39. Read the text carefully and answer the questions:
Many optical instruments consist of a number of lenses. They are combined to increase the magnification and
sharpness of the image. The net power (P) of the lenses placed in contact is given by the algebraic sum of the
powers of the individual lenses P1, P2, P3 ... as
P = P1 + P2 + P3 ...
This is also termed as the simple additive property of the power of lens, widely used to design lens systems of
cameras, microscopes and telescopes. These lens systems can have a combination of convex lenses and also
concave lenses.
(a) What is the nature (convergent/divergent) of the combination of a convex lens of power + 4D and a
concave lens of power - 2D?
(b) Calculate the focal length of a lens of power 2.5 D.
(c) Draw a ray diagram to show the nature and position of an image formed by a convex lens of power + 0.1
D, when an object is placed at a distance of 20 cm from its optical centre.
OR

How is a virtual image formed by a convex lens different from that formed by a concave lens? Under what
conditions do a convex and a concave lens form virtual images?
Set ~ 15
Series SCI/10/15
Q.P Code 10/15/15
Roll No. Candidates must write the Q.P Code
on the title page of the answer-book.

 Please check that this question paper contains 8 printed pages.


 Q.P. Code given on the right hand side of the question paper should be written on the title
page of the answer-book by the candidate.
 Please check that this question paper contains 39 questions.
 Please write down the serial number of the question in the answer-book before
attempting it.
 15 Minute time has been allotted to read this question paper. The question paper will be
distributed at 10:15 a.m From 10.15a.m to 10.30 a.m, the students will read the question
paper only and will not write any answer on the answer –book during this period

SCIENCE
Time Allowed : 3 hours Maximum Marks : 80

General Instructions:

i. This question paper consists of 39 questions in 5 sections.


ii. All questions are compulsory. However, an internal choice is provided in some questions.
A student is expected to attempt only one of these questions.
iii. Section A consists of 20 objective type questions carrying 1 mark each.
iv. Section B consists of 6 Very Short questions carrying 02 marks each. Answers to these
questions should be in the range of 30 to 50 words.
v. Section C consists of 7 Short Answer type questions carrying 03 marks each. Answers to
these questions should be in the range of 50 to 80 words.
vi. Section D consists of 3 Long Answer type questions carrying 05 marks each. Answer to
these questions should be in the range of 80 to 120 words.
vii. Section E consists of 3 source - based/case - based units of assessment of 04 marks
each with sub - parts

1
Section A 20 Marks

1. What happens in the test tube shown here?

a) H2O will produce b) SO2 will produce

c) No reaction d) FeO will produce


2. Chemically rust is:

a) Metal oxide b) Ferric oxide

c) Ferrous oxide d) Hydrated ferric oxide


3. A solution turns red litmus blue, its pH is likely to be

a) 4 b) 1

c) 10 d) 5
4. Common salt besides being used in kitchen can also be used as raw material for making ?
i. Washing soda
ii. Bleaching powder
iii. Baking soda
iv. Slaked lime

a) All of these b) A and D

c) A and C d) B and D
5. The image shows a substance X formed when potassium iodide and lead nitrate react together. What is X?
a) Lead iodide, Precipitate b) No change

c) Lead sulphate, Gas d) Potassium nitrate, Precipitate


6. Plaster of Paris hardens by

a) Releasing out a molecule of water b) Changing into CaCO3

c) Combining with water d) Passing of CO2 gas

7. Bleaching powder is a

a) white crystalline solid b) pale yellow powder

c) greyish white powder d) transparent crystalline solid


8. To show experimentally that CO2 is given out during respiration, student must use

a) KOH solution b) C2H5OH

c) Ca(OH)2 solution d) Al(OH)3 solution

9. In the excretory system of human beings, some substances in the initial filtrate such as glucose, amino acids,
salts and water are selectively reabsorbed in

a) Ureter b) Urethra

c) Urinary bladder d) Nephron


10. The shape of guard cells changes due to change in the

a) amount of water in cells b) temperature of cells

c) position of nucleus in the cells d) protein composition of cells


11. The gap between two neurons is called a:

a) impulse b) dendrite

c) axon d) synapse
12. The inner lining of stomach is protected by one of the following from hydrochloric acid. Choose the correct one

a) Salivary amylase b) Pepsin

c) Mucus d) Bile
13. When a ray of light travelling in glass enters into water obliquely:

a) it is refracted towards the normal b) it goes along the normal

c) it is not refracted at all d) it is refracted away from the normal


14. An electric motor takes 5 ampere current from a 220 volt supply line. The power of the motor and electric
energy consumed by it in 2:

a) 2.4 kWh b) 2.7 kWh

c) 2.2 kWh d) 2.1 kWh


15. To make all connections airtight, in the experiment, we use
a) Either vaseline or wax b) Wax

c) Vaseline d) Lubricant oil


16. Which part of the brain is concerned with muscular coordination in the body?

a) Temporal lobe b) Cerebellum

c) Pons d) Parietal lobe


17. Assertion (A): The food items containing oil and fat are flushed with nitrogen.
Reason (R): Oil and fat become rancid on oxidation which has the bad taste and smell.

a) Both A and R are true and R is the correct b) Both A and R are true but R is not the
explanation of A. correct explanation of A.

c) A is true but R is false. d) A is false but R is true.


18. Assertion (A): Plant hormones are chemicals produced in plants which help to coordinate growth, development
and response to stimulus and environment.
Reason (R): Abscisic acid is a plant hormone that promotes cell division.

a) Both A and R are true and R is the correct b) Both A and R are true but R is not the
explanation of A. correct explanation of A.

c) A is true but R is false. d) A is false but R is true.


19. Assertion (A): Tungsten metal is used for making filaments of incandescent lamps.
Reason (R): The melting point of tungsten is very low.

a) Both A and R are true and R is the correct b) Both A and R are true but R is not the
explanation of A. correct explanation of A.

c) A is true but R is false. d) A is false but R is true.


20. Assertion (A): Haemodialysis can save the life of patients with kidney failure.
Reason (R): Waste products like urea can be removed from the blood by haemodialysis.

a) Both A and R are true and R is the correct b) Both A and R are true but R is not the
explanation of A. correct explanation of A.

c) A is true but R is false. d) A is false but R is true.


Section B 12 Marks

21. Why does magnesium powder reacts faster than magnesium ribbon?
22. What type of respiration takes place in human muscles during vigorous exercise and why?

23. Draw a well labeled diagram of:


i. Chloroplast
ii. Open and closed stomata
OR
i. What is double circulation?
ii. Why is the separation of the right side and the left side of the heart useful? How does it help birds and mammals?
24. The image formed by a concave mirror is observed to be virtual, erect and larger than the object. where should
be the position of object ?Justify your answer .
25. i. If the cerebellum is not functioning properly, what activities of our body would get affected?
ii. Which part of brain controls involuntary actions?
OR
Organisms are classified on the basis of nitrogenous waste excreted by them. Explain.
26. One day when Geeta was reading newspaper she saw the picture of shortest and tallest person in the world. Next
day when she went to school she asked her teacher about this condition.
i. Which gland and hormones might be responsible for this condition?
ii. What is the cause of the condition given in the above article and give scientific terms associated with the
above two conditions?
iii. Can this disease be cured?
Section C 21 Marks
27. Give the characteristic tests for the following gases
i. CO2

ii. SO2
iii. O2
iv. H2

28. With the help of a chemical equation, explain how a soda-acid fire extinguisher helps in putting out a fire.
OR
Why are decomposition reactions called the opposite of combination reactions? Write equations for these reactions.
29. A figure given below shows a diagram of a kidney and its associated structures. The table list the percentage of
certain components found within the structures A and B.

In Structure A

Components Concentration %

Urea 0.03

Glucose 0.10

Amino acids 0.05


Salts 0.75

Proteins 8.00

In Structure B

Components Concentration %

Urea 2.00

Glucose 0.00

Amino acids 0.00

Salts 1.50

Proteins 0.00

i. Using only the information given in the tables, deduce the functions of the kidney.
ii. Explain how the proportions of components present in part B would change if a person is suffering from
diabetes mellitus.
30. Nervous and hormonal system together performs the functions of control and coordination in human beings.
Justify the statement.
31. A student wants to project the image of a candle flame on a screen 80 cm in front of a mirror by keeping the
candle flame at a distance of 20 m from its pole.
i. Which type of mirror should the student use?
ii. Find the magnificent of the image produced.
iii. Find the distance between the object and its image.
32. Five resistors are connected in a circuit as shown in figure. Find the ammeter reading when the circuit is closed.

33. If in figure, R1 = 10Ω , R2 = 40Ω , R3 = 30Ω , R4 = 20Ω , R5 = 60Ω , and a 12 V battery is connected to the

arrangement. Calculate

a. the total resistance in the circuit, and


b. the total current flowing in the circuit.
Section D 15 Marks
34. a. Write chemical equations for the following reactions:
i. Calcium metal reacts with water.
ii. Cinnabar is heated in the presence of air.
iii. Manganese dioxide is heated with aluminium powder.
b. What are alloys? List two properties of alloys.
OR
Give experiment to show that blue vitriol crystals contain water of crystallisation.
35. What are the common features between all the respiratory organs? Explain the mechanism of gaseous exchange
between tissues and blood.
OR
List the major endocrine glands and state their position in the human body.
36. What are Cartesian sign conventions used in spherical mirror ? What is the mirror formula ?
OR
a. List four characteristics of the image formed by a convex lens when an object is placed between its optical centre
and principal focus.
b. Size of the image of an object by a concave lens of focal length 20 cm is observed to be reduced to
1 rd of its size.
3

Find the distance of the object from the lens.


Section E 12 Marks
37. Read the text carefully and answer the questions:
The strength of acid and base depends on the number of H+ and the number of OH- respectively. If we take
hydrochloric acid and acetic acid of the same concentration, say one molar, then these produce different amounts
of hydrogen ions. Acids that give rise to more H+ ions are said to be strong acids, and acids that give less H+
ions are said to be weak acids. Can you now say what weak and strong bases are?

(a) Fresh milk has a pH of 6. How do you think the pH will change as it turns into curd?
(b) Is Gastric juice a weak acid?
(c) Milk of magnesia is an acid or base? For what purpose it can be used?
OR
What is the pH value of saliva after the meal?
38. Read the text carefully and answer the questions:
We have seen that the different parts of our body have specific functions. Our mouth waters when we see the
food we like without our meaning to. Our heart's beat without our thinking about it. In fact, we cannot control
these actions easily by thinking about them even if we wanted to. So, in between the simple reflex actions like
change in the size of the pupil, and the thought out actions such as moving a chair, there is another set of muscle
movements over which we do not have any thinking control. Many of these involuntary actions are controlled by
the mid-brain and hind-brain. All these involuntary actions including blood pressure, salivation and vomiting are
controlled by the medulla in the hind-brain. Think about activities like walking in a straight line, riding a
bicycle, picking up a pencil. These are possible due to a part of the hind-brain called the cerebellum. It is
responsible for the precision of voluntary actions and maintaining the posture and balance of the body. Imagine
what would happen if each of these events failed to take place if we were not thinking about it.

(a) Identify the part of the nervous system which controls the reflex action.
(b) Does reflex action involve all parts of the voluntary nervous system?
(c) Identify the part of the autonomic nervous system which controls involuntary actions.
OR
Beating of heart muscles, which type of action is this? Out of voluntary and involuntary action which is
slower?
39. Read the text carefully and answer the questions:
We know that a battery or a cell is a source of electrical energy. The chemical reaction within the cell generates
the potential difference between its two terminals that sets the electrons in motion to flow the current through a
resistor or a system of resistors connected to the battery. To maintain the current, the source has to keep
expanding its energy. Where does this energy go? A part of the source energy in maintaining the current may be
consumed for useful work (like in rotating the blades of an electric fan). The rest of the source energy may be
expended in heat to raise the temperature of the gadget. We often observe this in our everyday life. For example,
an electric fan becomes warm if used continuously for a long time, etc. On the other hand, if the electric circuit
is purely resistive, that is, a configuration of resistors only connected to a battery; the source energy continually
gets dissipated entirely in the form of heat. This is known as the heating effect of electric current. This effect is
utilized in devices such as an electric heater, electric iron, etc.

(a) Explain Joule's heating law.


(b) In practical situations, when an electric appliance is connected to a known voltage source, then how does
the heating effect of electric current can be calculated?
(c) Write the relation between heat energy produced in a conductor when a potential difference V is applied
across its terminals and a current I flows through for t.
OR
Two identical wires one of nichrome and the other of copper are connected in series and a current (I) is
passed through them. State the change observed in the temperatures of the two wires.
As Per Revised
CBSE Curriculum
2024-25
Classroom Teaching & Animated Videos Playlists

We take immense pleasure in serving you. Now, revel in our


seamless online services completely free of charge. view our animated
and classroom teaching Playlists customized for students from grade 1
to 12,Covering a wide range of subjects to enhance your
comprehension and knowledge. Simply click on the provided playlist
links to access Playlists based on the latest NCERT Syllabus for 2024-
25.
Our content includes Competency-Based Questions,
Assertion-Reason Questions, Previous Year Questions (PYQ), and
Case Study-Based Questions to enhance your learning
experience.For the most up-to-date videos, consider subscribing to
our YouTube channel at
https://www.youtube.com/@PrincipalsHandbookandDiaryadditionall Nageen Group of Schools
y, you're encouraged to join our expanding WhatsApp community
group to stay updated with the latest curriculum-related content and
updates.

We are committed to enriching your educational journey!!!


ANIMATED & CLASSROM TEACHING VIDEOS PLAYLISTS
(As per revised CBSE Curriculum– 2024-25)

ANIMATED VIDEOSPLAYLISTS (CLASS 1)


Class 1 EVS(EnglishLanguage)(CBSE) Click here for playlist
Class 1 Mathematics (EnglishLanguage)(CBSE) Click here for Playlist
Class 1 EVS (HindiLanguage)(CBSE) Click here for Playlist
Class 1 Mathematics(Hindi Language)(CBSE) Click here for Playlist

ANIMATED VIDEOS PLAYLISTS (CLASS 2)


Class 2 EVS (EnglishLanguage)(CBSE) Click here for Playlist
Class 2 Mathematics (EnglishLanguage)(CBSE) Click here for Playlist
Class 2 EVS(HindiLanguage)(CBSE) Click here for Playlist
Class 2 Mathematics (Hindi Language)(CBSE) Click here for Playlist

ANIMATED VIDEOS PLAYLISTS (CLASS 3)


Class 3 Mathematics (EnglishLanguage)(CBSE) Click here for Playlist
Class 3 EVS (EnglishLanguage)(CBSE) Click here for Playlist
Class 3 EVS (HindiLanguage)(CBSE) Click here for Playlist
Class 3 Mathematics (HindiLanguage)(CBSE) Click here for Playlist

ANIMATED VIDEOS PLAYLISTS (CLASS 4)


Class 4 Mathematics (EnglishLanguage)(CBSE) Click here for Playlist
Class 4 EVS(EnglishLanguage)(CBSE) Click here for Playlist
Class 4 Mathematics (HindiLanguage)(CBSE) Click here for Playlist
Class 4 EVS (HindiLanguage)(CBSE) Click here for Playlist

CLASSROOM TEACHING VIDEOS PLAYLISTS (CLASS 4)


Class 4 General Science (CBSE) Click here for Playlist

ANIMATED VIDEOS PLAYLISTS (CLASS 5)


Class 5 Mathematics (EnglishLanguage)(CBSE) Click here for Playlist
Class 5 Science (EnglishLanguage)(CBSE) Click here for Playlist
Class 5 Mathematics(HindiLanguage)(CBSE) Click here for Playlist
Class 5 Science (HindiLanguage)(CBSE) Click here for Playlist

CLASSROOM TEACHING VIDEOS PLAYLISTS (CLASS 5)


Class 5 General Science (CBSE) Click here for Playlist
Class 5 EVS (CBSE) Click here for Playlist
ANIMATED VIDEOS PLAYLISTS(CLASS 6)
Class 6 Mathematics (EnglishLanguage)(CBSE) Click here for Playlist
Class 6 Social Science (EnglishLanguage)(CBSE) Click here for Playlist
Class 6 Science (EnglishLanguage) (CBSE) Click here for Playlist
Class 6 Mathematics (Hindi Language)(CBSE) Click here for Playlist
Class 6 Science All Chapters (CBSE) Click here for Playlist

CLASSROOM TEACHING VIDEOS PLAYLISTS (CLASS 6)


Class 6 Mathematics (CBSE) Click here for Playlist
Class 6 Social Science (CBSE) Click here for Playlist
Class 6 Sanskrit (CBSE) Click here for Playlist
Class 6 Hindi (CBSE) Click here for Playlist
Class 6 Science (CBSE) Click here for Playlist

ANIMATED VIDEOS PLAYLISTS (CLASS 7)


Class 7 Science(CBSE) Click here for Playlist
Class 7 Mathematics(CBSE) Click here for Playlist
Class 7 Social Science(CBSE) Click here for Playlist
Class 7 Mathematics(CBSE) Click here for Playlist
Class 7 Science (CBSE) Click here for Playlist

CLASSROOM TEACHING VIDEOS PLAYLISTS (CLASS 7)


Class 7 Science (CBSE) Click here for Playlist
Class 7 Hindi (CBSE) Click here for Playlist
Class 7 Sanskrit (CBSE) Click here for Playlist
Class 7 Social Science (CBSE) Click here for Playlist
Class 7 Mathematics (CBSE) Click here for Playlist

ANIMATED VIDEOS PLAYLISTS (CLASS 8)


Class 8 Science(CBSE) Click here for Playlist
Class 8 Mathematics(CBSE) Click here for Playlist
Class 8 Social Science(CBSE) Click here for Playlist
Class 8 Mathematics(CBSE) Click here for Playlist
Class 8 Science(CBSE) Click here for Playlist

CLASSROOM TEACHING VIDEOS PLAYLISTS (CLASS 8)


Class 8 Hindi (CBSE) Click here for Playlist
Class 8 Sanskrit (CBSE) Click here for Playlist

ANIMATED VIDEOS PLAYLISTS (CLASS 9)


Class 9 Biology(CBSE) Click here for Playlist
Class 9 Physics(CBSE) Click here for Playlist
Class 9 Chemistry(CBSE) Click here for Playlist
Class 9 Social Science (CBSE) Click here for Playlist
Class 9 Mathematics (CBSE) Click here for Playlist
Class 9 Science (CBSE) Click here for Playlist

CLASSROOM TEACHING VIDEOS PLAYLISTS (CLASS 9)


Class 9 Social Science (CBSE) Click here for Playlist
Class 9 Mathematics(CBSE) Click here for Playlist
Class 9 English (CBSE) Click here for Playlist
Class 9 Hindi (CBSE) Click here for Playlist

ANIMATED VIDEOS PLAYLISTS (CLASS 10)


Class 10 Biology (CBSE) Click here for Playlist
Class 10 Physics (CBSE) Click here for Playlist
Class 10 Chemistry (CBSE) Click here for Playlist
Class 10 Social Science (CBSE) Click here for Playlist
Class 10 Mathematics(CBSE) (English Language) Click here for Playlist
Class 10 Mathematics(CBSE) (Hindi Language) Click here for Playlist
Class 10 Science(CBSE) (Hindi Language) Click here for Playlist

CLASSROOM TEACHING VIDEOS PLAYLISTS (CLASS 10)


Class 10 English (CBSE) Click here for Playlist
Class 10 Hindi (CBSE) Click here for Playlist
Class 10 Mathematics (CBSE) Click here for Playlist
Class 10 Social Science (CBSE) Click here for Playlist
Class 10 Magical Science Board Exam Preparation in 1 min (CBSE) Click here for Playlist
Class 10: Science (CBSE) Click here for Playlist

ANIMATED VIDEOS PLAYLISTS (CLASS 11)


Class 11 Physics (CBSE) (English Language) Click here for Playlist
Class 11 Chemistry (CBSE) (English Language) Click here for Playlist
Class 11 Biology (CBSE) (English Language) Click here for Playlist
Class 11 Mathematics(CBSE) (English Language) Click here for Playlist
Class 11 Accountancy (CBSE) (English Language) Click here for Playlist
Class 11 Business Studies (CBSE) (English Language) Click here for Playlist
Class 11 Statistics (CBSE) (English Language) Click here for Playlist
Class 11 Biology (CBSE) (Hindi Language) Click here for Playlist
Class 11 Mathematics (CBSE) (Hindi Language) Click here for Playlist
Class 11 Physics (CBSE) (Hindi Language) Click here for Playlist
Class 11 Chemistry (CBSE) (Hindi Language) Click here for Playlist
Class 11Micro Economy (CBSE) (English Language) Click here for Playlist

CLASSROOM TEACHING VIDEOS PLAYLISTS (CLASS 11)


Class 11Mathematics (CBSE) Click here for Playlist
Class 11 Accounts (CBSE) Click here for Playlist
Class 11 Business Studies (CBSE) Click here for Playlist
Class 11 Hindi (CBSE) Click here for Playlist
Class 11 Psychology (CBSE) Click here for Playlist
Class 11 Economics (CBSE) Click here for Playlist
Class 11 Physics (CBSE) Click here for Playlist
Class 11 Chemistry (CBSE) Click here for Playlist
Class 11 English (CBSE) Click here for Playlist
Class 11 Biology (CBSE) Click here for Playlist
Class 11 Biology Shorts (CBSE) Click here for Playlist

ANIMATED VIDEOS PLAYLISTS (CLASS 12)


Class 12 Physics (CBSE) Click here for Playlist
Class 12 Chemistry (CBSE) Click here for Playlist
Class 12 Biology(CBSE) Click here for Playlist
Class 12 Macro Economy (CBSE) Click here for Playlist
Class 12Economic (CBSE) Click here for Playlist
Class 12 Mathematics (CBSE) Click here for Playlist
Class 12 Accountancy (CBSE) Click here for Playlist
Class 12 Business Studies (CBSE) Click here for Playlist
Class 12 Physics (CBSE) Click here for Playlist
Class 12 Mathematics (CBSE) Click here for Playlist
Class 12 Biology (CBSE) Click here for Playlist
Class 12 Chemistry (CBSE) Click here for Playlist

CLASSROOM TEACHING VIDEOS PLAYLISTS (CLASS 12)


Class 12 CHEMISTRY (CBSE) Click here for Playlist
Class 12 Business Studies (CBSE) Click here for Playlist
Class 12 Hindi (CBSE) Click here for Playlist
NEET Biology in 1 min Click here for Playlist
Class 12 History (CBSE) Click here for Playlist
Class 12 Political Science (CBSE) Click here for Playlist
Class 12 Physics (CBSE) Click here for Playlist
Class 12 Biology (CBSE) Click here for Playlist
Class 12 : Accounts (CBSE) Click here for Playlist
JOIN SCHOOL OF EDUCATORS
WHSTSAPP & TELEGRAM
GROUPS FOR FREE
We are thrilled to introduce the School of Educators WhatsApp Group, a platform designed exclusively
for educators & Students to enhance your teaching & Learning experience and elevate student learning
outcomes. Here are some of the key benefits you can expect from joining our group:

BENEFITS OF SOE WHATSAPP GROUPS


• Abundance of Content: Members gain access to an extensive repository of educational materials
tailored to their class level. This includes various formats such as PDFs, Word files, PowerPoint
presentations, lesson plans, worksheets, practical tips, viva questions, reference books, smart
content, curriculum details, syllabus, marking schemes, exam patterns, and blueprints. This rich
assortment of resources enhances teaching and learning experiences.
• Immediate Doubt Resolution: The group facilitates quick clarification of doubts. Members can seek
assistance by sending messages, and experts promptly respond to queries. This real-time
interaction fosters a supportive learning environment where educators and students can exchange
knowledge and address concerns effectively.
• Access to Previous Years' Question Papers and Topper Answers: The group provides access to
previous years' question papers (PYQ) and exemplary answer scripts of toppers. This resource is
invaluable for exam preparation, allowing individuals to familiarize themselves with the exam
format, gain insights into scoring techniques, and enhance their performance in assessments.
• Free and Unlimited Resources: Members enjoy the benefit of accessing an array of educational
resources without any cost restrictions. Whether its study materials, teaching aids, or assessment
tools, the group offers an abundance of resources tailored to individual needs. This accessibility
ensures that educators and students have ample support in their academic endeavors without
financial constraints.
• Instant Access to Educational Content: SOE WhatsApp groups are a platform where students&
teachers can access a wide range of educational content instantly. This includes study materials,
notes, sample papers, reference materials, and relevant links shared by group members and
moderators.
• Timely Updates and Reminders: SOE WhatsApp groups serve as a source of timely updates and
reminders about important dates, exam schedules, syllabus changes, and academic events.
Teachers &Students can stay informed and well-prepared for upcoming assessments and activities.
• Interactive Learning Environment: Teachers &Students can engage in discussions, ask questions,
and seek clarifications within the group, creating an interactive learning environment. This fosters
collaboration, peer learning, and knowledge sharing among group members, enhancing
understanding and retention of concepts.
• Access to Expert Guidance: SOE WhatsApp groups are moderated by subject matter experts,
teachers, or experienced educators. Students can benefit from their guidance, expertise, and
insights on various academic topics, exam strategies, and study techniques.
• Sharing of Study Tips and Strategies: Group members often share valuable study tips, exam
strategies, and time management techniques that have proven effective for them. Students can
learn from each other's experiences and adopt helpful strategies to optimize their study routines
and improve their academic performance.
• Availability of Practice Materials: SOE WhatsApp groups frequently share practice questions,
quizzes, and mock tests to help students assess their understanding and practice exam-oriented
questions. This allows students to gauge their progress, identify areas of improvement, and refine
their exam preparation accordingly.
• Peer Support and Motivation: Being part of an SOE WhatsApp group provides students with a
supportive community of peers who share similar academic goals and challenges. Group members
can offer encouragement, motivation, and moral support to each other, especially during stressful
periods such as exams.
• Convenience and Accessibility:SOE WhatsApp is a widely used messaging platform accessible on
smartphones, making educational content and discussions easily accessible anytime, anywhere.
Students can review study materials, participate in discussions, and seek help conveniently from
their mobile devices.

Join the School of Educators WhatsApp Group today and unlock a world of resources, support, and
collaboration to take your teaching to new heights. To join, simply click on the group links provided
below or send a message to +91-95208-77777 expressing your interest.

Together, let's empower ourselves & Our Students and inspire the next generation of learners.

Best Regards,

Team
School of Educators
SCHOOL OF EDUCATORS WHATSAPP GROUPS
(For Teachers Only)
You will get Pre- Board Papers PDF, Word file, PPT, Lesson Plan, Worksheet, practical tips and Viva
questions , reference books , smart content , curriculum , syllabus , marking scheme , toppers answer
scripts , revised exam pattern , revised syllabus , Blue Print etc. here .Join Your Subject / Class WhatsApp
Group.

Kindergarten to Class XII (For Teachers Only)

Kindergarten Class 1 Class 2 Class 3

Class 4 Class 5 Class 6 Class 7

Class 8 Class 9 Class 10 Class 11 (Science )

Class 11 (Commerce) Class 11 (Humanities) Class 12 (Science) Class 12 (Commerce)

Class 12 (Humanities) Artificial Intelligence


(VI TO VIII)
Subject Wise Secondary and Senior Secondary Groups (IX & X)
Secondary Groups (IX & X)

SST Mathematics Science English

Hindi Information Technology Artificial Intelligence

Senior Secondary Groups (XI & XII)

Physics Chemistry English Mathematics

Biology Accountancy Economics BST

History Geography sociology Hindi Elective

Hindi Core Home Science Sanskrit psychology

Political science Painting vocal Music Comp. Science

IP physical Education APP. Mathematics IIT/NEET

Legal studies Entrepreneurship French Teachers Jobs

Artificial Intelligence
SCHOOL OF EDUCATORS WHATSAPP GROUPS
(For Students Only)

Class 1 class 2 class 3 class 4 class 5

class 6 class 7 class 8 class 9 class 10

Class 11(Science) Class 11(Com) Class 11(Hum) Class 12 (Sci) Class12 (Com)

Class 12 (Hum) Artificial Intelligence


(VI TO VIII)

Subject Wise Secondary and Senior Secondary Groups (IX & X)

Secondary Groups (IX & X)

SST Mathematics English


Science

Hindi B Hindi A IT Artificial Intelligence


Senior Secondary Groups (XI & XII)

Physics Chemistry Biology English

Accountancy Business Studies Economics Mathematics

History Geography Sociology Hindi Elective

Hindi Core Home Science Sanskrit Psychology

Political Science Painting Music Computer Science

IP Physical Education App. Mathematics IIT/NEET

Legal Studies Entrepreneurship French CUET

Artificial Intelligence

Rules & Regulations of the Group

• No introduction
• No Good Morning/Any wish type message
• No personal Chats & Messages
• No Spam
• You can also ask your difficulties here.

Just get learning resources & post learning resources. Helpline number only WhatsApp: +91-95208-77777
SOE CBSE Telegram Groups (Kindergarten to Class XII)

Kindergarten

All classes Class 1 Class 2 Class 3

Class 4 Class 5 Class 6 Class 7

Class 8 Class 9 Class 10 Class 11(Sci)

Class 11 (Com) Class 11 (Hum) Class 12 (Sci) Class 12 (Com)

Class 12 (Hum) JEE/NEET CUET NDA,OLYMPIAD,NTSE

Principal Professional Teachers Professional Project File Group


Group Group

SOE CBSE Telegram Channels (Kindergarten to Class XII)

Kindergarten Class I Class II Class III

Class IV Class V Class VI Class VII

Class VIII Class IX Class X Class XI (Sci)

Class XI (Hum) Class XI (Com) Class XII (Sci) Class XII (Hum)

Class XII (Com) JEE/NEET CUET NDA/OLYMPIAD/NTSE

Rules & Regulations of the Groups & Channel

• No introduction
• No Good Morning/Any wish type message
• No personal Chats & Messages
• No Spam
• You can also ask your difficulties here.

Just get learning resources & post learning resources. Helpline number only WhatsApp: +91-95208-77777
Available Resources on YouTube

 Enjoy animated videos covering all subjects from Kindergarten to Class 12, making learning fun for
students of all ages.
 Explore classroom teaching videos for grades 6 to 12, covering various subjects to enhance
understanding and knowledge.
 Access the most important questions and previous year's question papers (PYQ) to excel in exams and
assessments.
 Stay up-to-date with the latest CBSE Curriculum for 2023-24 with our videos aligned to the current
syllabus.
 Get informed about CBSE updates and circulars through our dedicated videos.
 Improve pronunciation skills and expand vocabulary with our "Word of the Day" series and other
language-related content and many more……….

Don't miss out on these valuable resources; subscribe to our channel now!

You might also like